You are on page 1of 91

2017-18 100 &

op kers
Class 11 T
By E ran culty
-JE Fa r
IIT enior emie .
S fP r es
o titut
Ins

MATHEMATICS
FOR JEE MAIN & ADVANCED
SECOND
EDITION

Exhaustive Theory
(Now Revised)

Formula Sheet
9000+ Problems
based on latest JEE pattern

2500 + 1000 (New) Problems


of previous 35 years of
AIEEE (JEE Main) and IIT-JEE (JEE Adv)

5000+Illustrations and Solved Examples


Detailed Solutions
of all problems available

Topic Covered Plancess Concepts


Tips & Tricks, Facts, Notes, Misconceptions,
Complex Number Key Take Aways, Problem Solving Tactics

PlancEssential
Questions recommended for revision
6. COMPLEX NUMBER

1. INTRODUCTION
The number system can be briefly summarized as N ⊂ W ⊂ I ⊂ Q ⊂ R ⊂ C , where N, W, I, Q, R and C are the
standard notations for the various subsets of the numbers belong to it.
N - Natural numbers = {1, 2, 3 …. n}
W - Whole numbers = {0, 1, 2, 3 …. n}
I - Integers = {….2, -1, 0, 1, 2 …..}
1 3 
Q – Rational numbers =  , ..... 
2 5 
IR – Irrational numbers = { 2, 3, π}
C – Complex numbers
A complex number is generally represented by the letter “z”. Every complex number z, can be written as, z= x + i y
where x, y ∈ R and i= −1 .
x is called the real part of complex number, and
y is the imaginary part of complex number.
Note that the sign + does not indicate addition as normally understood, nor does the symbol “i” denote a number.
These are parts of the scheme used to express numbers of a new class and they signify the pair of real numbers
(x, y) to form a single complex number. Y
Imaginary axis P(x,y)

X’ X
O Real axis

Y’
Figure 6.1: Representation of a complex number on a plane

Swiss-born mathematician Jean Robert Argand, after a systematic study on complex numbers, represented every
complex number as a set of ordered pair (x, y) on a plane called complex plane.
All complex numbers lying on the real axis were called purely real and those lying on imaginary axis as purely
imaginary.
Hence, the complex number 0 + 0i is purely real as well as purely imaginary but it is not imaginary.
6 . 2 | Complex Number

Note
Purely real if y = 0

z = x+iy is Purely imaginary if x = 0

x0
Complex number if
y0

Figure 6.2: Classification of a complex number

(a) The symbol i combines itself with real number as per the rule of algebra together with

i2 = −1 ; i3 = −i ; i4 = 1 ; i2014 = −1 ; i2015 = −i and so on.


In general, i4n = 1 , i4n+1 = i , i4n+ 2 = −1 , i4n+3 = −i , n ∈ I and i4n + i4n+1 + i4n+ 2 + i4n+3 =
0
Hence, 1 + i1 + i2 + ……. + i2014 + i2015 = 0
(b) The imaginary part of every real number can be treated as zero. Hence, there is one-one mapping between
the set of complex numbers and the set of points on the complex plane.

PLANCESS CONCEPTS
Complex number as an ordered pair: A complex number may also be defined as an ordered pair of real
numbers and may be denoted by the symbol (a, b) . For a complex number to be uniquely specified, we
need two real numbers in a particular order.
Vaibhav Gupta (JEE 2009, AIR 54)

2. ALGEBRA OF COMPLEX NUMBERS


(a) Addition: (a + ib) + (c + id) = (a + c) + i(b + d)
(b) Subtraction: (a + ib) − (c + id) = (a − c) + i(b − d)
(c) Multiplication: (a + ib) (c + id) = (ac − bd) + i (ad + bc)
(d) Reciprocal: If at least one of a, b is non-zero, then the reciprocal of a + ib is given by
1 a − ib a b
−i
a + ib (a + ib) (a − ib) 2 2
a + b2
2
= = a +b

(e) Quotient: If at least one of c, d is non-zero, then quotient of a + ib and c + id is given by

a + ib (a + ib) (c − id) (ac + bd) + i (bc − ad) ac + bd bc − ad


= = = +i
c + id (c + id) (c − id) 2
c +d 2
c2 + d2 c2 + d2

(f) Inequality in complex numbers is not discussed/defined. If a + ib > c + id is meaningful only if b = d = 0.


However, equalities in complex numbers are meaningful. Two complex numbers z1 and z2 are said to be equal
if Re (z1 ) = Re(z 2 ) and Im (z1 ) = Im(z 2 ). (Geometrically, the position of complex number z1 on complex plane)

(g) In real number system if p2 + q2 = 0 implies, p= 0= q . But if z1 and z 2 are complex numbers then z12 + z 22 =
0
does not imply z=1 z=2 0 . For e.g. z1 = i and z 2 = 1 .
However if the product of two complex numbers is zero then at least one of them must be zero, same as in
case of real numbers.
x, if x ≥ 0
(h) In case x is real, then | x | =  but in case of complex number z, | z | means the distance of the
point z from the origin. −x, if x < 0
M a them a tics | 6.3

PLANCESS CONCEPTS

• The additive inverse of a complex number z= a + ib is −z (i.e. − a − ib) .


1
• For every non-zero complex number z, the multiplicative inverse of z is .
z
• | z | ≥ Re(z) ≥ Re(z) and | z | ≥ Im(z) ≥ Im(z) .

z
• is always a uni-modular complex number if z ≠ 0 .
|z|
Vaibhav Krishnan (JEE 2009, AIR 22)

Illustration 1: Find the square root of 5 + 12i.  (JEE MAIN)

Sol: z= 5 + 12i
Let the square root of the given complex number be a + ib. Use algebra to simplify and get the value of a and b.
Let its square root = a + ib ⇒ 5 + 12i = a2 − b2 + 2abi
⇒ a2 − b2 =
5 … (i)
12 
⇒ 2ab = … (ii)
⇒ (a2 + b2 ) 2 = (a2 − b2 ) 2 + 4a2b2 ⇒ (a2 + b2 ) 2 = 25 + 144 =169 ⇒ a2 + b2 =
13  … (iii)
2 2
(i) + (iii) ⇒ 2a =
18 ⇒ a =
9 ⇒ a =± 3
If a = 3 ⇒ b =
2 If a = −3 ⇒ b =−2
∴ Square root = 3 + 2i, −3 − 2i ∴ Combined form ± (3 + 2i)

If z (x, y) ∈ C . Find z satisfying z 2 × (1 + i) = ( −7 + 17i) .


Illustration 2: = (JEE MAIN)

Sol: Algebra of Complex Numbers.


(x + iy) 2 (1 + i) =−7 + 17i
⇒ (x2 − y 2 + 2xyi) (1 + i) =−7 + 17i ; x2 − y 2 + i(x2 − y 2 ) + 2xyi − 2xy =−7 + 17i
⇒ (x2 − y 2 − 2xy) + i (x2 − y 2 + 2xy) =
−7 + 17i ⇒ x= 3, y= 2 ⇒ x =−3, y =−2
⇒ z =−3 + i ( −2 ) =−3 − 2i

Illustration 3: If x2 + 2(1 + 2i) x − (11 + 2i) =


0 . Solve the equation.  (JEE ADVANCED)

Sol: Use the quadratic formula to find the value of x.


−2 (1 + 2i) ± 4 − 16 + 16i + 44 + 8i
∴x =
2

⇒ 2x =( −2)(1 + 2i) ± 32 + 24i

⇒ x =( −1)(1 + 2i) ± 8 + 6i = −1 − 2i ± (3 + i) ; x = 2 – i, –4 – 3i
6 . 4 | Complex Number

Illustration 4: If f(x) = x 4 − 4x3 + 4x2 + 8x + 44 . Find f(3 + 2i) .  (JEE ADVANCED)

Sol: Let x = 3+2i, and square it to form a quadratic equation. Then try to represent f(x) in terms of this quadratic.
x= 3 + 2i
⇒ (x − 3) 2 =−4 ⇒ x2 − 6x + 13 =
0
x 4 − 4x3 + 4x2 + 8x + 44
= x2 (x2 − 6x + 13) + 2x3 − 9x2 + 8x + 44
= x2 (x2 − 6x + 13) + 2(x3 − 6x2 + 13x) + 3(x2 − 6x + 13) + 5
⇒ f(x) ⇒ f(x) =
5

3. IMPORTANT TERMS ASSOCIATED WITH COMPLEX NUMBER


Three important terms associated with complex number are conjugate, modulus and argument.
(a) Conjugate: If z = x + iy then its complex conjugate is obtained by changing the sign of its imaginary part and
denoted by z i.e. z = x – iy (see Fig 6.3).
The conjugate satisfies following basic properties
(i) z + z = 2Re (z) Y
(ii) z − z = 2i Im (z)
z = x+iy
z x2 + y 2
(iii) z =
(iv) If z lies in 1st quadrant then z lies in 4th quadrant and – z in
the 2nd quadrant. X’ X

(v) If x + iy = f (a + ib) then x – iy = f (a – ib)


For e.g. If (2 + 3i)3 = x + i y then (2 – 3i)3 = x – iy Conjugate of z , z = x+iy
Image of z in x-axis
and, sin(α + iβ) = x + iy ⇒ sin(α − iβ) = x – iy
Y’
(vi) z + z = 0 ⇒ z is purely imaginary
Figure 6.3: Conjugate of a complex number
(vii) z − z =0 ⇒ z is purely real

(b) Modulus: If P denotes a complex number z = x + iy then, OP = | z | = x2 + y 2 . Imaginary axis


Geometrically, it is the distance of a complex number from the origin.
Hence, note that | z | ≥ 0 , | i | = 1 i.e. −1 = 1. P(x, y)
r = lzl
All complex number satisfying |z| = r lie on the circle having centre at origin and

radius equal to ‘r’.
O Real axis
(c) Argument: If OP makes an angle θ (see Fig 6.4) with real axis in anticlockwise Figure 6.4: Modulus of a
sense, then θ is called the argument of z. General values of argument of z are complex number
given by 2nπ + θ, n ∈ I . Hence any two successive arguments differ by 2π .
Note: A complex number is completely defined by specifying both modulus and argument. However for the
complex number 0 + 0i the argument is not defined and this is the only complex number which is completely
defined by its modulus only.

(i) Amplitude (Principal value of argument): The unique value of θ such that −π < θ ≤ π is called
principal value of argument. Unless otherwise stated, amp z refers to the principal value of argument.
(ii) Least positive argument: The value of θ such that 0 < θ ≤ 2π is called the least positive argument.
y
If φ =tan−1 .
x
M a them a tics | 6.5

PLANCESS CONCEPTS

y y
• If x > 0, y > 0 (i.e. z is in first quadrant), then arg z = θ = tan−1 .
x -
y
• If x < 0, y > 0 (i.e. z is in 2nd quadrant, then arg z = θ = π − tan−1
x

y x
• If x < 0, y < 0 (i.e. z is in 3rd quadrant), then arg z = θ = − π + tan−1 O
x -+ -

y
• If x > 0, y < 0 (i.e. z is in 4th quadrant), then arg z = θ = − tan−1
x Figure 6.5

0, if x > 0
• If y = 0 (i.e. z is on the X-axis), then arg (x + i0) = 
 π, if x < 0
π
 , if y > 0
• If x = 0 (i.e. z is on the Y-axis), then arg (0 + iy) =  2
 3π , if y < 0
 2

Shrikant Nagori (JEE 2009, AIR 30)

Illustration 5: For what real values of x and y, are −3 + ix2 y and x2 + y + 4i complex conjugate to each other?
 (JEE MAIN)

Sol: As −3 + ix2 y and x2 + y + 4i are complex conjugate of each other. Therefore −3 + ix2 y = x2 + y + 4i .
−3 + ix2 y = x2 + y − 4i
Equating real and imaginary parts of the above question, we get
– 3 = x2 + y ⇒ y =−3 − x2  … (i)
and x2 y = – 4  … (ii)
2
Putting the value of y = −3 − x from (i) in (ii), we get
−3 ± 9 + 16 −3 ± 5 2 −8
x2 ( −3 − x2 ) = – 4 0 ⇒ x2 =
⇒ x 4 + 3x2 − 4 = = = , =1, − 4
2 2 2 2
∴ x2 =1 ⇒ x =±1
Putting value of x = ± 1 in (i), we get y = – 3 – (1)2 = – 3 – 1 = – 4
Hence, x = ± 1 and y = – 4.

1+i 1−i
Illustration 6: Find the modulus of − . (JEE MAIN)
1−i 1+i

Sol: As | z | = x2 + y 2 , using algebra of complex number we will get the result.

1+i 1−i (1 + i)(1 + i) (1 − i)(1 − i)


Here, we have − = −
1−i 1+i (1 − i) (1 + i) (1 + i) (1 − i)

1 + i2 + 2i 1 + i2 − 2i 1 − 1 + 2i 1 − 1 − 2i 2i ( −2i) 1+i 1−i


= − = − = − = i + i = 2i, ∴ ⇒ − =| 2i | =2 .
1+1 1+1 2 2 2 2 1−i 1+i
6 . 6 | Complex Number

Illustration 7: Find the locus of z if | z – 3 | = 3| z + 3|.  (JEE MAIN)

Sol: Simply substituting z = x + iy and by using formula | z | = x2 + y 2 we will get the result.
Let z = x + iy
| x + iy – 3 | = 3 | x + iy + 3 | | (x – 3) + iy | = 3 | (x + 3) + iy |

(x − 3) 2 + y 2 = 3 (x + 3) 2 + y 2 ; (x − 3) 2 + y 2 = 9(x + 3) 2 + 9y 2 .

β−α
Illustration 8: If α and β are different complex numbers with | β | = 1, then find .  (JEE ADVANCED)
1 − αβ

β−α
Sol: By using modulus and conjugate property, we can find out the value of .
1 − αβ

We have, | β | = 1 ⇒ | β | 2 = 1 ⇒ ββ = 1

Now,
β−α
1 − αβ
=
β−α
ββ − αβ
=
β−α
β (β − α )
=
|β−α|
|β||β−α |
=
1
|β|
= 1. {as | x + iy | =| x + iy |}

Illustration 9: Find the number of non-zero integral solution of the equation | 1 − i | x =.
2x (JEE ADVANCED)

Sol: As | z | = x2 + y 2 , therefore by using this formula we can solve it.

We have, | 1 − i | x =
2x
x x

( ) x
x
⇒  12 + 12  =2 x ⇒ = 2x 2 2x
⇒ 22 = ⇒ 0 ⇒ x =.
= 0
  2
∴ The number of non zero integral solution is zero.

a + ib a2 + b2
Illustration 10: If = p + iq. Prove that = p2 + q2 . (JEE MAIN)
c + id c + d2
2

a + ib
Sol: Simply by obtaining modulus of both side of = p + iq .
c + id
a + ib
We have, = p + iq
c + id

a + ib a2 + b2 a + ib a2 + b2
= ⇒ p + iq = p2 + q2 ; = | p + iq | ⇒ p2 + q2 .
=
c + id c2 + d2 c + id c2 + d2

x y
Illustration 11: If (x + iy) 1/3 = a + ib. Prove that + = 4 (a2 − b2 ) . (JEE ADVANCED)
a b
Sol: By using algebra of complex number. We have, (x + iy) 1/3 = a + ib
x + iy = (a + ib) 3 =a3 + i3b3 + 3a2ib + 3a(ib)2 = a3 − b3i + 3a2bi − 3ab2
x + iy = (a3 − 3ab2 ) + (3a2b − b3 )i ; x = a3 − 3ab2 = a (a2 − 3b2 ) ; y = 3a2b − b3
x y
+ = 4 (a2 − b2 ) .
a b
M a them a tics | 6.7

4. REPRESENTATION OF COMPLEX NUMBER

4.1 Graphical Representation


Every complex number x + iy can be represented in a plane as a point P (x, y). X-coordinate of point P represents
the real part of the complex number and y-coordinate represents the imaginary part of the complex number.
Complex number x + 0i (real number) is represented by a point (x, 0) lying on the x-axis. Therefore, x-axis is called
the real axis. Similarly, a complex number 0 + iy (imaginary number) is represented by a point on y-axis. Therefore,
y-axis is called the imaginary axis.
The plane on which a complex number is represented is called complex Y
Imaginary axis
number plane or simply complex plane or Argand plane (see Fig 6.6). The
figure represented by the complex numbers as points in a plane is known

lm(z) > 0
as Argand Diagram.

4.2 Algebraic Form Re(z) < 0 Re(z) > 0


X’ X
y O Real axis , lm(z) = 0
If z = x + iy; then x2 + y 2 ; z= x − iy , and θ =tan−1  
|z|=
x

lm(z) < 0
Generally this form is useful in solving equations and in problems
involving locus.

4.3 Polar Form Y’


Figure 6.6: Graphical representation
Figure 6.7 shows the components of a complex number along the x and
y-axes respectively. Then
z = x + iy = r (cos θ + isin θ) = r cis θ where | z | = r ; amp z = θ .
Aliter: z = x+iy Y
 
x y
⇒ z= x2 + y 2  +i 
 x2 + y 2 x + y2
2  z(x, y) = x+iy
 

z | z | ( cos θ + i sin=
θ ) r cis θ
r

= r sin

Note: (a) (cis α ) (cis β=
) cis(α + β) X
O r cos
(b) (cis α ) (cis( −β=
)) cis(α − β)
1 Figure 6.7: Polar form
−1
(c) = (cis α ) = cis( −α )
(cis α )

PLANCESS CONCEPTS

The unique value of θ such that −π < θ ≤ π for which=x r cos θ &=
y r sin θ is r (cos θ + i sin θ)
known as the principal value of the argument.
The general value of argument is (2nπ + θ) , where n is an integer and θ is the
principal value of arg (z). While reducing a complex number to polar form, we r c i s θ
always take the principal value. Figure 6.8
The complex number z = r (cos θ + isin θ) can also be written as r cis θ .
Nitish Jhawar (JEE 2009, AIR 7)
6 . 8 | Complex Number

4.4 Exponential Form


Euler’s formula, named after the famous mathematician Leonhard Euler, states that for any real number x,
eix cos x + isinx .
=
Hence, for any complex number
= z r (cos θ + isin θ) , z = reiθ is the exponential representation.
eix + e−ix eix − e−ix
Note: (a) cos x = and sin x = are known as Eulers identities.
2 2i
ex + e− x
(b) cos ix = = cos hx is always positive real ∀ x ∈ R and is > 1.
2
ex − e− x
and, sin ix = i = i sin hx is always purely imaginary.
2

4.5 Vector Representation


The knowledge of vectors can also be used to represent a complex number Y

z = x + iy. The vector OP , joining the origin O of the complex plane to the P (x, y), z = x+iy
point P (x, y), is the vector representation of the complex number z=x+iy,
 
(see Fig 6.9). The length of the vector OP , that is, | OP | is the modulus of z. OP

The angle between the positive real axis and the vector OP , more exactly, the
angle through which the positive real axis must be rotated to cause it to X’
O
X

have the same direction as OP (considered positive if the rotation is Y’
counter-clockwise and negative otherwise) is the argument of the complex Figure 6.9 Vector representation
number z.

 1  1
Illustration 12: Find locus represented by Re   < .  (JEE MAIN)
 x + iy  2
Sol: Multiplying numerator and denominator by x − iy.

 1  1  x − iy  1
We have, Re   < Re  <
 x + iy  2  x2 + y 2  2
 
x 1
⇒ < ⇒ x2 + y 2 − 2x > 0
2
x +y 2 2

Locus is the exterior of the circle with centre (1, 0) and radius = 1.

6π 6π
Illustration 13: If z = 1 + cos + isin . Find r and amp z. (JEE MAIN)
5 5

Sol: By using trigonometric formula we can reduce given equation in the form of
= z r ( cosθ + i sinθ ) .
3π 3π 3π 3π  3π 3π 
z = 2cos2 + 2isin cos = 2cos cos + isin 
5 5 5 5  5 5 

2π  2π 2π  2π  2π 2π  2π 2π
= −2cos  − cos + isin  = 2cos cos − isin  Hence, | z | = 2cos ; amp z = −
5  5 5  5  5 5  5 5
M a them a tics | 6.9

1
Illustration 14: Show that the locus of the point P(ω)denoting the complex number z + on the complex plane is
a standard ellipse where | z | = a, where a ≠ 0, 1. z (JEE ADVANCED)

Sol: Here consider w = x + iy and z = α + iβ and then solve this by using algebra of complex number.
1
Let w = z + where z = α + iβ , α2 + β2 = a2 (as | z | = a)
z
1 α − iβ  α  β   1   1 
x + iy = α + iβ + = α + iβ + =  α +  + i  β −  ∴ x =α  1 +  ; y =β 1 − 
α + iβ 2
α +β 2
 2
a   2
a   2
a   a2 

x2 y2 x2 y2
∴ + = α2 + β2 = a2 ; ∴ + = 1.
2 2 2 2
 1   1   1  1
1 + 2  1 − 2  a +  a − 
 a   a   a  a

5. IMPORTANT PROPERTIES OF CONJUGATE, MODULUS AND ARGUMENT


For z, z1and z 2 ∈ C ,

(a) Properties of Conjugate:

(i) z + z = 2Re (z)


(ii) z − z = 2i Im (z)
(iii) (z) = z

(iv) z1 + z 2 = z1 + z2

(v) z1 − z 2 = z1 − z2
z1 z 2 = z1 · z2
(vi)
 z1  z1
(vii)   = ; z2 ≠ 0
 z 2  z2

(b) Properties of Modulus:

(i) | z | ≥ 0 ; | z | ≥ Re(z) ; | z | ≥ Im(z) ;| z | =


| z |=
| −z |
1
(ii) zz = | z | 2 ; if | z | = 1, then z =
z
(iii) | z1 z 2 | = | z1 | · | z 2 |

z1 | z1 |
(iv) = , z2 ≠ 0
z1 | z2 |
n n
(v) | z | = | z |
2
(vi) | z1 + z 2 | + | z1 − z=
2 |
2
2 | z1 | 2 + | z 2 | 2 
 
(vii) | z1 | − | z 2 | ≤ | z1 + z 2 | ≤ | z1 | + | z 2 | [Triangle Inequality]
6 . 1 0 | Complex Number

(c) Properties of Amplitude:

(i) amp (z1 ·=


z 2 ) amp z1 + amp z 2 + 2kπ, k ∈ I

 z1 
(ii) amp =  amp z 1 − amp z 2 + 2kπ, k ∈ I
 z2 

(iii) amp
= (zn ) n amp(z) + 2kπ , where the value of k should be such that RHS lies in ( −π, π]

Based on the above information, we have the following

•• | Re(z) | + | Im(z) | ≤ 2 | z |

•• | z1 | − | z 2 | ≤ z1 − z 2 ≤ z1 + z 2 . Thus | z1 | + | z 2 | is the greatest possible value of | z1 + z 2 |

and | z1 | − | z 2 | is the least possible value of | z1 + z 2 | .

1 a + a2 + 4 −a + a2 + 4
•• If z + = a, the greatest and least values of |z| are respectively and .
z 2 2

•• | z1 + z12 − z 22 | + | z 2 − z12 − z 22 | = | z1 + z 2 | + | z1 − z 2 |

•• If z1 =z 2 ⇔ | z1 | =| z 2 | and arg z1 =argz 2

•• | z1 + z 2 | = | z1 | + | z 2 | ⇔ arg (z1 ) = arg(z 2 ) i.e. z1 and z 2 are parallel.

•• | z1 + z 2 | =| z1 | + | z 2 | ⇔ arg (z1 ) − arg(z 2 ) =2nπ, where n is some integer.

•• | z1 − z 2 | =| z1 | − | z 2 | ⇔ arg (z1 ) − arg(z 2 ) =2nπ, where n is some integer.


π
•• | z1 + z 2 | = | z1 − z 2 | ⇔ arg (z1 ) − arg(z 2 ) = (2n + 1) , where n is some integer.
2
If | z1 | ≤ 1, | z 2 | ≤ 1, then | z1 + z 2 | ≤ ( | z1 | − | z 2 | ) + ( arg (z1 ) − arg(z 2 ) ) , and
2 2 2
••
2.
| z1 + z 2 | 2 ≥ ( | z1 | + | z 2 | ) − ( arg (z1 ) − arg(z 2 ) )
2

z  z
Illustration 15: If z1= 3 + 5i and z 2= 2 − 3i , then verify that  1  = 1  (JEE MAIN)
z  z
 2 2

Sol: Simply by using properties of conjugate.

z1 3 + 5i (3 + 5i) (2 + 3i) 6 + 9i + 10i + 15i2 6 + 19i + 15( −1) 6 + 19i − 15 −9 + 19i −9 19


= = × = = = = = + i
z2 2 − 3i (2 − 3i) (2 + 3i) 4 − 9i2 4+9 13 13 13 13

 z   9 19  9 19
L.H.S. =  1=
 z   − 13 + 13 i  =
− − i
13 13
 2  

z1 3 + 5i 3 − 5i (3 − 5i) (2 − 3i)
R.H.S. = = = = ×
z2 2 − 3i 2 + 3i (2 + 3i) (2 − 3i)

6 − 9i − 10i + 15i2 6 − 19i + 15( −1) 6 − 19i − 15 9 19  z  z1


= = = = − − i ∴  1  =
4+9 13 13 13  z 2  z2
2
4 − 9i
M a them a ti cs | 6.11

Illustration 16: If z be a non-zero complex number, then show that (z −1 ) = (z)−1 . (JEE MAIN)

Sol: By considering z = a + ib and using properties of conjugate we can prove given equation.

Let z = a + ib Since, z ≠ 0 , we have x2 + y 2 > 0

z −1=
1
=
1
=
1
×
a − ib
z a + ib a + ib a − ib
=
2
a
a +b 2

2
ib
a +b 2
z −1
⇒=
2
a
a +b 2
+ ( )ib
a + b2
2
 … (i)

1 1 1 1 a + ib a b
and (z)−1 = = = = × = +i  … (ii)
z a + ib a − ib a − ib a + ib 2
a +b 2
a + b2
2

From (i) and (ii), we get (z −1 ) = (z)−1 .

(a + i) 2 (a2 + 1) 2
Illustration 17: If = p + iq, then show that p2 + q2 = . (JEE MAIN)
2a − i 4a2 + 1

Sol: Multiply given equation to its conjugate.


(a + i) 2
We have, p + iq =  … (i)
2a − i
 (a + i)2 
Taking conjugate of both sides, we get p + iq =
 
 (2a − i) 
 

(a + i)2   z1  z1  (a − i) 2
⇒ p − iq =    =  ⇒ p − iq = … (ii) using (z
= 2
) z= · z (z) 2 
· z z=
 
(2a − i) 
  z 2  z2  (2a + i)

 (a + i) 2   (a − i) 2 
Multiplying (i) and (ii), we get (p + iq) (p – iq) =   
 2a − i   2a + i 
  
(a2 − i2 ) 2 (a2 + 1) 2
⇒ p2 − i2q2 = ⇒ p2 + q2 = .
4a2 − i2 4a2 + 1

Illustration 18: Let z1 , z 2 , z3 , ……. zn are the complex numbers such that | z1 | = | z 2 | = ……. = | zn | = 1 . If z =
 n  n 1 
 ∑ zk   ∑  then prove that
=  k 1=   k 1 zk 
(i) z is a real number (ii) 0 < z ≤ n2  (JEE ADVANCED)

1  n  n 1 
Sol: Here | z1 | = | z 2 | = ……. = | zn | = 1, therefore zz = 1 ⇒ z = . Hence by substituting this to z =  ∑ zk   ∑ ,
z   z
we can solve above problem. =  k 1=  k 1 k 
1 1 1 
Now, z = (z1 + z 2 + z3 + ……. + zn )  + + ……. + 
 z1 z 2 zn 

= (z1 + z 2 + z3 + ……. + zn ) ( z1 + z2 + ……. + zn ) = (


(z1 + z 2 + z3 + ……. + zn ) z1 + z 2 + ……. + zn )
= | z1 + z 2 + z3 + ……. + zn | 2 which is real

≤ ( | z1 | + | z 2 | + | z3 | +…….+ | zn | ) = n2 ∴
2
0 < z ≤ n2 .
6 . 1 2 | Complex Number

Illustration 19: Let x1, x2 are the roots of the quadratic equation x2 + ax + b = 0 where a, b are complex numbers and
y1, y2 are the roots of the quadratic equation y 2 + | a | y + | b | =
0 . If | =
x1 | | =
x2 | 1 , then prove that | = y2 | 1 .
y1 | | =
 (JEE ADVANCED)
Sol: Solve by using modulus properties of complex number.

Let x2 + ax + b =0 where x1 and x2 are complex numbers


x1 + x2 = – a … (i)
and x1 x2 = b … (ii)
From (ii) | x1 | | x2 | = | b | ⇒ | b | =
1 Also | – a | = | x1 + x2 |

∴ | a | ≤ | x1 | + | x2 | or |a | ≤ 2

y1
Now consider y 2 + | a | y + | b | =
0 , where y1 and y 2 are complex numbers
y2

− | a | ±  4 − | a |2  i
− | a | ± | a |2 − 4 | b |   | a | 2 + 4− | a | 2
y1, 2 = = ∴ | y1, 2 | = =1
2 2 2

Hence, | = y2 | 1 .
y1 | | =

6. TRIANGLE ON COMPLEX PLANE


In a ΔABC, the vertices A, B and C are represented by the complex numbers z1, z2 and z3 respectively, then

z1 + z 2 + z3
(a) Centroid: The centroid ‘G’ is given by . Refer to Fig 6.10.
3
A(z1)

B(z2) D C(z3)

Figure 6.10: Centroid

a z1 + b z 2 + c z3
(b) Incentre: The incentre ‘I’ is given by . Refer to Fig 6.11.
a+b+c
A(z1)

B(z2) C(z3)

Figure 6.11: Incentre


M a them a ti cs | 6.13

z1 tanA + z 2 tanB + z3 tanC


(c) Orthocentre: The orhtocentre ‘H’ is given by .
∑ tanA
A(z1)

n
E
H

B(z2) p D q C(z3)

Figure 6.12: Orthocentre

p z3 + q z 2
Proof: From section formula, we have zD =
a
In ΔABD and ΔACD, p = c cos B and q = b cos C. Refer to Fig 6.12.
bcosC z 2 + c cosB z3
Therefore, zD =
a
Now, AE = c cos A;= = AEcosec C = c cos A cosec C
n AH
⇒n=2R cos A [Using Sine Rule]
and m = c cos B cot C or, m = 2R cos B cos C [Using Sine Rule]

mz1 + n ZD
Hence, zH = .
m+n
 bcosC z 2 + c cosB z3 
2R cosBcosC z1 + 2R cos A  
 a 
=
2R (cos A + cosBcosC)

acosB cosC z1 + b cos A cosC z 2 + c cos A cosB z3


=
a( − cos(B + C) + cosB cosC)

z1 (sinA cosB cosC) + z 2 (sinB cosC cos A) + z3 (sinC cos A cosB)


=
sinA (sinB sinC)

z1 tanA + z 2 tanB + z3 tanC z1 tanA + z 2 tanB + z3 tanC


∴ ZH = or
∑ tanA ∏ tanA
[If A + B + C = π , then tan A + tan B + tan C = tan A tan B tan C]

(d) Circumcentre:
Let R be the circumradius and the complex number z0 represent the circumcentre of the triangle as shown in Fig 6.11.

∴ | z1 − z 0 | = | z 2 − z 0 | = | z3 − z 0 |

Consider, | z1 − z 0 | 2 =| z 2 − z 0 | 2

(z1 − z 0 ) (z1 − z0 ) = (z 2 − z 0 ) (z2 − z0 )


6 . 1 4 | Complex Number

z1 (z1 − z 0 ) − z2 (z 2 − z=
0) z 0 (z1 − z 0 ) − (z 2 − z 0 )

) z 0 (z1 − z 2 ) 
z1 (z1 − z 0 ) − z2 (z 2 − z 0= … (i)

Similarly 1st and 3rd gives


) z 0 (z1 − z3 ) 
z1 (z1 − z 0 ) − z3 (z3 − z 0= … (ii)
On dividing (i) by (ii), z0 gets eliminated and we obtain z 0 .

Alternatively: From Fig 6.13, we have 


A(z1)
1 = ar PBC
BD m Ar. ∆ABD Ar. ∆PBD 2 = ar PCA
= = =
DC n Ar. ∆ADC Ar. ∆PDC 3 = ar PAB
m Ar. ∆ABD − Ar. ∆PBD ∆3 2
∴ = 3
n Ar. ∆ADC − Ar. ∆PDC ∆2 P(z0)
R2 1
sin2C
m 2 sin2C
∴ = = C(z3) D
B(z2)
n R 2 sin2B n m
sin2B
2
Figure 6.13: Circumcentre
sin2B (z 2 ) + sin2C(z3 )
Hence, ZD =
sin2B + sin2C

PA l ∆ABP ∆APC ∆ABP + ∆APC l ∆ + ∆2 sin2C + sin2B


Now, = = = = ∴ =3 =
PD k ∆PBD ∆CPD ∆PBD + ∆CPD k ∆1 sin2A

kz1 + l zD z1 sin2A + z 2 sin2B + z3 sin2C


Hence, z 0 = =
k+l
∑ sin2A

PLANCESS CONCEPTS
1
• The area of the triangle whose vertices are z, iz and z + iz is | z |2 .
2

3
• The area of the triangle with vertices z, ω z and z + ω z is | z |2 .
4
• If z1 , z 2 , z3 be the vertices of an equilateral triangle and z 0 be the circumcentre, then z12 + z 22 + z32 =
3z 02 .
• If z1 , z 2 , z3 ,…….zn be the vertices of a regular polygon of n sides and z 0 be its centroid, then
z12 + z 22 + ……. + zn2 = nz 02 .

• If z1, z2 , z3 be the vertices of a triangle, then the triangle is equilateral if (z1 ‒ z2)2 + (z2 ‒z3)2 + (z3 ‒ z1)2= 0
1 1 1
or z12 + z 22 + z32= z1 z 2 + z 2 z3 + z3 z1 or + + = 0.
z1 − z 2 z 2 − z3 z3 − z1
• If z1, z2 , z3 are the vertices of an isosceles triangle, right angled at z2 then z12 + 2z 22 + =
z32 2z 2 (z1 + z 3 ) .
2
• If z1, z2 , z3 are the vertices of a right-angled isosceles triangle, then (z1 − z 2 ) = 2 (z1 − z3 ) (z3 − z 2 ) .
• If z1, z2 , z3 be the affixes of the vertices A, B, C respectively of a triangle ABC, then its orthocentere
a (sec A) z1 + b(secB) z 2 + c(sec C) z3
  is .
asec A + bsecB + c sec C
Shivam Agarwal (JEE 2009, AIR 27)
M a them a ti cs | 6.15

z3
Illustration 20: If z1, z2 , z3 are the vertices of an isosceles triangle right angled
at z 2 then prove that z12 + 2z 22 + =
z32 2z 2 (z1 + z 3 ) (JEE MAIN)

Sol: Here (z1 − z 2 ) = (z3 − z 2 )e 2 . Hence by squaring both side we will get the result.
z1 z2
⇒ (z1 − z 2 )2 = i2 (z3 − z 2 )2
Figure 6.14
⇒ z32 + z 22 − 2z3 z 2 =−z12 − z 22 + 2z1 z 2 ⇒ z12 + 2z 22 + =
z32 2z 2 (z1 + z 3 ) .

Illustration 21: A, B, C are the points representing the complex numbers z1, z2 , z3 respectively and the circumcentre of
the triangle ABC lies at the origin. If the altitudes of the triangle through the opposite vertices meets the circumcircle at
D, E, F respectively. Find the complex numbers corresponding to the points D, E, F in terms of z1, z2 , z3. (JEE MAIN)
  A(z1)
Sol: Here the ∠BOD = π − 2B , hence OD = OB ei( π−2B) .
(/2)-B E()
 
i( π−2B)
From Fig 6.13, we have OD = OB e ; 2B
O
i ( π−2B) −i2B
α = z2 e = −z2 e  … (i) B(z2) C(z3)
-2B
also, z1 = z3 ei 2B  … (ii) D()
Figure 6.15
− z 2 z3
∴ αz1 =−z 2 z3 ⇒α=
z1
−z3 z1 −z z
Similarly, β = and γ = 1 2 .
z2 z3

Illustration 22: If zr (r = 1, 2, …,6) are the vertices of a regular hexagon then E(z5) D(z4)
6
prove that ∑ zr2 = 6z02 , where z 0 is the circumcentre of the regular hexagon.
r =1
 (JEE MAIN)
F(z6) z0 C(z3)

Sol: As we know If z1 , z 2 , z3 ,…….zn be the vertices of a regular polygon of n sides


and z 0 be its centroid, then z12 + z 22 + ……. + zn2 = nz 02 .
A(z1) B(z2)
Here by the Fig 6.14,
Figure 6.16
3z 20 = z12 + z32 + z52
6
and, 3z 20 = z 22 + z 24 + z 62 ⇒ 6z 20 = ∑ zr2 .
r =1

Illustration 23: If z1, z2 , z3 are the vertices of an equilateral triangle then prove that z12 + z 22 + z32= z1 z 2 + z 2 z3 + z3 z1
and if z0 is its circumcentre then 3z 20 = z12 + z 22 + z32 . (JEE ADVANCED)

Sol: By using triangle on complex plane we can prove


z1 + z 2 + z3
z12 + z 22 + z32= z1 z 2 + z 2 z3 + z3 z1 and by using z 0 = we can prove 3z 20 = z12 + z 22 + z32 .
3
Y
To Prove, z12 + z 22 + z32= z1 z 2 + z 2 z3 + z3 z1
A(z1)
As seen in the Fig 6.17,

C(z3)
z1 − z 2 (z3 − z 2 ) e 3 2
/3
 = ⇒ (z1 − z 2 ) (z1 − z3 ) =
−(z 2 − z3 )
z 2 − z3 iπ B(z2)
(z1 − z3 ) e3 O X

0 ∴ ∑ z12 = Figure 6.17


⇒ z12 − z1 z3 − z 2 z1 + z 2 z3 + z 22 + z32 − 2z 2 z3 = ∑ z1 z2
6 . 1 6 | Complex Number

Now if z0 is the circumcentre of the ∆, then we need to prove 3z 20 = z12 + z 22 + z32 .


z1 + z 2 + z3
Since in an equilateral triangle, the circumcentre coincides with the centroid, we have z 0 =
3
(3z0 )
2
⇒ (z1 + z 2 + z3 )2 =

⇒ ∑ z12 + 2∑ z1 z 2 =
9z 02 ∴ 3∑ z12 =
9z 02 .

Illustration 24: Prove that the triangle whose vertices are the points z1, z2 , z3 on the Argand plane is an equilateral
1 1 1
triangle if and only if + + = 0.  (JEE ADVANCED)
z 2 − z3 z3 − z1 z1 − z 2
Sol: Consider ABC is the equilateral triangle with vertices z1 , z 2 and z3 respectively.
A(z1)
Therefore | z 2 − z3 | = | z3 − z1 | = | z1 − z 2 | .
Let ABC be a triangle such that the vertices A, B and C are z1, z2 and z3 respectively. 60

Further, let α= z 2 − z3 , β= z3 − z1 and γ= z1 − z 2 . Then α + β + γ = 0  … (i)


As shown in Fig 6.16, let ∆ABC be an equilateral triangle. Then, BC = CA = AB
⇒ | z 2 − z3 | = | z3 − z1 | = | z1 − z 2 | ⇒ | α | = | β | = | γ | 60 60
B (z2) C (z3)
⇒ | α |2 =| β |2 =| γ |2 =λ(say)
Figure 6.18
⇒ αα = ββ = γγ = λ
λ λ λ
⇒α
= ,β
= ,=
γ  … (ii)
α β γ
Now, α + β + γ = 0 [from (i)]
λ λ λ
⇒ α + β + γ =0 ⇒ + + =0 [Using (ii)]
α β γ
1 1 1 1 1 1
⇒ + + =0⇒ + + 0 which is the required condition.
=
α β γ z 2 − z3 z3 − z1 z1 − z 2

Conversely, let ABC be a triangle such that


1 1 1 1 1 1
⇒ + + 0 i.e. ⇒ + + =
= 0
z 2 − z3 z3 − z1 z1 − z 2 α β γ
1 1 1
Thus, we have to prove that the triangle is equilateral. We have, + + =0
α β γ
1 1 1 1 β+ γ  1 α
⇒ =− +  ⇒ =−  ⇒ = ⇒ α2 = βγ ⇒ | α |2 = | βγ |
α β γ  α  βγ  α βγ

⇒ | α |2 = | β | | γ | ⇒ | α |3 = | α | | β | | γ |

Similarly, ⇒ | β |3 = | α | | β | | γ | and | γ |3 =| α | | β | | γ |
∴ |α|=|β|=| γ |
⇒ | z 2 − z3 | = | z3 − z1 | = | z1 − z 2 | ⇒ BC = CA = AB
Hence, the given triangle is an equilateral triangle.
M a them a ti cs | 6.17

1 1 1
Illustration 25: Prove that the roots of the equation + + 0 (where z , z , z are pair wise distinct
=
z − z1 z − z 2 z − z3 1 2 3

complex numbers) correspond to points on a complex plane, which lie inside a triangle with vertices z1, z2 , z3 excluding
its boundaries.  (JEE ADVANCED)

z − z1 z − z2 z − z3
Sol: By using modulus and conjugate properties we can reduce given expression as + +
| z − z1 | 2 | z − z2 | 2 | z − z3 | 2
1
= 0. Therefore by putting | z − zi | = , where i = 1, 2 and 3, we will get the result.
2
ti A(z1)
1
t1 (z − z1 ) + t2 (z − z2 ) + t3 (z − z3 ) = 0 where | z − z1 | 2 =etc and t1 , t2 , t3 ∈ R +
t1 t2

t1 (z − z1 ) + t2 (z − z 2 ) + t3 (z − z3 ) = 0 z’ t3
t z +t z +t z t1 z t1 + t2
(t1 + t2 + t3 ) z= t1 z1 + t2 z 2 + t3 z3 ⇒ z =1 1 2 2 3 3
t1 + t2 + t3
B (z2) C (z3)
t1 z1 + t2 z 2 t1 + t2 t3 z 3 t1 + t2 t3 z 3
⇒z · + = z' + Figure 6.19
t1 + t2 t1 + t2 + t3 t1 + t2 + t3 t1 + t2 + t3 t1 + t2 + t3
(t + t )z'+ t3 z3
⇒ z =1 2 ⇒ z lies inside the ∆z1 z 2 z3
t1 + t2 + t3

If t=
1 t=
2 t3 ⇒ z is the centroid of the triangle.
Also, it implies | z − z1 | =| z − z 2 | =| z − z3 | ⇒ z is the circumcentre .

Illustration 26: Let z1 and z 2 be roots of the equation z 2 + pz + q = 0, where the coefficients p and q may be
complex numbers. Let A and B represent z1 and z 2 in the complex plane. If ∠AOB = α ≠ 0 and OA = OB, where O
α
is the origin, prove that p2 = 4qcos2 .  (JEE ADVANCED)
2
 
Sol: Here OB = OAeiα . Therefore by using formula of sum and product of roots of quadratic equation we can prove
this problem.
Y
Since z1 and z 2 are roots of the equation z 2 + pz + q = 0
−p and z z = q B(z2)
z +z =
1 2 1 2 (1)
   A(z1)
Since OA = OB. So OB is obtained by rotating OA in anticlockwise direction through X’ X
angle α. O
  z z2
∴ ∴ OB = OAeiα ⇒ z2 = z1eiα ⇒ 2 = eiα ⇒ = cos α + isin α
Y’
z1 z1 Figure 6.20

z2 z 2 + z1 α α α α
⇒ + 1 = 1 + cos α + isin α ⇒ = 2cos  cos + isin
=  2cos e 2
z1 z1 2 2 2 2
iα 2
z 2 + z1 α  z + z1  α
⇒ 2cos e 2 ⇒  2
= 4 cos2 eiα
 =
z1 2  z1  2
2
 z + z1  α z2 α
⇒ ( z 2 + z1 ) =
2
⇒  2  = 4 cos2 4z1 z 2 cos2
 z1  2 z1 2
α α
⇒ ( − p) 2 = 4qcos2 ⇒ p 2 = 4qcos2 .
2 2
6 . 1 8 | Complex Number

Illustration 27: On the Argand plane z1, z2 and z3 are respectively the vertices of an isosceles triangle ABC with
AC = BC and equal angles are θ. If z4 is the incentre of the triangle then prove that (z2 ‒ z1) (z3 ‒ z1) = (1 + sec θ)
(z4 ‒ z1)2 (JEE ADVANCED)

Sol: Here by using angle rotation formula we can solve this problem. From Fig 6.21, we have
z 2 − z1 z 4 − z1
= eiθ /2 … (i) (clockwise) C(z3)
| z 2 − z1 | | z 4 − z1 |

z3 − z1 z 4 − z1
and = eiθ /2 … (ii) (anticlockwise)
| z3 − z1 | | z 4 − z1 |

I(z4)
Multiplying (i) and (ii) /2
/2 
(z 2 − z1 ) (z3 − z1 ) | (z 2 − z1 ) | | (z3 − z1 ) | AB| AC | 2(AD) (AC) 2(AD)2 AC A(z1) D B(z2)
= = = = ·
(z 4 − z1 ) 2 | z 4 − z1 | 2 (AI) 2
(AI) 2
(AI) 2 AD
Figure 6.21
θ
= 2cos2 sec θ = (1+ cos θ)sec θ .
2

7. REPRESENTATION OF DIFFERENT LOCI ON COMPLEX PLANE


(a) | z – (1 + 2i) | = 3 denotes a circle with centre (1, 2) and radius 3 (see Fig 6.22).
Y

(1,2)
Radius = 3
X’ X

Y’
Figure 6.22: Circle on a complex plane

(b) | z – 1 | = | z – i | denotes the equation of the perpendicular bisector of join of (1, 0) and (0, 1) on the Argand
plane (see Fig 6.24).
Y

B(0,1)

O Locus of z
X’ X
A(0,1)

Y’

Figure 6.23: Perpendicular bisector complex plane

(c) | z – 4i | + | z + 4i | = 10 denotes an ellipse with foci at (0, 4) and (0, – 4); major axis 10; minor axis 6 with
4
e= (see Fig 6.24). Y
5
(0,4)

X’ X

(0,-4)

Y’
Figure 6.24: Ellipse on a complex plane
M a them a ti cs | 6.19

 2 2 
2 36 64 4 x + y =1
e = 1− = ⇒ e= 9 25
100 100 5  

(d) | z – 1 | + | z + 1 | = 1 denotes no locus. (Triangle inequality).


(e) | z – 1 | < 1 denotes area inside a circle with centre (1, 0) and radius 1.
(f) 2 ≤ | z − 1 | < 5 denotes the region between the concentric circles of radii 5 and 2. Centred at (1, 0) including
the inner boundary (see Fig 6.25).
Y

(1,0)
X’ X

Y’

Figure 6.25: Circle disc on a complex plane

π
(g) 0 ≤ arg z ≤ (z ≠ 0) where z is defined by positive real axis and the part of the line x = y in the first quadrant.
4
It includes the boundary but not the origin. Refer to Fig 6.26.

y=x
X’ X
O

Y’

Figure 6.26

(h) Re (z 2 ) > 0 denotes the area between the lines x = y and x = – y which includes the x-axis.

0 ⇒ x2 − y 2 > 0
Hint: (x2 − y 2 ) + 2xyi = ⇒ (x – y) (x + y) > 0.

Illustration 28: Solve for z, if z 2 + | z |=


0 .  (JEE MAIN)

Sol: Consider z = x + iy and solve this using algebra of complex number.

Let z = x + iy ⇒ (x + iy) 2 + x2 + y 2 =
0 ⇒  x2 − y 2 + x2 + y 2  + (2ixy) =
0
 
⇒ Either x =0 or y =0 ; x = 0 ⇒ − y 2 + | y | =
0 ⇒ y= 0, 1, − 1 ∴ z= 0, i, − i
2
0
and, y = 0 ⇒ x + | x | = 0
⇒ x= ∴z =0
Therefore, z = 0, z = i, z = – i.

Illustration 29: If the complex number z is to satisfy |z| = 3, z − {a(1 + i) − i} ≤ 3 and | z + 2a – (a + 1) i | > 3
simultaneously for at least one z then find all a ∈ R . (JEE ADVANCED)

Sol: Consider z = x + iy and solve these inequalities to get the result.


6 . 2 0 | Complex Number

All z at a time lie on a circle | z | = 3 but inside and outside the circles | z – {a (1 + i) – i} | = 3 and | z + 2a – (a + 1)
i | = 3, respectively.
Let z = x + iy then equation of circles are x2 + y 2 =
9 … (i)
2 2
(x − a) + (y − a + 1) =9 … (ii)
and (x + 2a)2 + (y − a − 1)2 =9 … (iii)
Circles (i) and (ii) should cut or touch then distance between their centres ≤ sum of their radii.

+ +
⇒ (a − 0) 2 + (a − 1 − 0)2 ≤ 3 + 3 ⇒ a2 + (a − 1)2 ≤ 36
-
1-71 1+71
35
⇒ 2a2 − 2a − 35 ≤ 0 ⇒ a2 − a − ≤0 2 2
2
2 Figure 6.27
 1 71 1 − 71 1 + 71
⇒ a −  ≤ ∴ ≤a≤ … (iv)
 2 4 2 2

Again circles (i) and (iii) should not cut or touch then distance between their centres > sum of the radii

⇒ ( −2a − 0) 2 + (a + 1 − 0) 2 > 3 + 3 ⇒ 5a 2 + 2a + 1 > 6 ⇒ 5a 2 + 2a + 1 > 36

2a
⇒ 5a 2 + 2a − 35 > 0 ⇒ a2 + −7 > 0 +
5 +
-
-1-411 -1+411
 −1 − 4 11   −1 + 4 11 
Then  a −  a − >0 5 5
 5  5 
   Figure 6.28
 −1 − 4 11   −1 + 4 11 
∴ a ∈  −∞ , ∪ , ∞  … (v)
 5   5 
   
The common values of a satisfying (iv) and v are

 1 − 71 −1 − 4 11   −1 + 4 11 1 + 71 
a∈ , ∪ , 
 2 5   5 2 
   

8. DEMOIVRE’S THEOREM
Statement: (cos nθ + isin nθ) is the value or one of the values of (cos θ + isin θ) n , ∀ n ∈ Q . Value if n is an integer.
One of the values if n is rational which is not integer, the theorem is very useful in determining the roots of any
complex quantity.
Note: We use the theory of equations to find the continued product of the roots of a complex number.

PLANCESS CONCEPTS
The theorem is not directly applicable to (sin θ + icos θ) n , rather
n
n  π  π  π  π 
(sin θ + icos θ) = cos  − θ  + isin  − θ   = cos n  − θ  + isin n  − θ 
 2  2  2  2 
M a them a ti cs | 6.21

8.1 Application
Cube root of unity
−1 + i 3 −1 − i 3
(a) The cube roots of unity are 1, ,
2 2
[Note that 1 – i 3 = – 2 and 1 + i 3 = –2 ω2 ]

(b) If ω is one of the imaginary cube roots of unity then 1 + ω + ω2 = 0 .


In general 1 + ωr + ω2r = 0; where r = 1, and not a multiple of 3.
2π 2π 4π 4π
(c) In polar form the cube roots of unity are: cos 0 + isin 0 ; cos + isin ; cos + isin
3 3 3 3
(d) The three cube roots of unity when plotted on the argand plane constitute the vertices of an equilateral
triangle.
[Note that the 3 cube roots of i lies on the vertices of an isosceles triangle]

(e) The following factorization should be remembered.


For a, b, c ∈ R and ω being the cube root of unity,

(i) a3 − b3= (a − b) (a − ωb)(a − ω2b)

(ii) x2 + x + 1= (x − ω) (x − ω2 )

(iii) a3 + b3= (a + b) (a + ωb)(a + ω2b)

(iv) a3 + b3 + c3 − 3abc = (a + b + c) (a + ωb + ω2c) (a + ω2b + ωc)

nth roots of unity: If 1, α1 , α2 , α3 , ……., αn−1 are the n, nth roots of unity then
 2π 
i 
2π 2π
(i) They are in G.P. with common ratio e n 
= cos + isin
n n
(ii) 1p + α1p + αp2 + ……. + αnp−1 = 0 if p is not an integral multiple of n

1p + (α1 )p + (α2 ) p + ……. + (αn−1 )p =n if p is an integral multiple of n.



(iii) (1 − α1 ) (1 − α2 ) ……. (1 − αn−1 ) =n .

Steps to determine nth roots of a complex number


(i) Represent the complex number whose roots are to be determined in polar form.

(ii) Add 2mπ to the argument.

(iii) Apply De Moivre’s TheoremT

(iv) Put m = 0, 1, 2, 3, ……. (n – 1) to get all the nth roots.

1 1 1
 2mπ 2mπ 
Explanation: Let z = 1n = (cos 0 + isin 0)n = ( cos 2mπ + isin 2mπ ) n =  cos + isin 
 n n 
Put m = 0, 1, 2, 3, ……. (n – 1), we get

2π 2π 4π 4π 2(n − 1)π 2(n − 1)π


1, cos + isin , cos + isin , ……. , cos + isin (n, nth roots in G.P.)
n
 n n n n n
α
6 . 2 2 | Complex Number

1 − ( αp ) n 1 − ( αn ) p
Now, S = 1p + αp + α2p + α3p + ……. + α(n−1)p = =
1 − αp 1 − αp
 0
n p  = 0, if p is not an integral multiple of n
1 − (α ) non zero
= = 
1 − αp 0
 = indeterminant, if p is an integral multiple of n
0

Again, if x is one of the nth root of unity then xn – 1 = (x – 1) (x − α1 )(x − α2 ) ……. (x − αn−1 )
xn − 1
1 + x + x2 + …… + xn−1 = ≡ (x − α1 )(x − α2 ) ……. (x − αn−1 )
x −1
Put x = 1, to get (1 − α1 ) (1 − α2 ) ……. (1 − α n−1 ) =n

Similarly put x = – 1, is to get other result.

PLANCESS CONCEPTS

 | z | +a | z | −a 
Square roots of z = a + ib are ±  +i  for b > 0.
 2 2 

If 1, α1 , α2 , α3 , ……., αn−1 are the n, nth roots of unity then

(1 + α1 ) (1 + α2 ) ……. (1 + α n−1 ) = 0 if n is even and 1 if n is odd.

1 · α1 · α2 · α3 · ……. · αn−1 = 1 or – 1 according as n is odd or even.


0, if n = 3k

(ω − α1 ) (ω − α2 ) ……. (ω − αn−1 ) = 1, n 3k + 1 .
if =
1 + ω, if n= 3k + 2

Ravi Vooda (JEE 2009, AIR 71)

Illustration 30: If x= a + b , y = aω + bω2 and z = aω2 + bω , then prove that x3 + y 3 + z3= 3 (a3 + b3 )  (JEE MAIN)

Sol: Here x + y + z =.
0 Take cube on both side.
x + y + z =
0 ⇒ x3 + y 3 + z 3 =
3xyz ∴ LHS = 3xyz
= 3(a + b)(aω + bω2 )(aω2 + bω) = 3(a + b)(aω + bω2 )(aω2 + bω.ω3 ) = 3ω3 (a + b)(a + bω)(a + b=
ω2 ) 3 (a3 + b3 )

Illustration 31: The value of expression 1(2 − ω)(2 − ω2 ) + 2 (3 − ω) (3 − ω2 ) + ... + (n − 1)(n − ω)(n − ω2 ) .
 (JEE ADVANCED)

Sol: The given expression represent as x3 − 1 = (x – 1) (x − ω) (x − ω2 ) . Therefore by putting x = 2, 3, 4 … n, we will


get the result.
x3 − 1 = (x – 1) (x − ω) (x − ω2 )
Put x = 2 23 – 1 = 1 · (2 − ω) (2 − ω) 2 Put x = 3 33 – 1 = 2 · (3 − ω) (3 − ω2 ) :
Put x = n n3 – 1 = (n − 1) (n − ω) (n − ω2 )
2
 n (n + 1) 
= (23 + 33 + ……. + n3 ) − (n − 1) = (13 + 23 + 33 + ……=
∴ LHS . + n3 ) − n   −n
 2 
M a them a ti cs | 6.23

9. SUMMATION OF SERIES USING COMPLEX NUMBER

 nθ 
sin  
(a) cos θ + cos2θ + cos3θ + …… + cosnθ =  2  cos  n + 1  θ
 
θ
sin    2 
2

 nθ 
sin  
(b) sin θ + sin2θ + sin3θ + …… + sinnθ =  2  sin  n + 1  θ
 
θ
sin    2 
2

Note: If θ = , then the sum of the above series vanishes.
n

9.1 Complex Number and Binomial Coefficients


Try the following questions using the binomial expansion of (1 + x)n and substituting the value of x according to
the binomial coefficients in the respective question.
Find the value of the following
(i) C0 + C 4 + C8 + ……. (ii) C1 + C5 + C9 + …….
(iii) C2 + C6 + C10 + ……. (iv) C3 + C7 + C11 + …….
(v) C0 + C3 + C6 + C9 + …….
Hint (v): In the expansion of (1 + x)n , put x =1, ω,and ω2 and add the three equations.

Illustration 32: If 1, ω, ω2 , ……., ωn−1 are nth roots of unity, then the value of (5 − ω) (5 − ω2 ) ……. (5 − ωn−1 ) is equal
to (JEE MAIN)
1
Sol: Here consider x = (1)n , therefore xn − 1 =0 (has n roots i.e. 1, ω, ω2 , ……., ωn−1 ).
xn − 1
⇒ xn − 1= (x − 1)(x − ω) (x − ω2 ) …….(x − ωn−1 ) ⇒ = (x − ω) (x − ω2 ) …….(x − ωn−1 )
x −1
5n − 1
⇒ Putting x = 5 in both sides, we get ∴ (5 − ω) (5 − ω2 ) ……. (5 − ωn−1 ) = .
4

10. APPLICATION IN GEOMETRY

10.1 Distance Formula


Distance between A (z1 ) and B(z 2 ) is given by AB = | z 2 − z1 | . Refer Fig 6.29.

A(z1)

B(z2)
X’ X
O

Y’
Figure 6.29
6 . 2 4 | Complex Number

10.2 Section Formula n B(z2)

The point P(z) which divides the join of A(z1) and B(z2) in the ratio m: n is m P(z)
mz 2 + nz1
given by z = . Refer Fig 6.30. A(z1)
m+n
Figure 6.30

10.3 Midpoint Formula


1
Mid-point M(z) of the segment AB is given by z = (z + z 2 ) .
2 1

1 1
A(z1) M(z) B(z2)

Figure 6.31 Mid point formula

10.4 Condition For Four Non-Collinear Points


Condition(s) for four non-collinear A(z1 ), B(z 2 ), C(z3 ) and D(z 4 ) to represent vertices of a

(a) Parallelogram: The diagonals AC and BD must bisect each other


A(z1) D(z4)
1 1
⇔ (z1 + z 3 ) = (z + z 4 )
2 2 2
⇔ z1 + z 3 = z 2 + z 4

(b) Rhombus:
B(z2) C(z3)
(i) The diagonals AC and BD bisect each other Figure 6.32
⇔ z1 + z 3 = z 2 + z 4 , and A(z1)

D(z4)
(ii) A pair of two adjacent sides are equal, for instance AD = AB
⇔ | z 4 − z 1 | = | z2 − z 1 |

(c) Square:
B(z2) C(z3)
(i) The diagonals AC and BD bisect each other
Figure 6.33
⇔ z1 + z 3 = z 2 + z 4

(ii) A pair of adjacent sides are equal; for instance, AD = AB A(z1) D(z4)

⇔ | z 4 − z 1 | = | z2 − z 1 |

(iii) The two diagonals are equal, that is AC = BD
⇔ | z3 − z 1 | = | z 4 − z 2 |
B(z2) C(z3)
Figure 6.34
(d) Rectangle:
(i) The diagonals AC and BD bisect each other A(z1) D(z4)

⇔ z1 + z 3 = z 2 + z 4

(ii) The diagonals AC and BD are equal


⇔ | z3 − z 1 | = | z 4 − z 2 | B(z2) C(z3)

Figure 6.35
M a them a ti cs | 6.25

10.5 Triangle
In a triangle ABC, let the vertices A, B and C be represented by the complex numbers z1, z2, and z3 respectively. Then

(a) Centroid: The centroid (G), is the point of intersection of medians of ∆ABC . It is given by the formula
1
z= (z + z 2 + z3 )
3 1
A(z1)

B(z2) C(z3)

Figure 6.36 (a)

(b) Incentre: The incentre (I) of ∆ABC is the point of intersection of internal angular bisectors of angles of
∆ABC . It is given by the formula
az1 + bz 2 + cz3
z= ,
a+b+c A(z1)

B(z2) C(z3)

Figure 6.36 (b)

(c) Circumcentre: The circumcentre (S) of ∆ABC is the point of intersection of perpendicular bisectors of sides
of ∆ABC . It is given by the formula
| z1 | 2 z1 1 A(z1)
| z2 | 2 z2 1
2
2 2 2
| z1 | (z 2 − z3 )+ | z 2 | (z3 − z1 )+ | z3 | (z1 − z 2 ) | z3 | z3 1
z= =
z1 (z 2 − z3 ) + z2 (z3 − z1 ) + z3 (z1 − z 2 ) z1 z1 1 S
z2 z2 1
B(z2)
z3 z3 1 C(z3)

z1 (sin2A) + z 2 (sin2B) + z3 (sin2C) Figure 6.36 (c)


Also, z =
sin2A + sin2B + sin2C

H 2 G 1 S
(Orthocentre) (Centroid) (Circumcentre)

Figure 6.37

(d) Euler’s Line: The orthocenter H, the centroid G and the circumcentre S of a triangle which is not equilateral
lies on a straight line. In case of an equilateral triangle these points coincide.
G divides the join of H and S in the ratio 2 : 1 (see Fig 6.37).
1
Thus,=
zG (z + 2zS )
3 H
6 . 2 6 | Complex Number

10.6 Area of a Triangle A(z1)

Area of ∆ABC with vertices A(z1 ), B(z 2 ) and C(z3 ) is given by


z1 z1 1
1 1
∆ =| z 2 z2 1 | = Im(z1 z 2 + z2 z3 + z3 z1 )
4i 2
z3 z3 1
B(z2) C(z3)

Figure 6.38
10.7 Conditions for Triangle to be Equilateral
The triangle ABC with vertices A(z1 ), B(z 2 ) and C(z3 ) is equilateral
1 1 1
iff + + =0
z 2 − z3 z3 − z1 z1 − z 2
A(z1)
⇔ z12 + z 22 + z32= z 2 z3 + z3 z1 + z1 z 2 ⇔ z1 z2 = z 2 z3 = z3=
z1 ⇔ z12 z 2 z3 =
and z 22 z1 z3
60
1 z2 z3
z − z1 z3 − z 2
⇔ 1 z3 0 ⇔ 2
z1 = =
z3 − z 2 z1 − z 2
1 z1 z2
60 60
1 1 1 1
⇔ + + = 0 where z= (z1 + z 2 + z3 ) . B(z2) C(z3)
z − z1 z − z 2 z − z3 3 Figure 6.39

10.8 Equation of a Straight line

(a) Non-parametric form: An equation of a straight line joining the two points A(z1 ) and B(z 2 ) is

z z 1 C(z3)
 z − z1 
Arg   = 0 z1 z1 1 = 0
 z 2 − z1  z2 z2 1
B(z2)
z − z1 z − z1
or =
z 2 − z1 z2 − z1 A(z1)

or z (z1 − z2 ) − z(z1 − z 2 ) + z1 z2 − z 2 z1 =
0 Figure 6.40

(b) Parametric form: An equation of the line segment between the points A(z1 ) and B(z 2 ) is
z = tz1 + (1 − t)z 2 , t ( 0,1 ) where t is a real parameter.

(c) General equation of a straight line: The general equation of a straight line is az + az + b =0 where, a is
non-zero complex number and b is a real number.

10.9 Complex Slope of a Line


z1 − z 2
If A(z1 ) and B(z 2 ) are two points in the complex plane, then complex slope of AB is defined to be µ =
z1 − z2
Two lines with complex slopes µ1 and µ2 are
(i) Parallel, if µ1 =µ2 (ii) Perpendicular, if µ1 + µ2 = 0
 −a 
The complex slope of the line az + az + b =0 is given by   .
 a 
M a them a ti cs | 6.27

10.10 Length of Perpendicular from a Point to a Line A


p
Length of perpendicular of point A(ω) from the line az + az + b =0.
| aω + aω + b |
Where a ∈ C − {0}, and b ∈ R is given by p =
2|a|
Figure 6.41

10.11 Equation of Circle z


r or
(a) An equation of the circle with centre z 0 and radius r is | z − z 0 | = r
z= z 0 + reiθ ,0 ≤ θ < 2π (parametric form) or zz − z 0 z − z0 z + z 0 z0 − r 2 = 0 z0

(b) General equation of a circle is zz + az + az + b =0 … (i) Figure 6.42

Where a is a complex number and b is a real number such that aa − b ≥ 0 . Centre of (i) is – a and its radius is
aa − b

(c) Diameter form of a circle: An equation of the circle one of whose diameter is the segment joining A(z1) and
B(z2) is (z − z1 ) (z − z2 ) + (z − z1 )(z − z 2 ) =
0
P(z)
(d) An equation of the circle passing through two points A(z1) and B(z2)
B(z2)

Centre
z z 1
A(z1)
is (z − z1 ) (z − z2 ) + (z − z1 )(z − z 2 ) + i k z1 0 where k is a real parameter.
z1 1 =
z2 z2 1 Figure 6.43

(e) Equation of a circle passing through three non-collinear points.


Let three non-collinear points be A(z1 ), B(z 2 ) and C(z3 ) and P(z) be any point on the circle through A, B and C.
Then either ∠APB
∠ACB = [when angles are in the same segment]
or, ∠ACB + ∠APB = π [when angles are in the opposite segment] (see Fig 6.44).

 z − z2   z − z2   z3 − z 2   z − z1 
⇒ arg  3 − arg   = 0 or, arg   + arg   =
π
 z − z 
 3 1   z − z1   z3 − z1   z − z2 

 z − z 2   z − z1  
⇒ arg  3   0
  = P(z)

 z3 − z1   z − z 2   C(z3) 

 z − z 2   z − z1  
or, arg  3     = π
 z3 − z1   z − z 2  
B(z2)
A(z1) -
P(z)
(z − z1 ) (z3 − z 2 )
In any case, we get is purely real. Figure 6.44
(z − z 2 )(z3 − z1 )

(z − z1 ) (z3 − z 2 )(z − z1 ) (z3 − z2 )


⇔ =
(z − z 2 )(z3 − z1 ) (z − z2 )(z3 − z1 )

(f) Condition for four points to be concyclic.


(z 4 − z1 ) (z3 − z 2 )
Four points z1 , z 2 , z3 andz 4 will lie on the same circle if and only if is purely real.
(z 4 − z 2 )(z3 − z1 )
(z 4 − z1 ) (z3 − z 2 ) (z4 − z1 ) (z3 − z2 )
⇔ =
(z 4 − z 2 )(z3 − z1 ) (z4 − z2 )(z3 − z1 )
6 . 2 8 | Complex Number

PLANCESS CONCEPTS

z1 z1 1
Three points z 1 , z 2 and z3 are collinear if z 2 z2 1 = 0.
z3 z3 1

If three points A(z1 ), B(z 2 ) and C(z3 ) are collinear then slope of AB = slope of BC = slope of AC
z1 − z 2 z 2 − z3 z1 − z3
⇒ = =
z1 − z2 z2 − z3 z1 − z3

Akshat Kharaya (JEE 2009, AIR 235)

2z + 1
Illustration 33: If the imaginary part of is – 4, then the locus of the point representing z in the complex
plane is iz + 1

(a) A straight line (b) A parabola (c) A circle (d) An ellipse (JEE MAIN)

Sol: Put z = x + iy and then equate its imaginary part to – 4.

2z + 1 2 (x + iy) + 1 (2x + 1) + 2iy [(2x + 1) + 2iy] [(1 − y) − ix]


Let z = x + iy, then = = =
iz + 1 i (x + iy) + 1 (1 − y) + ix (1 − y)2 + x2

 2z + 1  2y (1 − y) − x(2x + 1)
As Im   = – 4, we get =–4
 iz + 1  x2 + (1 − y)2
⇒ 2x2 + 2y 2 + x − 2y= 4x2 + 4(y 2 − 2y + 1) ⇒ 2x2 + 2y 2 − x − 6y + 4 =0 . It represents a circle.

Illustration 34: The roots of z5= (z − 1) 5 are represented in the argand plane by the points that are
(a) Collinear (b) Concyclic
(c) Vertices of a parallelogram (d) None of these (JEE MAIN)

Sol: Apply modulus on both the side of given expression.


Let z be a complex number satisfying z5= (z − 1) 5 .
⇒ | z5 | =| (z − 1)5 | ⇒ | z | 5 =−
| z 1 |5 ⇒ | z | =| z − 1 |
1
Thus, z lies on the perpendicular bisector of the segment joining the origin and (1 + i0) i.e. z lies on Re(z) = .
2
z z
Illustration 35: Let z1 and z 2 be two non-zero complex numbers such that 1 + 2 = 1, then the origin and points
represented by z1 and z 2 z 2 z1

(a) Lie on straight line (b) Form a right triangle


(c) Form an equilateral triangle (d) None of these (JEE ADVANCED)

z1
Sol: Here consider z = and z1 and z 2 are represented by A and B respectively and O be the origin.
z2
z1 1
Let z = , then z + =1 ⇒ z2 − z + 1 =0
z2 z

1 ± 3i z 1 ± 3i
⇒ z= ⇒ 1 =
2 z2 2
M a them a ti cs | 6.29

If z1 and z2 are represented by A and B respectively and O be the origin, then

ΟA |z | 1 ± 3i 1 3
= 1 = = + = 1 ⇒ OA =
OB
OB | z2 | 2 4 4

AB | z − z1 | z 1 3  1 3 1 3
Also, = 2 = 1− 1 = 1− ± i =  i = + =1
| z2 | z2   2 2 4 4
OB 2 2 

OB
⇒ AB = Thus, OA = OB = AB ∴ ∆OAB is an equilateral triangle.

Illustration 36: If z1 , z 2 , z3 are the vertices of an isosceles triangle, right angled at the vertex z 2 , then the value
of (z1 − z 2 ) 2 + (z 2 − z3 ) 2 is
(a) -1 (b) 0 (c) (z1 − z3 ) 2 (d) None of these (JEE ADVANCED)

Sol: Here use distance and argument formula of complex number to solve this problem.
As ABC is an isosceles right angled triangle with right angle at B, C(z3)

z −z  π
BA = BC and ∠ABC = 90° ⇒ | z1 − z 2 | = | z3 − z 2 | and arg  3 2  =
 z1 − z 2  2
z3 − z 2 | z3 − z 2 |  π π 
⇒= cos   + isin    = i
z1 − z 2 | z1 − z 2 |  2
   2  B(z2) A(z1)

Figure 6.45
⇒ (z3 − z 2 )2 =
−(z1 − z 2 )2 ⇒ (z1 − z 2 )2 + (z 2 − z3 )2 =
0.

11. CONCEPTS OF ROTATION OF COMPLEX NUMBER


Let z be a non-zero complex number. We can write z in the polar form as follows:
z = r (cos θ + isin θ) = reiθ where r = | z | and arg (z) = θ (see Fig 6.46). Y
Q(zei)
Consider a complex number zeiα .
P(z)
zeiα = (r eiθ )eiα = rei( θ+α )

 X
O

Thus, ze represents the complex number whose modulus is r and argument is θ + α.
Figure 6.46
Geometrically, zeiα can be obtained by rotating the line segment joining
O and P(z) through an angle α in the anticlockwise direction.

Corollary: If A(z1 ) and B(z 2 ) are two complex number such that
| z2 |
θ , then z 2 =
∠AOB = z1eiθ (see Fig 6.47). Y
| z1 |
B(z2)
Let z1 = r1eiα and z 2 = r2eiβ where = , | z 2 | r2 .
| z1 | r1=
r2
iβ A(z1)
z 2 r2e r  r1
Then= = 2 ei(β−α ) 
z1 r eiα r1  X
1 O

z2 r2 | z2 |
Thus, = eiθ (∵ β − α = θ ) ⇒ z2 = z eiθ Figure 6.47
z1 r1 | z1 | 1
6 . 3 0 | Complex Number

PLANCESS CONCEPTS

Multiplication of a complex number,z with i.

 π π  π  π 
Let z = r (cos θ + isin θ) and i =  cos + isin  , then iz = r cos  + θ  + isin  + θ   .
 2 2  2  2 
Hence, iz can be obtained by rotating the vector z by right angle in the positive sense. And so on, to
multiply a vector by – 1 is to turn it through two right angles.
Thus, multiplying a vector by (cos θ + isin θ) is to turn it through the angle θ in the positive sense.
Anvit Tawar (JEE 2009, AIR 9)

Illustration 37: Suppose A( z1 ), B( z 2 ) and C( z3 ) are the vertices of an equilateral triangle inscribed in the circle
| z | = 2. If z1= 1 + 3 i , then z 2 and z3 are respectively.

(a) −2, 1 − 3 i (b) − 1 + 3 i, − 2


(c) −2, − 1 + 3 i (d) −2, 2 + 3 i  (JEE ADVANCED)

Sol: As we know x + iy = reiθ . Hence by using this formula we can obtain z 2 and z3 .

z1= 1 + 3 i = 2e 3
2 πi 2 πi Y
2π 2π
Since, ∠AOC =and ∠BOC = , z 2 = z1e 3 and z3 = z 2e 3 A(z1)
3 3
5 πi
B(z2) /3
⇒ z3 = 2eπi = 2(cos π + isin π) = −2 and z3 = 2e 3
O
X

  π  π 
= 2 cos  2π −  + isin  2π −   C(z3)
  3   3  Figure 6.48
 π π 1 3 
= 2 cos − isin  = 2  − i = 1 − 3 i .
 3 3  2 2 

PROBLEM-SOLVING TACTICS

(a) On a complex plane, a complex number represents a point.


(b) In case of division and modulus of a complex number, the conjugates are very useful.

(c) For questions related to locus and for equations, use the algebraic form of the complex number.

(d) Polar form of a complex number is particularly useful in multiplication and division of complex numbers. It
directly gives the modulus and the argument of the complex number.

(e) Translate unfamiliar statements by changing z into x+iy.

(f) Multiplying by cos θ corresponds to rotation by angle θ about O in the positive sense.
M a them a ti cs | 6.31

a + ib
(g) To put the complex number in the form A + iB we should multiply the numerator and the denominator
c + id
by the conjugate of the denominator.

(h) Care should be taken while calculating the argument of a complex number. If z = a + ib, then arg(z) is not
b
always equal to  tan−1   . To find the argument of a complex number, first determine the quadrant in which
a
it lies, and then proceed to find the angle it makes with the positive x-axis.
b π −3π
For example, if z = – 1 – i, the formula tan−1   gives the argument as  , while the actual argument is  .
a
  4 4

FORMULAE SHEET

(a) Complex number z= x + i y , where x, y ∈ R and i= −1 .

(b) If z = x + iy then its conjugate z = x – iy.

(c) Modulus of z, i.e. | z | = x2 + y 2

 −1 y
 tan x > 0, y > 0
 x
 y
 π − tan−1 x < 0, y > 0
 x
(d) Argument of z, i.e. θ =
−π + tan−1 y
x < 0, y < 0
 x

 −1 y
 − tan x
x > 0, y < 0

y -1 y
where,  =tan x
=- =

x
=-+ O =-

0, if x > 0
(e) If y=0, then argument of z, i.e. θ =
π, if x < 0
 π
 , if y > 0
(f) If x=0, then argument of z, i.e. θ = 2
 3 π , if y < 0
 2

(g) In polar form x = rsinθ , therefore


rcosθ and y = = z r ( cosθ + i sinθ )

(h) In exponential form complex number z = reiθ , where e=



cosθ + isinθ .
6 . 3 2 | Complex Number

eix + e−ix eix − e−ix


(i) cos x = and sin x =
2 2i
( j) Important properties of conjugate

(i) z + z = 2Re (z) and z − z = 2 Im(z)


(ii) z= z ⇔ z is purely real
(iii) z + z = 0 ⇔ z is purely imaginary
(iv)
= zz [Re(z)] 2 + [Im(z)] 2

(v) z1 + z 2 = z1 + z2

(vi) z1 − z 2 = z1 − z2

(vii) z1 z 2 = z1 z2

 z1  z1
(viii)
=  z  z if z 2 ≠ 0
 2 2

(k) Important properties of modulus


If z is a complex number, then

(i) | z | = 0 ⇔ z =
0
(ii) | z | = | z | =| −z | =| − z |
(iii) − | z | ≤ Re(z) ≤ | z |
(iv) − | z | ≤ Im(z) ≤ | z |
(v) zz = | z | 2
If z1 , z 2 are two complex numbers, then

(i) | z1 z 2 | = | z1 | | z 2 |

(ii) z1 | z1 | , if z ≠ 0
= 2
z2 | z2 |
2 2 2 2 2
(iii) | z1 + z 2 | = | z1 | + | z 2 | + z1 z 2 + z1 z2 = | z1 | + | z 2 | +2Re (z1 z2 )
2 2 2
(iv) | z1 − z 2 | = | z1 | + | z 2 | − z1 z 2 − z1 z2 = | z1 | 2 + | z 2 | 2 −2Re (z1 z2 )

(l) Important properties of argument


(i) arg(z) = − arg(z)
(ii) arg(z
= 1 z 2 ) arg(z1 ) + arg(z 2 )

In fact arg(z1=
z 2 ) arg(z1 ) + arg(z 2 ) + 2kπ
0, if − π < arg(z1 ) + arg(z 2 ) ≤ π

where,k= 1, if − 2π < arg(z1 ) + arg(z 2 ) ≤ −π
−1, if π < arg(z1 ) + arg(z 2 ) ≤ 2π

(iii) arg(z
= 1 z2 ) arg(z1 ) − arg(z 2 )

 z1 
(iv) arg=
  arg(z1 ) − arg(z 2 )
 z2 
M a them a ti cs | 6.33

π
(v) | z1 + z 2 | = | z1 − z 2 | ⇔ arg(z1 ) − arg(z 2 ) =
2
(vi) | z1 + z 2 | = | z1 | + | z 2 | ⇔ arg(z1 ) =
arg(z 2 )

If=
z1 r1 (cos θ1 + isin θ1 ) and=
z 2 r2 (cos θ2 + isin θ2 ) , then

z 2 | 2 | z1 | 2 + | z 2 | 2 +2 | z1 | | z 2 | cos(θ1 − θ2 ) = r12 + r22 + 2r1r2 cos(θ1 − θ2 )


(vii) | z1 + =

z 2 | 2 | z1 | 2 + | z 2 | 2 −2 | z1 | | z 2 | cos(θ1 − θ2 ) = r12 + r22 − 2r1r2 cos(θ1 − θ2 )


(viii) | z1 − =

(m) Triangle on complex plane

z1 + z 2 + z3
(i) Centroid (G), zG =
3
a z1 + b z 2 + c z3
(ii) Incentre (I), zI =
a+b+c
z1 tanA + z 2 tanB + z3 tanC
(iii) Orthocentre (H), zH
∑ tanA
z1 (sin2A) + z 2 (sin2B) + z3 (sin2C)
(iv) Circumcentre (S), zS
sin2A + sin2B + sin2C

( cos θ + i sin θ=
)
n
(n) cosnθ + i sin nθ

 z +x z −x
(o) z =x + i y =± +i   for  y > 0
 2 2 
 
(p) Distance between A (z1 ) and B(z 2 ) is given by | z 2 − z1 |
(q) Section formula: The point P (z) which divides the join of the segment AB in the ratio m : n
mz 2 + nz1
is given by z = .
m+n
1
(r) Midpoint formula: z = (z + z 2 ) .
2 1
(s) Equation of a straight line
(i) Non-parametric form: z (z1 − z2 ) − z(z1 − z 2 ) + z1 z2 − z 2 z1 =
0
(ii) Parametric form: z = tz1 + (1 − t)z 2
(iii) General equation of straight line: az + az + b =0
z1 − z 2
(t) Complex slope of a line, µ = . Two lines with complex slopes µ1 and µ2 are
z1 − z2
(i) Parallel, if µ1 =µ2
(ii) Perpendicular, if µ1 + µ2 = 0
(u) Equation of a circle: | z − z 0 | =
r
6 . 3 4 | Complex Number

Solved Examples

JEE Main/Boards ∴ L.H.S = 1 + ω3m+1 + ω6m+ 2 = 1 + ω + ω2 = 0

Example 1: f z1 and z 2 are 1 – i, – 2 + 4i respectively. (b) Let n = 3m + 2


z z 
Find Im  1 2  . 1 + ω3m+ 2 + ω6m+ 4 = 1 + ω2 + ω4 = 1 + ω2 + ω = 0.
 z1 
z1 z 2 (1 − i) ( −2 + 4i) −2 + 2i + 4i + 4 z −3
Sol: = = Example 4: Show that = 2 represents a circle.
z1 1+i 1+i z+3

2 + 6i 1 − i 2 + 6i − 2i + 6
= × = = 4 + 2i Sol: Consider z = x + iy and then by taking modulus we
1+i 1−i 2 will get the result.
z z 
2.
∴ Im  1 2  = Let z = x + iy
 z1 
z −3 x − 3 + iy
∴ 2 ⇒
= 2
=
Example 2: Find the square root of z = – 7 – 24i. z+3 x + 3 + iy

Sol: Consider z 0= x + iy be a square root then | 2 22 | x + 3 + iy | 2


∴ | x − 3 + iy=
z 02 =−7 − 24i .
2
– 7 – 24i = x − y + 2ixy2
or (
(x − 3) 2 + y 2= 4 (x + 3) 2 + y 2 )
⇒ 3x2 + 3y 2 + 30x + 27 =
0
Equating real and imaginary parts we get
which represents a circle.
x2 − y 2 =
−7  … (i)

and 2xy = – 24  … (ii) Example 5: If | z1 | = | z 2 | = ……. = | zn |= 1

(x2 + y 2 ) 2 = (x2 − y 2 )2 + 4x2 y 2 1 1 1


prove that | z1 + z 2 + ……. + z=
n | + + ……. +
z1 z 2 zn
= ( −7)2 + ( −24) 2 = 625

∴ x2 + y 2 = 
25 … (iii) Sol: | z j | = 1 ⇒ z j z j= 1 ∀ j= 1, …… , n

Solving (i) and (iii), we get, ( zz = | z 2 |)

(x, y) = (3, – 4);(– 3, 4) by (ii) L.H.S.

∴ z 0 =±(3 − 4i) . 1 1 1
| z1 + z 2 + ……. + z=
n | + + ……. + =
z1 z2 zn
Example 3: If n is a positive integer and ω be an
imaginary cube root of unity, prove that 1 1 1 1
+ + + ……. +
z1 z 2 z3 zn
3, when n is a mulitpleof 3
1 + ωn + ω2n 
0, when n is not a mulitpleof 3 1 1 1 1
= + + + ……. + = R.H.S.
z1 z 2 z3 zn
Sol: Case I: n = 3m; m ∈ I
∴ 1 + ωn + ω2n = 1 + ω3m + ω6m
Example 6: If | z1 + z 2 | = | z1 − z 2 | , prove that
3
= 1 + 1 + 1 [ ω =1] =3 π
argz1 − argz 2 = odd multiple of .
2
Case II: n = 3m + 1 or 3m + 2; m ∈ I
Sol: As we know | z | = z.z . Apply this formula and
(a) Let n = 3m + 1
M a them a ti cs | 6.35

consider
= z r ( cosθ + i sinθ ) . Equating real and imaginary parts.
n
| z1 + z 2 | 2 =| z1 − z 2 | 2 nπ
2 2 cos = a0 – a2 + a4 + …..
4
⇒ (z1 + z 2 ) (z1 + z2 ) = (z1 − z 2 ) (z1 − z2 ) or n

2 2 sin = a1 – a3 + a5 + …..
z1 z1 + z 2 z2 + z 2 z1 + z1 z2 = z1 z1 + z 2 z2 − z 2 z1 − z1 z2 4

or 0 ; Re (z1 z2 ) = 0
2(z 2 z1 + z1 z2 ) = Example 9: Solve the equation zn–1 = z :n∈N
Let=
z1 r1 (cos θ1 + isin θ1 ) and=
z 2 r2 (cos θ2 + isin θ2 );
Sol: Apply modulus on both side.
then
= z1 z2 r1r2 ( cos(θ1 − θ2 ) + isin(θ1 − θ2 ) )
Zn–1 = z ; |z|n–1 = | z | = |z|
∴ ∴= ( as Re(z1 z2 ) 0 )
cos (θ1 − θ2 ) 0=
∴ |z| = 0 or |z| = 1 If |z| = 0 then z = 0,
π
θ1 − θ2 = odd multiple of .
2 Let |z| = 1; then, zn = z z = 1
2mπ 2mπ
Example 7: If | z – 1 | < 3, prove that | iz + 3 − 5i |< 8 . ∴ z cos
= + isin : m = 0, 1, …., n – 1
n n
Sol: Here we have to reduce iz + 3 – 5i as the sum
1 − iz
of two complex numbers containing z – 1. because we Example 10: If z = x + iy and ω =
z −i
have to use
with |w| =1, show that, z lies on the real axis.
| z – 1 | < 3.
Sol: Substitute value of ω in |w| =1.
| iz + 3 – 5i | = | iz – i + 3 – 4i |
1 − iz
| ω |= = 1 ⇒ |1 – iz| = |z – i|
= | 3 – 4i + i (z – 1) | ≤ | 3 − 4i | + | i (z − 1) | z −i
(by triangle inequality) < 5 + 1 · 3 = 8 or, |1 – ix + y| = |x + i(y – 1)|
or, (1 + y)2 + x2 = x2 + (y – 1)2 or, 4y = 0
Example 8: If (1 + x) = a0 + a1x + a2x +……….…+ anx ,
n 2 n
Hence z lies on the real axis.
then show that
n
nπ Example 11: If a complex number z lies in the interior
(a) a0 – a2 + a4 + ….. = 2 2 cos
4 or on the boundary of a circle of radius as 3 and centre
n
nπ at (0, – 4) then greatest and least value of |z + 1| are-
(b) a1 – a3 + a5 + … = 2 2 sin
4
(A) 3 + 17 , 17 − 3 (B) 6, 1
Sol: Simply put x = i in the given expansion and then
by using formula (C) 17 , 1 (D) 3, 1

z r ( cosθ + i sinθ ) and ( cos θ + i sin θ )


n
= Sol: Greatest and least value of |z + 1| means maximum
= cosnθ + i sin nθ , we can solve this problem. and minimum distance of circle from the point (– 1, 0).
In circle greatest and least distance of it from any point
Put x = i in the given expansion is along the normal.
Y
(1 + i)n = a0 + a1i + a2i2 + …+ anin.
n
  π π 
 2  cos + isin   (-1,0) (0,0)
  4 4  X

= (a0 – a2 + a4 - …) + i (a1 – a3 + a5 - ….)


(0,-4)
n/2  nπ nπ 
2  cos + isin 
 4 4 

= (a0 – a2 + a4 + …….) + i(a1 – a3 + a5 + …..)


6 . 3 6 | Complex Number

∴ Greatest distance = 3 + 12 + 42 = 3 + 17 equilateral triangle. Also show that |z1| = |z2| = |z3|.

Least distance = 12 + 42 − 3 = 17 − 3 Sol: Use algebra to solve this problem.

Given z1 + z2 + z3 = 0, and from 2nd relation z2z3 + z3z1


Example 12: Find the equation of the circle for which + z1z2 = 0
 z − 6 − 2i 
arg   = π/ 4. ∴ z2z3 = – z1(z2 + z3) = – z1 (– z1) = z12
 z − 2 − 2i 
∴ z13 = z1z2z3 = z23 = z33
 z − 6 − 2i 
Sol: arg   = π / 4 represent a major arc of
 z − 2 − 2i  ∴ |z1|3 = |z2|3 = |z3|3
circle of which Line joining (6, 2)) and (2, 2) is a chord
Above shows that distance of origin from A, B, C is
π same.
that subtends an angle at circumference.
4
Origin is circumcentre, but z1 + z2 + z3 = 0

Z implies that centroid is also at the origin so that the


A O triangle must be equilateral.
A
A
A(2,2) M B(6,2) JEE Advanced/Boards
Example 1: For constant c ≥ 1, find all complex numbers
Clearly AB is parallel to real (x) axis, M is mid-point, z satisfying the equation z + c | z + 1| + i = 0
M ≡ (4, 2), OM = AM = 2
Sol: Solve this by putting z = x + iy.
∴ O = (4, 4) and OA = OM + AM = 2 2 Equation of
2 2 2
Let z = x + iy.
required circle is
The equation z + c |z + 1| + i = 0 becomes
| z − 4 − 4i | =
2 2
x + iy + c (x + 1)2 + y 2 + i =0
Example 13: If |z| ≥ 3, prove that the least value of
1 8 or x + c (x + 1)2 + y 2 + i(y + 1) =
0
z+ is .
z 3
Equating real and imaginary parts, we get
1 1
Sol: Here z + ≥ | z | − . y+1=0⇒y=–1 … (i)
z |z|
Now |z| ≥ 3
and x + c (x + 1)2 + y 2 =
0: x < 0  ...(ii)
1 1 1 1
∴ ≤ or – ≥−  … (i) Solving (i) and (ii), we get
|z| 3 |z| 3
Adding the two like inequalities x+c (x + 1)2 + 1 =0 or x2 = c2[(x + 1)2 + 1]
1 1 8
|z|− ≥3 − =  … (ii) or (c2 – 1) x2 + 2c2x + 2c2 = 0
|z| 3 3
1 8 If c =1, then x = – 1. Let c > 1 ; then,
Hence from (i) and (ii), we get z + ≥
z 3
8 −2c2 ± 4c 4 − 8c2 (c2 − 1) −c2 ± c 2 − c2
∴ Least value is x= =
3 2(c2 − 1) c2 − 1

As x is real and c > 1, we have: 1 < c ≤ 2


Example 14: If z1, z2, z3 are non-zero complex numbers
such that z1 + z2 + z3 = 0 and z1−1 + z 2−1 + z3−1 =
0 then (Thus, for c > 2 , there is no solution). Since both
prove that the given points are the vertices of an values of x satisfy (ii), both values are admissible.
M a them a ti cs | 6.37

Example 2: Find the sixth roots of z = 64i. Re(z) > 3 (see the Figure above)
π π (b) Let z = x + iy, then, x > 0 and y > 0
Sol: Here i = cos + isin and sixth root of z
2 2
i.e. zr = z1/6. y π y π
arg z = tan−1 tan ≤ ≤ tan
x 6 x 4
 π π
=z 64  cos + isin  ∴ zr = z1/6 Y
 2 2
y=x
 π π
 2rπ + 2rπ + 
= 2 cos 2 + isin 2 
 6 6  arg(z)=6
 
X
Where r = 0, 1, 2, 3, 4, 5 O

The roots z0, z1, z2, z3, z4, z5 are given by 1 y


≤ ≤ 1 ; x ≤ 3y and y ≤ x
 π π  3 x
=z 0 2  cos + isin 
 12 12  Hence the given inequality represents the region
1
 5π 5π  bounded by the rays y = x and y = x except the
=z1 2  cos + isin  3
 12 12  origin.

=
 9π 9π 
z 2 2  cos + isin  Example 4: If z12 + z22 – 2z1z2 cos θ = 0, show that the
 12 12  points z1, z2 and the origin, in the argand plane, are the
vertices of an isosceles triangle.
 13π 13π   π π 
=z3 2  cos + isin −2  cos + isin 
 =
 12 12   12 12  Sol: By using formula of roots of quadratic equation we
can solve it.
 17π 17π   5π 5π 
=z 4 2  cos + isin =−2  cos + isin  z12 + z22 - 2z1z2 cos θ = 0
 12 12   12 12 
2
z  z 
 21π 21π   9π 9π  ⇒  1  − 2  1  cos θ + 1 = 0
=z5 2  cos + isin −2  cos + isin 
 =
 12 12   12 12   z2   z2 

 z  2cos θ ± 4 cos2 θ − 4
⇒  1  =
Example 3: Locate the region in the Argand plane for
 z2  2
the complex number z satisfying
π π
(a) |z – 4| < |z – 2| (b) ≤ arg z ≤ B(z)
6 4
Sol: Consider z = x + iy and solve by using properties B(z1)
of modulus and argument.
(a) Let z = x + iy O

|x + iy – 4| < |x + iy – 2| = cos θ ± i sin θ


(x – 4) + y < (x – 2) + y or – 4x + 12 < 0
2 2 2 2
z1
⇒ = | cos θ ± i sin q| – 1
y z2
z1
⇒ =1 ⇒ |z1| = |z2| or OA = OB
z2
x’ x
O Hence points A(z), B(z) and the origin are the vertices of
an isosceles triangle.

y’ x=3
6 . 3 8 | Complex Number

Example 5: Let three vertices A, B, C (taken in clock


eiA −eiB −eiC
wise order) of an isosceles right angled triangle with
right angle at C, be affixes of complex numbers z1, z2, =−1 −eiA eiB −eiC , by (2)
z3 respectively. Show that (z1 – z2)2 = 2(z1 – z3) (z3 – z2). −eiA −eiB eiC
z 2 − z3
Sol: Here = e−iπ /2 . Therefore solve it using
z1 − z3 Take eiA, eiB and eiC common from C1, C2 and C3 and
algebra method. again put ei(A+B+C) = eiπ = –1.
π 1 −1 −1
Given CB = CA and angle ∠C =
2 ∴ ∆ = (– 1) (– 1) −1 1 −1
z 2 − z3 −1 −1 1
= e−iπ /2 or (z3 – z2)2 = i2(z1 – z3)2
z1 − z3
Now make two zeros and expand
(z3 – z2)2 = – (z1 – z3)2
∆ = – 4 which is purely real.
A(z1)

Example 7: Prove that |a + b|2 + |a – b|2


= 2(|a|2 + |b|2). Interpret the result geometrically and

deduce that c + c2 − d2 + c − c2 − d2 = |c + d| + |c
C(z3) B(z2)
– d| ; all numbers involved being complex
or

z32 + z22 – 2z2z3 + z12 + z32 – 2z1z3 = 0 Sol: By using algebra of complex number and modulus
property we can prove given expresson. And then by
Add and subtract 2z1z2, we get using Appolonius theorem we can interpret the result
geometrically.
z12 + z22 – 2z1z2 + 2z32 – 2z2z3 – 2z1z3 + 2z1z2 = 0, or
S = |a + b|2 + |a – b|2
(z1 – z2) + 2[z3(z3 – z2) – z1(z3 – z2)] = 0 or
2

= |a|2+ |b|2 + 2Re(ab) +|a|2+ |b|2 - 2Re(ab) = 2(|a|2 +


(z1 – z2)2 + 2(z3 – z1) (z3 – z2) = 0, or
|b|2) (proved)
(z1 – z2) = 2(z1 – z3) (z3 – z2).
2
Now |a + b|2 + |a – b|2 = 2(|a|2 + |b|2)

Example 6: If A, B, C be the angles of triangle then This is nothing but Appolonius theorem. In DOAB, M
is midpoint of AB on applying Appolonius theorem we
e2iA e−iC e−iB get
prove that e−iC e2iB e−iA is purely real. (b)
−iB −iA 2iC B(a+b)
e e e

Sol: Here A+B+C = π, therefore epi = cos π+i sin π = – 1. M(a)


And by using properties of matrices we can solve this O
problem.
e–pi = – 1  … (i)
A(a - b)

OA2 + OB2 = 2(AM2 + OM2)


ei(B+C) = ei(π–A) = epi e–iA = – e–iA
c+d c−d
e–i(B+C) = – eiA  … (ii) Now take a = and b =
2 2
Take eiA, eiB and eiC common from R1, R2 and R3 Then using result
respectively. ∆ = ei(A+B+C)
|a + b|2 + |a – b|2 = 2(|a|2 + |b|2)
iA −i(A + C) −i(A +B)
e e e  2 2
c+d c−d 
e −i(B + C)
e iB
e −i(B + A)
=RHS 2  + = |c + d| + |c – d|
 2 2 
−i(B + C) −i(C + A)
e e eiC  
M a them a ti cs | 6.39

2
2
 z1 a 1
 c+d c−d  c+d c−d 
L.H.S. 
= +  +  −  we get z 2 b 1 =0
 2 2   2 2 
    z3 c 1
On simplifying we get
Example 9: If b1 + b2 + b3 + b4 = 0 where b1 etc. are
L.H.S. = c + c2 − d2 + c − c2 − d2 non-zero real numbers, sum of no two being zero,
and b1z1 + b2z2 + b3z3 + b4z4 = 0 where no three of the
points z1, z2, z3, z4 are collinear then prove that the four
Example 8: Show that the triangles whose vertices are
point concyclic if b1b2 |z1 – z2|2 = b3b4 |z3 – z4|2.
z1, z2, z3 and a, b, c are

z1 a 1 Sol: Here the four points A, B, C, D will be concyclic if


PA.PB = PC.CD. therefore obtain PA, PB, PC and CD and
similar if z 2 b 1 = 0.
simplify.
z3 c 1
Here b1 + b2 = – (b3 + b4)
Sol: Consider triangle ABC and DEF are similar, therefore Also b1z1 + b2z2 = – (b3z3 + b4z4)
AB BC
= and ∠ABC = ∠DEF.
DE EF b1 z1 + b2 z 2 b3 z3 + b 4 z 4
Dividing these, =
Suppose z1, z2, z3 are given by A, B, C respectively and a, b1 + b2 b3 + b 4
b, c are given by D, E, F respectively. Since the triangle
AB BC z3
ABC and DEF are similar and
z1
= A C
DE EF b2
b4
∠ABC = ∠DEF = α(say)
P b1
 z − z2  a−b
b3
We have ∠B = arg  1
 z − z  = arg  c − b 
z4 z2
 3 2  D B
z1 − z 2 AB
⇒ = (cos α + i sin α)  … (i)
z3 − z 2 BC The left side gives the point that divides the line
DE segment joining A(z1), B(z2) in the ratio b2: b1 and the
a−b
and = (cos α + i sin α)  … (ii) right side gives the point that divides the line segment
c −b EF
joining the points C(z3), D(z4) in the ratio b4: b3. So the
A(Z1) D(a) line segments intersect at P which is
b1 z1 + b2 z 2 b3 z3 + b 4 z 4
Represented by as well as
b1 + b2 b3 + b 4
Now, AB = |z1 – z2|
B(X2) C(z3) E(b) F(c)
b2 b1
∴ PA
= z 2 ) & PB
(z1 − = (z1 − z 2 )
b1 + b2 b1 + b2
AB DE
Since AB = BC we have =
DE EF BC EF Also, CD = |z3 – z4|

Thus, from (i) and (ii) we get b4 b3


∴ PC
= z 4 ) & PD
(z3 − = (z3 − z 4 )
z1 − z 2 z − z2 a − b b3 + b 4 b3 + b 4
a−b
= ⇒ 1 =0
z3 − z 2 c − b z3 − z 2 c − b The four points A, B, C, D will be concyclic if PA.PB =
PC.CD
z1 − z 2 a−b 0 b1b2 2 b3b 4 2
⇒ z2 b 1 =0 i.e. = z1 − z 2 z3 − z 4
(b1 + b2 )2 (b3 + b 4 )2
z3 − z 2 c −b 0
i.e. b1b2 |z1 – z2|2 = b3b4 |z3 – z4|2
Applying R1 → R1 + R2 and R3 → R3 + R2
(∵ b1 + b2 = – (b3 + b4)}
6 . 4 0 | Complex Number

Example 10: Show that all the roots of the equation 2i sinnθ
= = itannθ . Taking negative sign,
z cos q0 + z cos q1 + z cos q2+......... + z cos qn–1 +
n n–1 n–2
2cosnθ
1 z 2n − 1 −2isin nθ
cos qn = 2 lie outside the circle | z |= where q0, q1 etc. similarly we get = = – i tan nθ
2 z 2n + 1 2cosnθ
are real.
z 2n − 1
∴ = | ± i tan nq| = |tan nq|,
Sol: By using triangle inequality. z 2n + 1
Here |zn cos q0 + zn–1 cos q1 + zn–2 cos q2 + …… + z For |± i| = 1.
cos qn–1 + cos qn| = 2  …. (i)

By triangle inequality. Example 12: Find the complex number z which satisfies
the condition |z – 2 + 2i| = 1 and has the least absolute
2 = |zn cos θ n + 2n–1 cos q1 + 2n–1 cos q2 + ….. + z
value.
cos qn–1 + cos qn | ≤ | zn cos qn| + |zn–1 cos q1|+
Sol: Here z – 2 + 2i = cos θ + i sin θ, therefore by obtaing
|zn–2 cos q2| + …..++ |z cos qn–1| + |cos qn| modulus of z we can solve above problem.

= |zn| |cos qn| + |zn–1| |cos qn| + ……..+ |z| |cos qn–1| + |z – 2 + 2i| = 1

|cos qn| ≤ |z|n + |z|n–1 + …. + |z| + 1 ⇒ z – 2 + 2i = cos θ + i sin θ

(∵ | cos q1| ≤ 1 and |zn+1| = |z|n+1) Where θ is some real number.

1− | z |n+1 1 1 ⇒ z = (2 + cos θ) + (sin θ – 2)i


= < ∴ 2<
1− | z | 1− | z | 1− | z |
⇒ |z| = [(2 + cos θ)2 + (sin θ – 2)2]1/2
1
So 1 – |z| is positive and 1-|z| <
2 = [8 + cos2 θ + sin2 θ + 4(cos θ – sin θ)]1/2
1 1
∴ |z| > 1 – =
1/2
  π 
2 2 = 9 + 4 2 cos  θ +  
  4 
1
∴ All z satisfying (i) lie outside the circle | z |=
2 |z| will be least if cos (θ + π/4) is least, that is, if cos

1 z 2n − 1 (θ + π/4)= – 1 or θ = . Thus, least value of |z| is
Example 11: If z + = 2cos θ , prove that 4
z
= |tan nq|. z 2n + 1
( )  1   1 
1/2
9−4 2 for z =2 −  + i − 2
Sol: By using formula of roots of quadratic equation,  2  2 
we can solve this problem.
Example 13: For every real number c ≥ 0, find all the
1
Here z + = 2 cos θ; complex numbers z which satisfy the equation. 2|z| – 4
z cz + 1 + ic = 0.
∴ z2 – 2 cos θ. z + 1 = 0
Sol: Substitute z = x + iy and equate real and imaginary
2cos θ ± 4 cos2 θ − 4 part to zero.
∴ z= = cos θ ± i sin θ
2
2 x2 + y 2 - 4c (x + iy) + 1 + ic = 0
Taking positive sign, z = cos θ + i sin θ
1 1
∴ = (cos θ + i sin θ)–1 = cos θ – i sin θ ∴ – 4 cy + c = 0 ⇒ y =  … (i)
z 4
1 1
2n zn − n 2 x2 + − 4cx + 1 =0 or
z −1 (cos θ + isin θ)n − (cos θ − isin θ)n
∴ = z
=
16
z + 1 zn + 1
2n
(cos θ + isin θ)n + (cos θ − isin θ)n  1 
n 4  x2 +  = (4 cx – 1)
2
z  16 
3
=
cosnθ + isin θ − (cosnθ − isinnθ) 4x2 (4c2 – 1) – 8 cx + =0
cosnθ + isinnθ + (cosnθ − isinnθ 4
M a them a ti cs | 6.41

π
8c ± 64c2 − 12(4c2 − 1) z3 − z1 z3 − z1 i 4  π π
∴ x= = = e 2  cos + isin 
8(4c2 − 1) z 2 − z1 z 2 − z1  4 4

4c ± 4c2 + 3 ⇒ z3 = z1 + (z2 – z1) (1 + i)


or x =  … (ii)
4(4c2 − 1) Similarly z4 = z1 + i(z3 – z1)
x is real as c ≥ 0, z = (x, y) as given by (i) and (ii), c ≥ 0. A(z1) D(z4)

Example 14: Consider a square ABCD such that z1, z2, z3, /4
and z4 represent its vertices A, B, C and D respectively.
Express ‘z3’ and ‘z4’ in terms of z1 & z2.

Sol: Consider the rotation of AB about A through an


π B(z2) C(z3)
angle .
4
π
z3 − z1 z3 − z1 i 4
Therefore = e .
z 2 − z1 z 2 − z1

JEE Main/Boards

Exercise 1

Q.10 If z = 1 + i tan α, where π < α < . find the value
Q.1 Find all non-zero complex numbers z satisfying of |z| cos α. 2
z = iz 2 .
Q.11 If 1, ω, w2 be the cube roots of unity, find the roots
1 + 2i + 3i2
Q.2 Express in the form A + iB. of the equation (x – 1)3 + 8 = 0.
1 − 2i + 3i2

Q.3 Find x and y if (x + iy) (2 – 3i) =(4 + i) Q.12 If |z| < 4, prove that |i z + 3 – 4i| < 9.

(1 + i) x − 2i (2 − 3i)y + i
Q.4 Find x and y if + i
= Q.13 2 + i 3 is a vertex of square inscribed in circle |z –
3+i 3−i
1| =2. Find other vertices.
Q.5 If x = a + b, y = aα + bβ and z = aβ +bα, where α and
β are complex cube roots of unity, show that xyz = a3 + b3. Q.14 Find the centre and radius of the circle formed
by the points represented by z = x +iy satisfying the
1 + 7i |z−α|
Q.6 in the polar form. relation = k(k ≠ 1) where α & β are constant
(2 − i)2 | z −β|
Q.7 Find the square root of – 8 – 6i. complex number’s given by α =α1 + iα2 & β =β1 + iβ2
Q.15 Prove that there exists no complex number z such
Q.8 Find the value of smallest positive integer n, for 1 a

n that | z |< and ∑ ar zr =1 where |ar| < 2.


1 + i 3 r =1
which   =1.
1 −i Q.16 Let a complex number α, α ≠ 11, be a root of
Q.9 Show that the complex number z = x + iy which the equation zp + q – zp – zq + 1 = 0, where p, q are
z − 5i distinct primes. Show that either 1 + α + α2 + ……. +
satisfies the equation = 1 lies on the x-axis. αp–1 = 0 or 1 + α + α2 + ……. + αq–1 = 0, but not both
z + 5i
together.
6 . 4 2 | Complex Number

Q.17 Show that the area of the triangle on the Argand Q.28 If |z| ≤ 1, |w| ≤ 1, show that
diagram formed by the complex numbers: z, iz and
|z – w|2 ≤ (|z| – |w|)2 + (arg z – arg w)2.
1
z + iz is: | z |2 .
2
Q.29 Let A and B be two complex numbers such
Q.18 If iz3 + z2 – z + i = 0 then show that |z| =1. A B
that + = 1, prove that the origin and the points
B A
Q.19 Find the value of the expression represented by A and B form the vertices of an
equilateral triangle.
1(2 – ω) (2 – w2) + 2(3 – ω) (3 – w2) + ….

+ (n – 1) (n – ω) (n – w2) where ω is an imaginary cube Q.30 Let z1, z2, z3 be three complex numbers and a, b, c
root of unity. be real number not all zero, such that a + b + c = 0 and
az1 + bz2 + cz3 = 0.
Q.20 If x =
1
2
( )
5 − 3i , then find the value of Show that z1, z2, z3 are collinear.
x4 – x3 – 12x2 + 23x + 12.
Q.31 If |z – 4 + 3i | ≤ 2, find the least and the greatest
Q.21 Let the complex numbers z1, z2 and z3 be the vertices values of |z| and hence find the limits between which
of an equilateral triangle. Let z0 be the circumcentre of |z| lies.
the triangle. Then prove that: z12 + z22 + z32 = 3z02. z1 − z 2
Q.32 If |z1| < 1 and < 1, then show that |z2| < 1.
1 − z1 z 2
Q.22 If z1, z2, z3 are the vertices of an isosceles triangle,
right angled at z2, prove that z12 + 2z22 + z32 = 2z2 (z1 + z3). | z |2 − | z | + 1
Q.33 Find the locus of points z if log < 2.
3 2+ | z |
Q.23 Show that the equation
2 2 2
Q.34 For complex numbers z and ω, prove that |z|2ω – |
2
A B C H ω |2 z = z – ω if and only if z = w or zω =1 .
+ + + ...... = x+,
x −a x −b x −c x −h
Where A, B, C, …., a, b, c, …. and  are real, cannot have
imaginary roots. Exercise 2
Single Correct Choice Type
Q.24 Find the common roots of the equation
z3 + 2z2 + 2z + 1 = 0 and z1985 + z100 + 1 = 0. Q.1 |z + 4| ≤ 3, Z∈ C: then the greatest and least value
of |z + 1| are:
Q.25 If n is an odd integer greater than 3 but not a (A) (7, 1) (B) (6, 1) (C) (6, 0) (D) None
multiple of 3, prove that [(x + y)n – xn – yn] is divisible by
xy (x + y) (x2 + xy + y2). Q.2 The maximum & minimum values of |z +1| when |z
+ 3| ≤ 3 are
Q.26 If α and β are any two complex numbers, (A) (5, 0) (B) (6, 0) (C) (7, 1) (D) (5, 1)

show that α + α2 − β2 + α − α2 − β2
Q.3 The points z1 = 3 + 3 i and z2 = 2 3 + 6i are given
= |α + β| + |α – β| on a complex plane. The complex number lying on the
bisector of the angle formed by the vectors z1 and z2 is:

Q.27 Let z1 = 10 + 6i and z2 = 4 + 6i. If z is any complex = (3 + 2 3) 3+2


(A) z + i
z − z1 π 2 2
number such that the argument of is , then
z − z2 4 (B) z = 5 + 5i
prove that |z – 7 – 9i| =3 2 . (C) z = – 1 – I
(D) None of these
M a them a ti cs | 6.43

Q.4 If z1, z2, z3, z4 are the vertices of a square in that Q.9 If q1, q2, q3 are the roots of the equation, x3 + 64 = 0,
order, then which of the following do(es) not hold q1 q2 q3
good?
then the value of the determinant q2 q3 q1 is:
z1 − z 2
(A) is purely imaginary q3 q1 q2
z3 − z 2 (A) 1 (B) 4
z1 − z3 (C) 10 (D) none of these
(B) is purely imaginary
z2 − z 4

z1 − z 2 Q.10 z = (3 + 7i) (p + iq) where p, q ∈ I – {0} purely


(C) is purely imaginary imaginary then minimum value of |z|2 is
z3 − z 4

(D) None of these 3364


(A) 0 (B) 58 (C) (D) 3364
3

Q.5 Let z1, z2 and z3 be the complex numbers Q.11 On the complex plane triangles OAP & OQR are
representing the vertices of a triangle ABC respectively similar and  (OA) = 1. If the points P and Q denotes the
and a, b, c are lengths of BC, CA, AB. If P is a point complex numbers z1 & z2 then the complex number ‘z’
representing the complex number z0 satisfying: denoted by the point R is given by:
a(z1 – z0) + b(z2 – z0) + c(z3 – z0) = 0, then w.r.t. the z1 z2 z1 + z 2
triangle ABC, the point P is its: (A) z1z2 (B) (C) (D)
z2 z1 z2
(A) Centroid (B) Orthocentre
Q.12 If A and B be two complex numbers satisfying
(C) Circumcentre (D) Incentre
A B
+ = 1 . Then the two points represented by A and B
B A
Q.6 Three complex numbers α, β & γ are represented
in the Argand diagram by the three points A, B, C and the origin form the vertices of
respectively. The complex number represented by D (A) An equilateral triangle
where A, B, C, D form a parallelogram with BD on a
(B) An isosceles triangle which is not equilateral
diagonal is:
(C) An isosceles triangle which is not right angled
(A) α – β + g (B) – α + β + g
(D) A right angled triangle
(C) α + β – g (D) α – β – g

Q.13 The solutions of the equation in z, |z|2 – (z + z ) +


Q.7 If the complex number z satisfies the condition |z| ≥ i (z – z ) + 2 = 0 are:
1
3, then the least value of z + is (A) 2 + i, 1 – i (B) 1 + i, 1 – i
z
(C) 1 + 2i, – 1 – I (D) 1 + i, 1 + i
5 8 11
(A) (B) (C) (D) None of these
3 3 3
Q.14 If z1 = – 3 + 5i: z2 = – 5 – 3i and z is a complex
number lying on the line segment joining z1 & z2 then
Q.8 Point z1 & z2 are adjacent vertices of a regular
arg z can be:
octagon. The vertex z3 adjacent to z2(z3 ≠ z1) can be
represented by: 3π π π 5π
(A) – (B) − (C) (D)
1 4 4 6 6
(A) z 2 + (1 ± i)(z1 + z 2 )
2 Q.15 The points of intersection of the two curves
1 |z – 3| = 2 and |z| = 2 in an argand plane are:
(B) z 2 + (1 ± i)(z1 − z 2 )
2
1 1
1 (A) (7 ± i 3) (B) (3 ± i 7)
(C) z 2 + (1 ± i)(z 2 − z1 ) 2 2
2
3 7 7 3
(C) ±i (D) ±i
(D) None of these 2 2 2 2
6 . 4 4 | Complex Number

Q.16 Let z to be complex number having the argument Previous Years’ Questions
π
θ, 0 < θ < and satisfying the equality |z – 3i| = 3.
2 Q.1 The smallest positive integer n for which
6 n
Then cot θ – is equal to: 1 + i
z   = 1 , is (1980)
1 −i
(A) 1 (B) – 1 (C) i (D) – i
(A) 8 (B) 16
Q.17 The locus represented by the equation, |z – 1| + (C) 12 (D) None of these
|z + 1| = 2 is:
(A) An ellipse with foci (1, 0): (– 1, 0) Q.2 The complex numbers z = x + iy which satisfy the
(B) One of the family of circles passing through the z − 5i
equation = 1 lie on  (1981)
points of intersection of the circles |z + 1| = 1 z + 5i
(C) The radical axis of the circles |z – 1| = 1 and |z + 1| = 1 (A) The x-axis

(D) The portion of the real axis between the points (B) The straight line y = 5
(1, 0) and (– 1, 0) (C) A circle passing through the origin
(D) None of these
Q.18 Let P denotes a complex number z on the
Argand’s plane, and Q denotes a complex number
Q.3 If z = x + iy and w = (1 – iz) / (z – i), then |w| = 1
π 
2 | z |2 cos  + θ  where θ = amp z if ‘O’ is the origin, implies that, in the complex plane  (1983)
 4 
(A) z lies on the imaginary axis
then the DOPQ is:
(B) z lies on the real axis
(A) Isosceles but not right angled
(C) z lies on the unit circle
(B) Right angled but not isosceles
(D) None of these
(C) Right isoscles
(D) Equilateral Q.4 The points z1, z2, z3, z4 in the complex plane are the
vertices of a parallelogram taken in order, if and only if
Q.19 Let z1, z2, z3 be three distinct complex numbers  (1983)
satisfying |z1 – 1| = |z2 –1| = |z3 – 1|. (A) z1 + z4 = z2 + z3 (B) z1 + z3 = z2 + z4
If z1 + z2 + z3 = 3 then z1, z2, z3 must represent the (C) z1 + z2 = z3 + z4 (D) None of these
vertices of:
(A) An equilateral triangle Q.5 If z1 and z2 are two non-zero complex numbers
such that |z1 + z2| = |z1| + |z2|, then arg (z1) – arg (z2) is
(B) An isoseles triangles which is not equilateral
equal to (1987)
(C) A right triangle π π
(A) – π (B) – (C) 0 (D)
(D) Nothing definite can be said 2 2
Q.6 The complex numbers sin x + i cos 2x and cos x – i
Q.20 If p = a + bω + cw ; q = b + cω + aw ; and
2 2 sin 2x are conjugate to each other, for (1988)
r = c + aω + bw2 where a, b, c ≠ 0 and ω is the complex (A) x = nπ (B) x = 0
cube root of unity, then:
 1
(C) x =  n +  π (D) No value of x
(A) p + q + r = a + b + c  2
(B) p2 + q2 + r2 = a2 + b2 + c2
Q.7 If ω (≠ 1) is a cube root of unity and (1 +ω)7 = A +
(C) p2 + q2 + r2 = 2(pq + qr + rp)
Bω, then A and B are respectively  (1995)
(D) None of these
(A) 0, 1 (B) 1, 1 (C) 1, 0 (D) – 1, 1
M a them a ti cs | 6.45

Q.8 Let z and w be two non-zero complex numbers z2


such that |z| = |w| and arg (z) + arg(w) = π, then z equals Q.16 If z ≠ 1 and is real, then the point represented
z −1
 (1995)
by the complex number z lies (2012)
(A) w (B) – w (C) ω (D) −ω
(A) Either on the real axis or on a circle passing through
the origin.
Q.9 If ω is an imaginary cube root of unity, then
(B) On a circle with centre at the origin.
(1 + ω – ω2)7 is equal to (1998)
(C) Either on the real axis or on a circle not passing
(A) 128 ω (B) – 128 ω (C) 128 w2 (D) – 128 w2
through the origin.
13
Q.10 The value of sum ∑ (in + i−n+1 ) where i= −1 (D) On the imaginary axis.
n=1 n
equals  (1998)
Q.17 If z is a complex number of unit modulus and
(A) i (B) i – 1 (C) – i (D) 0 1+z 
argument θ , then arg   equals (2013)
1 + z 
6i −3i 1
Q.11 If 4 3i −1 = x + iy , then  (1998) π
(A) −θ (B) θ (C) π − θ (D) −θ
2
20 3 i

(A) x = 3, y = 1 (B) x = 1, y = 1 Q.18 If z is a complex number such that | z | ≥ 2 , then


(C) x = 0, y = 3 (D) x = 0, y = 0 1
the minimum value of z +  (2014)
2

Q.12 If z1, z2 and z3 are complex numbers such that 5


(A) Is equal to
2
1 1 1
| z= 2 | | z3 | =
1 | | z= + + 1,
= (B) Lies in the interval (1, 2)
z1 z 2 z3
then |z1 + z2 + z3| is  (2000) 5
(C) Is strictly greater than
2
(A) Equal to 1 (B) Less than 1 3 5
(D) Is strictly greater than but less than
2 2
(C) Greater than 3 (D) Equal to 3
Q.19 A complex number z is said to be unimodular if
1 3 |z| = 1. Suppose z1 and z2 are complex numbers such
Q.13 Let ω = − + i , then value of the determinant
2 2 z − 2z 2
that 1 is unimodular and z 2 is not unimodular.
1 1 1 2 − z1 z2
1 −1 − ω 2
ω2 is  (2002) Then the point z1 lies on a:  (2015)
1 ω2 ω (A) Straight line parallel to y-axis

(A) 3w (B) 3 ω (ω – 1) (C) 3ω 2 (D) 3 ω (1 – ω) (B) Circle of radius 2


(C) Circle of radius 2
Q.14 If ω (≠1) be a cube root of unity and (1 + w2)n =
(D) Straight line parallel to x-axis
(1 + ω4)n, then the least positive value of n is (2004)
(A) 2 (B) 3 (C) 5 (D) 6 2 + 3i sin θ
Q.20 A value of θ for which is purely
1 − 2i sin θ
Q.15 A man walks a distance of 3 units from the origin imaginary, is: (2016)
towards the North-West (N 45° E) direction. From there,  3
π
he walks a distance of 4 units towards the North-West (A) (B) sin−1  
6  4 
(N 45° W) direction to reach a point P. Then, the position  
of P in the Argand plane is (2007)  1  π
(C) sin−1   (D)
 3 3
(A) 3eiπ / 4 + 4i (B) (3 − 4i)eiπ / 4
(C) (4 + 3i)eiπ / 4 (D) (3 + 4i)eiπ / 4
6 . 4 6 | Complex Number

JEE Advanced/Boards

Exercise 1 Q.8 If the expression z5 – 32 can be factorised into linear


and quadratic factors over real coefficients as (z5 – 32)
Q.1 Prove that with regard to the quadratic equation = (z – 2) (z2 – pz + 4) (z2 – zq + 4) then find the value
z2 + (p + ip′) z + q + iq′ = 0 where p, p′, q, q′ are all real. of (p2 + 2p).

(i) If the equation has one real root then q′2 – pp′ q′ +
Q.9 Let z1 & z2 be any two arbitrary complex numbers
qp′2 = 0
then prove that:
1 z z
(ii) If the equation has two equal roots then p2 – q′2 = | z1 | + | z 2 | ≥ (| z1 | + | z 2 |) 1 + 2 .
4q and pp′ = 2q ′. 2 | z1 | | z 2 |

state whether these equal roots are real or complex. Q.10 Let zi (i = 1, 2, 3, 4) represent the vertices of a
square all of which lie on the sides of the triangle with
Q.2 Let z = 18 + 26i where z0 = x0 + iy0 (x0, y0∈ R) is the vertices (0, 0), (2, 1) and (3, 0). If z1 and z2 are purely
cube roots of z having least positive argument. Find the real, then area of triangle formed by z3, z4 and origin
value of x0 y 0 (x0 + y 0 ) . is m (where m and n are in their lowest form). Find the
value of (m + n).
Q.3 Show that the locus formed by z in the equation
z3 + iz =1 never crosses the coordinate axes in the
Q.11 (i) Let Cr ’s denotes the combinatorial coefficients
Argand’s plane.
in the expansion of (1 + x)n, n ∈ N. If the integers
− Im(z)
Further show that | z |= an = C0 + C3 + C6 + C9 + ……
2Re(z)Im(z) + 1
bn = C1 + C4 + C7 + C10 + ……
Q.4 Consider the diagonal matrix An = dia (d1, d2, d3, …. and cn = C2 + C3 + C8 + C11+…..
dn) of order where
i2 π
then prove that
Di =ai–1, 1 ≤ i ≤ n and α =e n ; i= −1 , is the nth root
of unity. (a) an3 + bn3 + cn3 – 3anbncn = 2n.

Let L: represent the value of Tr. (A7)7. (b) (an – bn)2 + (bn – cn)2 +(cn – an)2=2

M: denotes the value of det (A2n+1) + det (A2n). (ii) Prove the identity:
(C0 – C2 + C4 – C6 + ……… )2
Find the value of (L + M).
+ (C1 – C3 + C5 – C7 + ….. )2 = 2n.
[Note: Tr(A) denotes trace of square matrix A]

Q.12 Let z1, z2, z3, z4 be the vertices A, B, C, D


Q.5 Let z1, z2 ∈ C such that z12 + z22 ∈ R. If z1 (z12 – 3z22) respectively of a square on the Argand diagram taken
in anticlockwise direction then prove that:
= 10 and z2(3z12 – z22)=30. Find the value of (z12 + z22).
(i) 2z2 = (1 + i) z1 + (1 – i) z3 & (ii) 2z4 = (1 – i) z1 + (1 + i) z3
Q.6 If the equation (z + 1) + z = 0 has roots z1, z2, ……
7 7

z7, find the value of Q.13 A function f is defined on the complex number by
7 7 f(z) = (a + bi)z, where ‘a’ and ‘b’ are positive numbers.
(a) ∑ Re(Zr ) and ∑ Im(Zr ) This function has the property that the image of each
r =1 r =1
point in the complex plane is equidistant from that
point and the origin. Given that |a + bi| = 8 and that
Q.7 If z is one of the imaginary 7th roots of unity, then
find the equation whose roots are (z + z4 + z2) and u
b2 = where u and v are co-primes. Find the value
(z6 + z3 + z5). v
of (u + v).
M a them a ti cs | 6.47

Q.14 Prove that Q.21 C is the complex number. f: C → R is defined


(a) cos x+ nC1 cos 2x + nC2 cos 3x + ….. + nCn cos (n + 1) by f(z)= |z3 – z + 2|. Find the maximum value of f(z)
if |z| = 1.
x n+2
x = 2n. cosn .cos  x
2  2  Q.22 Let a, b, c are distinct integers and w, w2 are the
(b) sin x + nC1 sin 2x + nC2 sin 3x + …….. + nCn sin(n + 1) imaginary cube roots of unity. If minimum value of |a +
1
x n+2 bw + cw2| + |a + bw2 + cw| is n 4 where n ∈ N, then find
x = 2 . cos
n n
.sin   x.
2  2  the value of n.

Q.15 Let f(x) = ax3 + bx2 + cx + d be a cubic polynomial Q.23 If the area of the polygon whose vertices are the
with real coefficients satisfying f(i)= 0 and f(1 + i) = 5. solutions (in the complex plane) of the equation
Find the value of a2 + b2 + c2 + d2.
x7 + x6 + x5 + x4 + x3 + x2 + x + 1 = 0
a b +c
Q.16 Let w1, w2, w3, …. wn be the complex numbers. A can be expressed in the simplest form as , find
the value of (a + b + c + d). d
line L on the complex plane is called a mean line for the
points w1, w2, w3, …. wn if L contains the points (complex
n Q.24 If a and b are positive integer such that
numbers) z1, z2, z3, …. zn such that ∑ (zk − ωk ) = 0 . N = (a + ib)3 – 107 i is a positive integer.
k =1

Now for the complex number w1 = 32 + 170i, w2 = Find N.


– 7 + 64i, w3 = –9 + 200i, ω4 = 1 + 27i and ω5 = – 14 + 43i,
Q.25 If the biquadratic x4 + ax3 + bx2 + cx + d = 0 (a, b, c, d
there is a unique mean line with y-intercept 3. Find the
∈ R) has 4 non real roots, two with sum 3 + 4i and the
slope of the line. other two with product 13 + i. Find the value of ‘b’.

Q.17 A particle start to travel from a point P on the Q.26 Resolve z5 + 1 into linear and quadratic
curve C1: |z – 3 – 4i| = 5, where |z| is maximum. From
factors with real coefficients. Deduce that:
3
P, the particle moves through an angle tan−1 in π π
4 4 sin cos = 1 .
anticlockwise direction on |z – 3 – 4i| = 5 and reaches 10 5
at point Q. From Q, it comes down parallel to imaginary
axis by 2 units and reaches at point R. Find the complex Q.27 If x = 1 + i 3 : y = 1 – i 3 & z = 2,
number corresponding to point R in the Argand plane.
prove that xp + yp = zp for every prime p > 3.
p
32  10  2qπ 2qπ  
Q.18 Evaluate: ∑ (3p − 2)  ∑  sin − icos Q.28 Dividing f(z) by z – i, we get the remainder i and
q 1 
= p 1=  11 11   dividing it by z + i we get the remainder 1 + i. Find the
remainder upon the division of f(z) by z2 + 1.
Q.19 Let a, b, c be distinct complex numbers
Q.29 (a) Let z = a + b be a complex number, where x
a b c
such that = = = k. and y are real numbers. Let A and B be the sets defined
1−b 1−c 1−a by
Find the value of k.
A = {z| |z| ≤ 2} and

Q.20 Let α, β be fixed complex numbers and z is a B = {z| (1 – i)z + (1 + i) z ≥ 4} .


variable complex number such that
Find the area of the region A ∩ B .
2 2
| z − α | + | z − β | =k
Find out the limits for ‘k’ such that the locus of z is a
circle. Find also the centre and radius of the circle.
6 . 4 8 | Complex Number

1 Q.2 The complex numbers whose real and imaginary


(b) For all real numbers x, let the mapping f(x) = , parts are integers and satisfy the relation zz 3 + z3 z =
350
x −i
form a rectangle on the Argand plane, the length of
where i= −1 . If there exist real number a, b, c and d
whose diagonal is-
for which f(a), f(b), f(c) and f(d) form a square on the
complex plane. Find the area of the square. (A) 5 (B) 10 (C) 15 (D) 25

Q.3 Let z1 and z2 be non-zero complex numbers


Q.30
satisfying the equation, z12 – 2z1z2 + 2z22 = 0.
Column I Column II The geometrical nature of the triangle whose vertices
(A) L et ω be a non-real cube root of unity then (p) 4 are the origin and the points representing z1 & z2.
the number of distinct elements in the set
(A) An isosceles right angled triangle
{(1 + ω + ω 2
}
+ ... + ωn )m |m,n∈N
(B) A right angled triangle which is not isosceles

(C) An equilateral triangle


(B) Let 1, ω, w2 be the cube root of unity. The (q) 5
least possible degree of a polynomial with (D) An isosceles triangle which is not right angled
real coefficients having roots

2ω,(2+3ω),(2+3w2),(2–ω-w2), is Q.4 The set of points on the Argand diagram which


π
satisfy both | z | ≤ 4 & Arg z = is:
3
(C) α=6+4i and β=(2 +4i) are two complex (r) 6 (A) A circle and line (B) A radius of a circle
numbers on the complex plane. A complex
number z. A complex number z satisfying (C) A sector of a circle (D) An infinite part line
z−α π
amp   = moves on the major Q.5 If z1 & z2 are two complex numbers & if
 z −β  6
z1 + z 2 π
segment of a circle whose radius is arg = but |z1 + z2| ≠ |z1 – z2| then the figure
z1 − z 2 2
formed by the points represented by 0, z1, z2 & z1 + z2 is:
(s) 7
(A) A parallelogram but not a square rectangle or a
rhombus
(B) A rectangle but not a square
Exercise 2
(C) A rhombus but not a square
Single Correct Choice Type (D) A square

Q.1 The set of points on the complex plane such that z2


Q.6 If z1, z2, z3 are the vertices of the ∆ABC on the
+ z + 1 is real and positive. (where z = x + iy, x, y ∈ R) is:
complex plane & are also the roots of the equation z3 –
(A) Complete real axis only 3az2 + 3bz + x = 0, then the condition for the ∆ABC to
be equilateral triangle is:
(B) Complete real axis or all points on the line 2x + 1 = 0
(A) a2 = β (B) α = b2
(C) Complete real axis or a line segment joining points
(C) a2 = 2β (D) α = 3b2
 1 3  1 3
− , &  − , −  excluding both.
 2 2   2 2  Q.7 Let A, B, C represent the complex numbers z1, z2, z3
   
respectively on the complex plane. If the circumcentre of
(D) Complete real axis or set of points lying inside the the triangle. ABC lies at the origin then the orthocenter
rectangle formed by the lines. is represented by the complex number:

2x + 1 = 0; 2x – 1 = 0; 2y – 3 = 0 & 2y + 3 =0 (A) z1 + z2 – z3 (B) z2 + z3 – z1


(C) z3 + z1 – z2 (D) z1 + z2 + z3
M a them a ti cs | 6.49

Q.8 Which of the following represents a point in an Q.15 If Ar (r = 1, 2, 3, ….., n) are the vertices of a regular
argand’s plane, equidistant from the roots of equation polygon inscribed in a circle of radius R, then
(z + 1)4 = 16z4?
(A1A2)2 + (A1A3)2 + (A1A4)2 + …+ (A1An)2 =
 1  1   2 
(A) (0, 0) (B)  − ,0  (C)  ,0  (D)  0,  nR 2
 3  3   5 (A) (B) 2nR2
2
π
Q.9 The equation of the radical axis of the two circles (C) 4R2 cot (D) (2n – 1) R2
presented by the equations. 2n

|z – 2| =3 and |z – 2 – 3i| = 4 on the complex plane is: Q.16 If the equation z4 + a1z3 + a2z2 + a3z + a4 = 0,
(A) 3y + 1 = 0 (B) 3y – 1 = 0 where a1, a2, a3, a4 are real coefficients different from
zero has a pure imaginary root then the expression
(C) 2y – 1 = 0 (D) None of these a3 aa
+ 1 4 has the value equal to
a1a2 a2a3
Q.10 Number of real solution of the equation, z3 + iz – 1
= 0 is (A) 0 (B) 1 (C) – 2 (D) 2
(A) Zero (B) One (C) Two (D) Three
Q.17 All roots of the equation (1 + z)6 + z6 = 0
Q.11 A point ‘z’ moves on the curve |z – 4 – 3i| = 2 in (A) Lie on a unit circle with centre at the origin
an argand plane. The maximum and minimum values
(B) Lie on a unit circle with centre at (– 1, 0)
of |z| are:
(C) Lie on the vertices of a regular polygon with centre
(A) 2, 1 (B) 6, 5 (C) 4, 3 (D) 7, 3
at the origin
 π (D) Are collinear
Q.12 Let z = 1 – sin α + i cos α where α ∈ 0,  , then
 2
the modulus and the principle value of the argument of Q.18 Number of roots of the equation z10 – z5 – 992 = 0
z are respectively: with real part negative is:
π α π α (A) 3 (B) 4 (C) 5 (D) 6
(A) 2(1 − sin α ),  +  (B) 2(1 − sin α ),  − 
 4 2  4 2
Q.19 z1 and z2 are two distinct points in argand plane.
π α π α
(C) 2(1 + sin α ),  +  (D) 2(1 + sin α ),  −  az1 bz 2
4 2 4 2 If a |z1| = b|z2|, then the point + is a point on
bz 2 az1
the (a, b ∈ R)
Q.13 z1 and z2 are complex numbers. Then
(A) Line segment [– 2, 2] of the real axis

z1 + z 2 z1 + z 2 (B) Line segment [– 2, 2] of the imaginary axis


Equ. + z1 z 2 + − z1 z 2 =
2 2 (C) Unit circle |z| = 1
(D) The line with arg z = tan–1 2
(A) 2 z1 + z 2 (B) 2 z1 − z 2

 2 2   2 2  Q.20 If ω is an imaginary cube root of unity, then the


(C) 2  z1 + z2  (D)  z1 + z2  value of (p + q)3 + (pω + qω2)3 + (pω2 + qω)3 is
   
(A) p3 + q3
Q.14 If α, β be the roots of the equation u – 2u + 2 = 0 2
(B) 3(p3 + q3)
n n
(x + α ) − (x + β)
and if cot θ = x + 1, then is equal to: (C) 3(p3 + q3) – pq (p + q)
α −β
(D) 3(p3 + q3) + pq (p + q)
sinnθ cosnθ sinnθ cosnθ
(A) (B) (C) (D)
n n n
sin θ cos θ cos θ sinn θ
6 . 5 0 | Complex Number

( ) Q.26 For two complex numbers z1 and z2


x
Q.21 The solution set of the equation 1 + i 3 − 2x =
0

(A) Form an A.P. (B) Form a G.P.


( az1 + bz1 )( cz2 + dz2 ) = ( cz1 + dz1 )( az2 + bz2 )
If (a, b, c, d ∈ R):
(C) Form an H.P. (D) Is a empty set
a c a b
(A) = (B) =
b d d c
Multiple Correct Choice Type
(C) |z1| = |z2| (D) arg z1 = arg z2
Q.22 In the quadratic equation x2 + (p + iq) x + 3i = 0,
p and q are real. If the sum of the squares of the roots
Q.27 Let z1, z2 be two complex numbers represented by
is 8 then
point on the circle |z1| = 1 and |z2| = 2 respectively, then:
(A) p = 3, q = – 1 (B) p = 3, q = 1
(A) Max |2z1 + z2| = 4 (B) Min |z1 – z2| = 1
(C) p = –3, q = – 1 (D) p = – 3, q = 1
1
(C) z 2 + ≤3 (D) None of these
Q.23 Let z1 and z2 be complex numbers such that z1 ≠ z2 z1
and |z1| – |z2|. If z1 has positive real part and z2 has negative
z + z2 Q.28 If α, β any two complex numbers such that
imaginary part, then 1 may be
z1 − z 2 α −β
= 1 , then
1 − αβ
(A) Zero (B) Real & positive
(C) Real and negative (D) Purely imaginary (A) |α| = 1 (B) |β| = 1
(C) α = e iθ , θ ∈ R (D) β = eiθ , θ ∈ R
Q.24 Given a, b, x, y ∈ R then which of the following
statement(s) hold good? Q.29 On the argand plane, let α = – 2 + 3z, β = – 2 – 3z
(A) (a + ib) (x + iy) = a – ib ⇒ x + y = 1
–1 2 2 and |z| = 1. Then the correct statement is:

(B) (1 – ix) (1 + ix)–1 = a – ib ⇒ a2 + b2 = 1 (A) α moves on the circle, centre at (– 2, 0) and radius 3

(C) (a + ib) (a – ib)–1 = x – iy ⇒ |x + iy| = 1 (B) α and β describe the same locus

(D) (y – ix) (a + ib)–1 = y + ix ⇒ |a – ib| = 1 (C) α and β move on different circles


(D) α – β moves on a circle concentric with |z| = 1
Q.25 If z = x = iy = r (cos θ + i sin θ) then the values of
Q.30 The value of in + i–n, for i= −1 and n ∈ I
z is equal to:
is:
(A) ±
1
2
( )
r + x + i r − x for y ≥ 0
(A)
2n
+
(1 + i)2n
(B)
(1 + i)2n
+
(1 − i)2n
(1 − i)2n 2n 2n 2n
(B) ±
1
2
( )
r + x − i r − x for y ≥ 0
(1 + i)2n 2n 2n 2n
(C) + (D) +
2n (1 − i)2n (1 + i)2n (1 − i)2n
(C) ±
1
2
( )
r + x + i r − x for y ≤ 0

Q.31 A complex number z satisfying the equation,


(D) ±
1
2
( )
r + x − i r − x for y ≤ 0
log14 (13 | z 2 − 4i |) + log196
1
0
=
(13 + | z 2 + 4i |)2
(A) Can be purely real
(B) Can be purely imaginary
(C) Must be imaginary
(D) Must be real or purely imaginary
M a them a ti cs | 6.51

Q.32 Let S be the set of real values of x satisfying the z1 z1′ 1


inequality
(A) z 2 z′2 1 = 0
| x + 1 + 2i | −2
1 − log2 ≥ 0 , then S contains: z3 z3′ 1
2 −1
(B) The two triangles are congruent
(A) [– 3, – 1) (B) (– 1, 1]
(C) The two triangles are similar
(C) [– 2, 2) (D) [1, 2]
(D) The two triangles have the same area.
Q.33 If x = cos α; y = cos β ; z = cos γ ; Where α, β, γ ∈
Q.38 If ‘z’ be any complex number in a plane (|z| ≠ 0)
R, then
then the complex number z for which the multiplication
inverse is equal to the additive inverse is:
(A) ∑× = Π × ⇒ cos (α – β) = 1
(A) 0 + i (B) 0 – i (C) 1 – i (D) 1 + i
x−y α −β
(B) Π 8Π cos
=
z 2 1 − ix
Q.39 Given z =a + bi = ; a, b, x ∈ R, then which of
1 + ix
x+y
(C) Π is real the following holds good?
z
π
(A) – < arg z ≤ 0 (B)– π < arg z ≤ 0
(D) ∑= Im(x) sin ( ∑ α )
(Re)x cos ( ∑ α ) , ∑ = 2
(C) |z| = 1 (D) arg z = π ; |z| = 1
π
Q.34 If xr = cos   for 1 ≤ r ≤ n; r, n ∈ N then:
 2r 
 n   n  Previous Years’ Questions
(A) Lim Re  ∏ xr  = −1 (B) Lim Re  ∏ xr  = 0
n→ ∞ r = 1  n→ ∞ r = 1 
    Q.1 If z is any complex number satisfying |z – 3 – 2i| ≤ 2,
then the minimum value of |2z – 6 + 5i| is ……  (2011)
 n   n 
(C) Lim Im  ∏ xr  = 1 (D) nLim Im  ∏ xr  = 0 2π
n→ ∞ r = 1  →∞ r = 1  i
    Q.2 Let ω =e and a, b, c, x, y, z be non-zero complex
3

number such that a + b + c = x, a + bω + cω2 = y,


Q.35 If 1, z1, z2, z3 …… zn–1 be the nth roots of unity and a + bω2 + cω = z. Then, the value of
ω be a non real complex cube root of unity then the
n −1 | x |2 + | y |2 + | z |2
is …… (2011)
product ∏ ( ω − zr ) can be equal to: | a |2 + | b |2 + | c |2
r =1

(A) 0 (B) 1 (C) – 1 (D) 2


Paragraph (for Q.3, 4, 5)

Q.36 Identify the correct statements(s). Read the following passage and answer the following
questions.
(A) No non zero complex number z satisfies the
equation, z = −4z Let A, B, C be three sets of complex number as defined
below
(B) z = z implies that z is purely real
(C) z = −z implies that z is purely imaginary A = {z: |Im z ≥ 1|}

(D) If z1, z2 are the roots of the quadratic equation B = {z: |z – 2 – i| = 3}


az2 + bz + c = 0 such that Im (z1z2) ≠ 0 then a, b, c must
C = {z: Re ((1 – i)z) + 2 ) (2008)
be real numbers

Q.37 If the complex numbers z1, z2, z3 & z1′, z2′ and z3′ Q.3 The number of elements in the set A ∩ B ∩ C is
are representing the vertices of two triangles such that (A) 0 (B) 1 (C) 2 (D) ∞
z3 = (1 – z0) z1 + z0z2 and z3 ‘ = (1 – z0) z1’ + z0z2’ where
z0 is also a complex number then:
6 . 5 2 | Complex Number

Q.4 Let z be any point in A ∩ B ∩ C . The |z + 1– i|2 + | Q.12 If i= −1 , then


z – 5 – i|2 lies between
334 365
 1 i 3  1 i 3
(A) 25 and 29 (B) 30 and 34 4 + 5 − +  + 3 − + 
 2 2   2 2 
(C) 35 and 39 (D) 40 and 44  
is equal to (1999)
Q.5 Let z be any point in A ∩ B ∩ C and let w any point
(A) l – i 3 (B) – 1 + i 3 (C) i 3 (D) − i 3
satisfying |w – 2 – i|< 3. Then |z| – |w| + 3 lies between
(A) – 6 and 3 (B) – 3 and 6 Q.13 If arg (z) < 0, then arg (– z) – arg (z) equal(2000)
(C) – 6 and 6 (D) – 3 and 9 π π
(A) π (B) – π (C) − (D)
2 2
5 5
 3 i  3 i
Q.6 If z =  +  + −  , then (1982) Q.14 If z1 = a + ib and z2 = c + id are complex numbers
 2 2  2 2
    such that |z1| = |z2| =1 and Re ( z1 z2 ) = 0 , then the pair
(A) Re (z) = 0 of complex numbers w1 = a + ic and w2 = b + id satisfied
(B) Im (z) = 0 by (1985)
(A) |w1| = 1 (B) |w2| = 1
(C) Re (z) > 0, Im (z) > 0
(D) Re (z) > 0, Im (z) < 0 (C) Re ( w1 w2 ) = 0 (D) None of these

Q.7 The inequality |z – 4| < |z – 2| represents the region Q.15 Let z1 and z2 be two distinct complex numbers
given by  (1982) and let z = (1 – t) z1 + tz2 for some real number t with 0
< t < 1. If arg (w) denotes the principal argument of a
(A) Re (z) ≥ 0 (B) Re (z) < 0 non-zero complex number w, then (2010)
(C) Re (z) > 0 (D) None of these (A) |z – z1| + |z – z2| = |z1 – z2|

Q.8 If a, b, c and u, v, w are the complex numbers (B) arg (z – z1) = arg (z – z2)
representing the vertices of two triangles such that z − z1 z − z1
c= (1–r) a+rb and w = (1 – r) u + rv, where r is a complex (C) =0
z 2 − z1 z2 − z1
number, then the two triangles  (1985)
(A) Have the same area (B) Are similar (D) arg (z – z1) = arg (z2 – z1)

(C) Are congruent (D) None of these


Q.16 Match the statements in column I with those in
6 column II. (2010)
 2πk 2πk 
Q.9 The value of ∑  sin 7
− icos
7 
 is (1987)
[Note: Here z takes values in the complex plane and Im
k =1 
z and Re z denote, respectively, the imaginary part and
(A) – 1 (B) 0 (C) – i (D) i the real part of z.]

Q.10 Let z and w be two complex numbers such that Column I Column II
|z| ≤ 1, |w| ≤ 1 and |z + iw| = |z – iw | = 2, then z equals (A) The set of points z (p) An ellipse with
 (1995) satisfying |z-i|z|| = |z + i|z|| 4
(A) 1 or i (B) i or – i (C) 1 or – 1 (D) i or –1 eccentricity
Is contained in or equal to 5

Q.11 For positive integers n1, n2 the value of expression (B) The set of points z (q) The set of points z
n n n2 n
satisfying |z+4|+|z-4|=10 is satisfying Im = z = 0
(1 + i) 1 + (1 + i3 ) 1 + (1 + i5 ) + (1 + i7 ) 2 ,Here i= −1 contained in or equal to
is a real number, if and only if (1996) (C) If |w|=2, then the set (r) The set of points z
(A) n1 = n2 + 1 (B) n1 = n2 – 1 1 satisfying | Im z | ≤ 1
of points z= w − is
w
(C) n1 = n2 (D) n1 > 0, n2 > 0 contained in or equal to
M a them a ti cs | 6.53

Then the number of distinct matrices in the set S is


(D) If |w| = 1, then the set (s) The set of points z
 (2011)
1 satisfying | Re z | ≤ 2
of points z= w + is (A) 2 (B) 6 (C) 4 (D) 8
w
contained in or equal to
The set of points z Q.22 Let z be a complex number such that the imaginary
satisfying | z | ≤ 3 part of z is nonzero and a = z 2 + z + 1 is real. Then a
cannot take the value  (2012)
1 1 3
2π 2π (A) -1 (B) (C) (D)
Q.17 Let ω be the complex number cos + i sin . 3 2 4
3 3
Then the number of distinct complex number z
1
Q.23 Let complex numbers α and lies on circle
satisfying  (2010) α
( x − x0 ) + ( y − y 0 ) r 2 and ( x − x0 ) + ( y − y0 ) =
2 2 2 2
= 4r 2 ,
2
z +1 ω ω
2
respectively. If z=
0 x0 + iy 0 satisfies the equation
ω z+ω 1 = 0 is equal to
ω2 1 z+ω |2 r 2 + 2 , then | α |=
2| z 0 = (2013)

1 1 1 1
(A) (B) (C) (D)
Q.18 Let z1 and z 2 be two distinct complex numbers 2 2 7 3
and let z = (1 − t ) z1 + t z2 for some real number t with
3 +i
0 < t < 1. If Arg (w) denotes the principal argument of a
nonzero complex number w, then  (2010)
Q.24 Let ω =
2
and P =ωn : n = {
1,2,3,..... . Further }
 1  −1 
z ∈ C : Rez >  and H2 =
H1 = z ∈ C : Rez <  ,
(A) | z − z1 | + | z − z 2 |= | z1 − z 2 |  2  2 
where C is the set of all complex numbers. If
(B) Arg ( z − z1=
) Arg ( z − z2 )
z1 ∈ P ∩ H1 , z 2 ∈ P ∩ H2 and O represents the origin,
z − z1 z − z1
(C) =0
z 2 − z1 z2 − z1 then ∠ z1 Oz 2 = (2013)

π π 2π 5π
(D) Arg ( z − z=
1) Arg ( z 2 − z1 ) (A) (B) (C) (D)
2 6 3 6

Q.19 if z is any complex number satisfying z − 3 − 2i ≤ 2


Q.25 Let ω be a complex cube root of unity with ω ≠ 1
, then the minimum value of 2z − 6 − 5i is  (2011)
and P = pij  be a n × n matrix with pij = ωi+ j . Then
 
Q.20 Let ω =ei π /3 , and a, b, c, x, y, z be non-zero P2 ≠ 0 , when n = (2013)
complex numbers such that (A) 57 (B) 55 (C) 58 (D) 56
a+b+c=x
Paragraph (for Q.26 and Q.27)  (2013)
a + b ω + c ω2 = y
Let S =S1 ∩ S2 ∩ S3 , where
a + b ω2 + c ω = z
  z − 1 + 3 i 
| x |2 + | y |2 + | z |2 S1 ={z ∈ C : | z | < 4} , S2 =
z ∈ C : Im   > 0
Then the value of is  (2011)   1 − 3 i  
| a |2 + | b |2 + | c |2
and S3 ={z ∈C : Re Z > 0}
Q.21 Let ω ≠ 1 be a cube root of unity and S be the set
Q.26 Area of S =
of all non-singular matrices of the form
 1 a b 10 π 20 π 16 π 32 π
  2
(A) (B) (C) (D)
 ω 1 c  where each if a, b and c is either ω or ω . 3 3 3 3
ω2 ω 1 
 
6 . 5 4 | Complex Number

Q.27 Min 1 − 3i − z = kπ kπ


z∈S Q.29 For any integer k, =
let αk cos   + i sin  ,
 7   7 
2− 3 2+ 3 3− 3 3+ 3 where i= −1 . The value of the expression.
(A) (B) (C) (D) 12
2 2 2 2
∑ | αk +1 − αk |
k =1
is  (2015)
Q.28 Match the following: (2014) 3
∑ | α 4 k −1 − α 4k −2 |
k =1
Column I Column II
(i) The number of polynomials f(x) with (p) 8 −1 + 3 i
non-negative integer coefficients of
Q.30 Let z = , where i= −1 , and r, s ∈ {1,2,3} .
2  
degree ≤ 2 , satisfying f (0) = 0 and  −z r z 2s 
1
Let P = 
( )  and I be the identity matrix of order
∫ f ( x )dx = 1 , is  z 2s
 z r 

0
2. Then the total number of ordered pairs (r, s) for which
(ii) The number of points in the (q) 2
P2 = −I is  (2016)
interval  − 13, 13  at which
 
( )
f ( x ) sin x2 + cos x2 attains its
= ( ) Q.31 Let a, b ∈  and a2 + b2 ≠ 0.
maximum value, is  1 
Suppose S =z ∈  : z = t ∈  , t ≠ 0  , where
(r) 4  a + ibt' 
2
3x2
(iii) ∫ dx equals i= −1 . If z = x + iy and z ∈ S, then (x, y) lies on
−2 (1 + e )x
 (2016)

(s) 0 1  1 
 1/2 (A) The circle with radius and centre  , 0  for
1 + x   2a  2a 
 ∫ cos 2x.log   dx  a > 0, b ≠ 0
  1 − x  
(iv)  −1/2
equals 1  1 
 1/2 1 + x   (B) The circle with radius − and centre  − , 0  for
 ∫ cos 2x.log   dx  a < 0, b ≠ 0 2a  2a 
  1 − x  
 0
(C) The x-axis for a ≠ 0, b = 0
Codes:
(D) The y-axis for a = 0, b ≠ 0
(i) (ii) (iii) (iv)
(A) r q s p
(B) q r s p
(C) r q p s
(D) q r p s
M a them a ti cs | 6.55

PlancEssential Questions
JEE Main/Boards JEE Advanced/Boards

Exercise 1 Exercise 1
Q.6 Q.9 Q.15 Q.7 Q.11 Q.13

Q.18 Q.22 Q.24 Q.16 Q.18 Q.25

Q.28 Q.31 Q.34 Q.29 Q.30

Exercise 2 Exercise 2
Q.2 Q.8 Q.10 Q.2 Q.6 Q.9

Q.13 Q.16 Q.18 Q.15 Q.19 Q.22

Q. 25 Q.27 Q.31
Previous Years’ Questions
Q. 33 Q. 36 Q. 39
Q.2 Q.4 Q.7

Q.10 Q. 13 Q.15 Previous Years’ Questions


Q.2 Q.4 Q. 8

Q.11 Q.14 Q.15

Answer Key

JEE Main/Boards
Exercise 1

3 i 5 14
Q.1 z = 0, i, ± – Q.2 – i Q.3
= x = ,y
2 2 13 13
  3π   3π  
Q.4 x = 3, y – 1 Q.6 2 cos   + isin    Q.7 ± (1 – 3i)
  4   4 
Q.8 n = 4 Q.10 – 1 Q.11 – 1, 1 – 2ω, 1 – 2ω2

Q.13 z1 =(1 − 3) + i ; z 2 = −i 3 ; z3 =(1 + 3) − i

 α − K 2β  α + β −K (n − 1)n 2
Q.14 Centre   , radius Q.19 [n + 3n + 4]
2
 1 − K  1 − K2 4
6 . 5 6 | Complex Number

Q.20 5 Q.24 ω, w2

Q.31 3 ≤ | z | ≤ 7 Q.33 Interior of circle x2 + y2 = 25

Exercise 2

Single Correct Choice Type

Q.1 C Q.2 D Q.3 B Q.4 C Q.5 D Q.6 A Q.7 B

Q.8 C Q.9 D Q.10 D Q.11 A Q.12 A Q.13 D Q.14 D

Q.15 B Q.16 C Q.17 D Q.18 C Q.19 A Q.20 C

Previous Years’ Questions

Q.1 D Q.2 A Q.3 B Q.4 B Q.5 C Q.6 D Q.7 B

Q.8 D Q.9 D Q.10 B Q.11 D Q.12 A Q.13 B Q.14 B

Q.15 D Q.16 A Q.17 B Q.18 B Q.19 B Q.20 C

JEE Advanced/Boards
Exercise 1
7
Q.2 12 Q.4 7 Q.5 10 Q.6 (a) − , (b) zero Q.7 x2 + x + 2 = 0 Q.8 4
2
Q.10 41 Q.13 259 Q.15 26 Q.16 163 Q.17 (3 + 7i) Q.18 48 (1 – i)
2
Q.19 – ω or –w2 Q.20 k > α − β

Q.21 If (z) is maximum when z = ω, when ω is the cube root unity v and If (z) = 13

Q.22 144 Q.23 8 Q.24 198 Q.25 51

iz 1
Q.26 (z + 1) (z2 – 2z cos 36° + 1) (z2 – 2z cos 108° + 1) Q.28 + + i
2 2
1
Q.29 (a) π – 2; (b) Q.30 A → s; B → q; C → p
2

Exercise 2

Single Correct Choice Type

Q.1 C Q.2 B Q.3 A Q.4 B Q.5 C Q.6 A Q.7 D

Q.8 C Q.9 B Q.10 A Q.11 D Q.12 A Q.13 D Q.14 A

Q.15 B Q.16 B Q.17 D Q.18 A Q.19 A Q.20 B Q.21 A


M a them a ti cs | 6.57

Multiple Correct Choice Type

Q.22 B, C Q.23 A, D Q.24 A, B, C, D Q.25 A, D Q.26 A, D Q.27 A, B, C Q.28 A, B, C, D

Q.29 A, B, D Q.30 B, D Q.31 A, B, D Q.32 A, B Q.33 D Q.34 A, D Q.35 A, B, D

Q.36 A, B, C Q.37 A, C Q.38 A, B Q.39 A, B, C, D

Previous Years’ Questions

Q.1 5 Q.2 3 Q.3 B Q.4 C Q.5 D Q.6 B Q.7 D

Q.8 B Q.9 D Q.10 C Q.11 D Q.12 C Q.13 A Q.14 A, B, C

Q.15 A, C, D Q.16 A → q, r; B → p; C → p, s, t; D → q, r, s, t Q.17 3 Q.18 A, C, D Q.19 5

Q.20 3 Q.21 A Q.22 D Q.23 C Q.24 C, D Q.25 B, C, D Q.26 B

Q.27 C Q.28 C Q.29 4 Q.30 1 Q.31 A, C, D

Solutions

JEE Main/Boards Sol 3: (x + iy)(2–3i) = 4 + i


⇒(2x + 3y) + i(2y– 3x) = 4 + i
Exercise 1
⇒2x + 3y = 4 and 2y – 3x = 1

Sol 1: z = i(z2) ⇒ Let x = a + ib 5 14


⇒x = and y =
13 13
⇒ a – ib = i(a2 – b2 + 2abi)
⇒ a = –2ab and –b = a2 – b2 (1 + i)x – 2i (2 – 3i)y + i
Sol 4: + i
=
a(1 + 2b) = 0 and a = b – b
2 2 3+i 3–i
1 [x + i(x – 2)][3 – i] + [3 + i][2y + i(1 – 3y)]
a = 0 or b = – =
2 10
if a = 0 ⇒ b = 0, 1 [3x + x – 2 + i(3x – 6 – x)] + [6y + 3y – 1 + i(2y + 3 – 9y)]
=
1 3 10
if b = – ,a=±
2 2 4 x – 2 + i(2x – 6) + (9y – 1) + i(–7y + 3)
=
3 i 10
Complex numbers are z = 0, i, ± −
2 2 4x + 9y – 3 + i(2x – 7y – 3)
= =i
10
1 + 3i2 + 2i −2 + 2i 1 − i
Sol 2: = =
1 + 3i2 – 2i −2 − 2i 1 + i 2x – 7y – 3 = 10 and 4x + 9y – 3 = 0
⇒x = 3 and y = –1
1 + i2 − 2i
=  1 − i  1 − i  =
  
= −i
 1 + i  1 − i  1 − i2
6 . 5 8 | Complex Number

Sol 5: x = a + b Solving (i) and (ii), we get


y = αa + bβ a = –1 & b = +3
z = aβ + bα a = 1 & b = –3
α and β are complex cube roots of unity So square root
⇒ αβ = 1, α2 = β, β2 = α = (–1 + 3i) and (1 – 3i)
(as α = ω, β = ω2, α2 = β)  ….. (i) n
1 + i
xyz = (a + b)(αa + bβ) (aβ + bα) Sol 8:   =1
1 −i
= (αa2 + αab + βab + b2β) (aβ + bα) (1 + i)2n
=1
= αa3β + αa2bβ + α2bβ2 + ab2β2 [(1 + i)(1 − i)]n
n
+ α2a2b + α2ab2 + ab2aβ + b3αβ (1 + i)2 
(1 + i)2n n
 = 1 ⇒ [2i] = 1
=1⇒ 
= aβ(a3 + b3 + a2b + ab2) + α2(a2b + b2a) 2n 2n 2n

+ β2 (a2b + b2a) ⇒in = 1 ⇒ n = 4, 8, 12

from eqn. (i) Minimum value of n is 4

= a3 + b3 +a2b+ab2+ (a2b + b2a) (α2 + b2)


z − 5i
Sol 9: =1
= a3 + b3 + a2b + ab2 + (a2b + b2a) (–1) z + 5i

= a3 + b3 + a2b + ab2 – a2b – ab2 z = x + iy


⇒|x + i(y – 5)| = |x + i(y+5)|
= a3 + b3 hence proved.
⇒x2 + (y – 5)2 = x2 + (y + 5)2
2 2
1 + 7i (2 + i) (1 + 7i)(2 + i) ⇒y = 0
Sol 6: × =
(2 − i) 2 2
(2 + i) 25
i.e. complex part of z is zero.
(1 + 7i)(3 + 4i) –25 + 25i z is pure real i.e. it lies on x axis.
= =
25 25
z = –1 + i Sol 10: z = 1 + itana
z(θ) = |z|e iq
|z| = 1 + tan2 =
α sec2 =
α sec α
|z| = 1 + 1 =2  π 3π 
For α ∈  , 
1 2 2 
tanθ = = –1
–1 secα < 0 ⇒ |seca| = –seca
3π |z| = –seca
⇒ θ = tan–1(–1) =
4
3π |z|cosα = –1
i   3π   3π  
⇒ z(θ) = 2e 4 = 2 cos   + isin   
  4   4  Sol 11: (x – 1)3 = –8

Sol 7: –8 – 6i= a + ib x – 1 = (–8)1/3

–8 – 6i = a2 – b2 + i(2ab) x – 1 = (81/3)(–1)1/3 ⇒ 1 – x = (8)1/3(1)1/3

⇒a2 – b2 = –8  …… (i) (–x + 1) = 2, 2ω, 2ω2

2ab = – 6 x = –1, 1 – 2ω, 1 – 2ω2

⇒ab = – 3  …… (ii)
M a them a ti cs | 6.59

Sol 12: |z| < 4  2α − 2β k 2   2α − 2β k 2 


⇒ x2 + y2 + x  1 2 1  +y  2 2

|3 + i(z – 4)| < 9(To prove)  k 2 − 1 
 k − 1 
We know that |z1 + z2| ≤ |z1| + |z2|  k 2β2 + k 2β2 − α2 − α2 
+  1 2 1 2
 =0
|iz + (3 – 4i)| ≤ |z| + |3 – i4|  2
k −1 

|z| < 4 Eqn. of circle

⇒|iz + (3 – 4i)| < 4 + 5  β k2 − α β k2 − α 


with centre as  1 1
, 2 2

 k 2 − 1 k 2 − 1 
⇒|iz + (3 – 4i)| < 9 Hence proved
 α − k 2β 
or  2

Sol 13: 2 + i 3 is vertex of square inscribed in  1 − k 
|z – 1| = 2
(α1 + β1 − k)2 + (α2 + β2 )2 α +β −k
and radius = =
1−k 2
1 − k2

n
1
Sol 15: |z| <
3
and ∑ ar zr = 1  … (i)
r =1

⇒ |a1z1 + a2z2 + a3z3 …… anzn| = 1

|z1 + z2 + z3+ …..zn| ≤ |z1| + |z2| + …..|zn| … (ii)

Here one vertex is A (2, 3) and equation of circle is ⇒ 1 ≤ |a1z| + |a2z2| + |a3z3| …..|anzn|
(x − 1)2 + y 2 =
4
⇒ |a1||z| + |a2||z|2 + |a3||z|3 …..|an||z|n ≥ 1
Radius of the circle is 2. Hence side of the square will
1
be 2 2 . ⇒ |z| + |z|2 …….|z|n ≥
2
Points that lie on the circle and are at a distance 2 2 ⇒ (Limiting case n → ∞)
from A are B (1 − 3,1) and D (1 + 3, −1) . z 1 1
≥ ⇒ z ≥ − (2)
The point C will be the other end of diameter of A. 1− z 2 3
Hence C (0, − 3) . From (i) and (ii), we can say that there is no ‘z’ satisfying
both conditions.
Hence the four vertices are

2 + i 3 , 1 − 3 + i , − 3i and 1 + 3 − i Sol 16: zp+q ‒ zp ‒ zq + 1 = 0


zp(zq ‒ 1).(zq ‒ 1) = 0
z−α (zp ‒ 1).(zq ‒ 1) = 0
Sol 14: =k
z −β zp = 1 or zq = 1
⇒|z – α|2 = k2|z – β| If α is roots, then αo, α, α2,………….. αn-1 also roots of
zp = 1 or zq = 1
α = α1 + iα2, β = b1 + iβ2 Sum of roots = 1 + α + α2 +…………………..+ αn-1 = 0
⇒ (x – α1)2 + (y – α2)2 = k2[(x – B1)2 + (y – β2)2]
Sol 17: z, iz, z + iz
⇒ x2 + α12 + y2 + α22 – 2xα1 – 2yα2
(x, y) (–y, x) (x – y, y + x)
= k2x2 + k2y2 + k2b12 + k2b22 – 2xb1k2 – 2yb2k2
x y 1
1 1
⇒ x (k –1) + y (k –1)+x(2α1 – 2b1k )
2 2 2 2 2
∆= −y x 1 = |–x2 – y2|
2 2
x−y x+y 1
+y(2α2–2b2k2)+k2b12+k2b22–α12–α22=0
6 . 6 0 | Complex Number

1 2 1 2 = (x – 3)[x3 + 2x2 – 6x + 5] + 27
⇒ |D| = (x + y2) = z
2 2
= (x – 3)[(x – 3)(x2 + 5x + 9) + 32] + 27

Sol 18: iz3 + i + z2 – z = 0 = (x – 3)2[x2 + 5x + 9] + 32(x – 3) + 27

iz(z2 + i) + i(z2 + i) = 0 = (x – 3)2[x2 – 3x + 8x – 24 + 33]+32(x – 3)+27

(iz + 1) (z2 + i) = 0 = (x – 3)3 (x + 8) + 33(x – 3)2 + 32(x – 3) + 27

Either z = i or z2 = i = w3(x + 8) + 33(w2) + 32w + 27

If z = I ⇒ |z| = |i| = 1 = w + 1 + 32w + 33w2 + 27

If z 2 =⇒
i | z |2 =1 ⇒ | z |=1 = w + w2 + 32w + 32w2 + 38

Hence, |z| = 1 = –1 – 32 + 38 = 5

Sol 19: 1(2 – w) (2 – w2) + 2(3 – w) (3 – w2) + …… (n – 1) Sol 21: z2 – z1 = (z3 – z1)e–iπ/3
(n – w) (n – w2)
z3 – z2 = (z1 – z2) e–iπ/3
Tn = n(n + 1 – w) (n + 1 – w ) 2
z3 − z1
z 2 − z1
=
= (n + n – nw) (n + 1 – w )
2 2 z3 − z 2 z1 − z 2

= n3 + n2 – n2w2 + n3 + n – nw2 – n2w – nw + nw3 2 2 2


⇒ z1 + z 2 + z3 = z1z2 + z2z3 + z3z1
= n3 + n2(2 – w2 – w) + n(1 – w – w2 + 1)  z1 + z 2 + z3 
2
2
  = z 0
= n3+ 3n2 + 3n  3 

Sn = STn = Σ(n3 + 3n2 + 3n) z12 + z 22 + z32 + 2(z12 + z 22 + z32 )


⇒ = z 20
9
= Sn3 + 3Sn2 + 3Sn
⇒ z12 + z 22 + z32 2
= 3 z0
2
 n(n + 1)  3n(n + 1)(2n + 1) 3n(n + 1)
=   + +
 2  6 2 Sol 22: z1 z2 z3 of an isosceles ∆L at z2
z1
n(n + 1)  n(n + 1) 
=  + (2n + 1) + 3
2  2 
n(n + 1) 2
= [n + n + 6 + 4n + 2]
4
n(n + 1) 2
Sn = STn = [n + 5n + 8]
4
z2
(n − 1)n 2 z3
Sn–1 = [n + 1 − 2n + 5n − 5 + 8]
4 Considering z2 at origin
(n − 1)n 2
= [n + 3n + 4] z1-z2
4

1 1
Sol 20: x = (5 − i 3) =(6 − 1 − i 3) =
3+w
2 2
x4 – x3 – 12x2 + 23x + 12
O z3-z2
= x3(x – 3) + 2x2(x – 3) – 6x(x – 3) (z1 – z2) = (z3 – z2)e iπ/2

+ 5(x – 3) + 27 z1 – z2 = (z3 – z2)i


M a them a ti cs | 6.61

(z1 – z2)2 = –(z3 – z1)2


2 α + α2 − β2 α − α2 − β2
⇒ z1 + z3 + 2z2 = 2z1z3 + 2z1z2
2 2 2

   α + α2 − β2 
⇒ z12 + 2z22 + z32 = 2z2(z1 + z3) =  α + α2 + β2   α + α2 − β2  +  
    

A2 B2 H2  2 2 
 α − α − β  + 2|α – a + b |
2 2 2
Sol 23: + .......... x +  Let x = p + iq
=
x −a x −b x −H  
2  2 2
2
A2 A (p− a) − iq = 2  α + α2 − β2  + 2 β
(p +  ) + iq = ∑ =∑  
(p − a) + iq (p − a)2 + q2
2 2
= 2 α + 2 β + 2 α2 − β2
A2 (p− a) A2q
= ∑ (p − a)2 + q2 − i∑ (p − a)2 + q2 2
= 2 α +2 β +2 α +β α −β
2

Equating the imaginary terms we get = αα + ββ + αβ + αβ + αα + ββ –


2
A q αβ − βα + 2 α + β α − β
q+∑ 0
=
(p − a)2 + q2 2 2
= α +β + α −β +2 α +β α −β
 A2 
⇒ q1 + ∑  =0 2
 (p − a)2 + q2  =  α + β + α − β 

A2 T = α + β + α − β Hence proved
⇒ q = 0 or ∑ (p − a)2 + q2 = −1

Sol 27: z1 = 10 + 6i; z2 = 4 + 6i


Here A, p, a and q are all real hence q = 0 is the only
solution. z − z1 x − 10 + (y − 6)i  (x − 4) − i(y − 6) 
=  
⇒ x cannot have imaginary roots. z − z2 x − 4 + (y − 6)i  (x − 4) − i(y − 6) 

(x − 10)(x − 4) + (y − 6)2 + i (y − 6)(x − 4) + (x − 10)(6 − y)


Sol 24: z3 + 2z2 + 2z + 1 = 0 =
(x − 4)2 + (y − 6)2
⇒ z = –1, ω , ω2
z − z1 π
z1985 + z100 + 1 = 0 arg =
z − z2 4
⇒ z = ω , ω2
⇒ (y – 6)(x – 4)+(x – 10)(6 – y)
Common roots are ω , ω2
= (x – 10) (x – 4) + (y – 6)2
Sol 25: Let f(x,y) = (x+y) – x –y n n n
⇒ (y–6)(x – 4 – x + 10)=x2 – 14x + 40 + (y–6)2
xy(x+y) (x +xy+y ) = (x-0)(y-0)(x+y)(x-wy)(x-w y)
2 2 2
⇒ 6(y – 6) = x2 – 14x + 40 + y2 + 36 – 12y
f(x=0)=0 and f(y=0)=0
⇒ x2 + y2 – 18y – 14x + 112 = 0
f(y=–x)=(x–x)n–(x)n–(–x)n=0
f(x=wy)=(wy+y)n–wnyn–yn ; = yn[w2n + wn +1] = 0 ⇒ (x – 7)2 + (y – 9)2 = 18

Similarly f(x=w2y)=0 ⇒ |z – 7 – 9i| = 3 2


∴ f(x) is divisible by xy(x+y)(x2+xy+y2)
( z − w )( z − w )
2
Sol 28: z − w =

Sol 26: T = α + α2 − β2 + α − α2 − β2 2 2
= z + w − z w − zw + 2 z w − 2 z w

( )
2 2 2
T2 = α + α2 − β2 + α − α2 − β2 + = z −w − z w − zw + 2 z w

Let z = r1cosθ1; w = r2cosθ2


6 . 6 2 | Complex Number

z1 – z2 = k1(z3 – z2)
(z − w)
2
= − 2r1r1 cos ( θ1 − θ2 ) + 2r1r2
⇒ z1, z2 and z3 are collinear (by vector)
  θ − θ 
( )
2
=z − w + 2r1r1  2sin2  1 2  
 
  2  Sol 31: |z – 4 + 3i| ≤ 2
2
 θ − θ2 
( )
2
=z − w + 4r1r1  sin 1 
 2 

We know, sin θ ≤ θ and r1, r2, ≤ 1


2
 θ − θ2 
( )
2
≤ z −w + 4 ×  1 
 2 

( ) + ( θ1 − θ2 )2
2
≤ z −w

( ) + (arg z − arg w )
2 2
≤ z − w
The shaded area show z
Min and max value = Distance of centre from origin ±
A B radius
Sol 29: + = 1
B A
= 5 ± 2 = 3, 7
A2 + B2 = AB
⇒ 3 ≤ |z| ≤ 7
A B
= 1−
B A
R z1 − z 2
A A −B Sol 32: <1
= 1 − z1 z 2
B A
2 2
lA-Bl z1 − z 2 < 1 − z1 z 2
lRl
A2
= A −B (z1 – z2) (z1 − z2 ) < (1 − z1 z 2 )(1 − z1 z2 )
B
lAl ⇒ |z1|2 + |z2|2 – z1 z2 – z1 Z2
2 O A
B
= B−A < – z1 z2 – z2 z1 + |z1|2|z2|2 + 1
A
⇒ |z1|2 – 1 < (|z1|2 – 1) |z2|2
2 2
A B
A −B = B − A ⇒ = 2
z1 − 1
B A
|z2| <
2
2
⇒|A | = |B |3 3 z1 − 1

⇒|A| = |B| = |A – B| |z2|2 < 1 ⇒ |z2| < 1

i.e. all sides are equal it forms an equilateral D


 z 2 − z + 1
Sol 30: a + b + c = 0 ⇒ b = –a – c Sol 33: log  <2
3  z +2 
az1 + bz2 + cz3 = 0  
2
z − z +1
az1 + cz3 – (a + c)z2 = 0 <3
z +2
az1 + cz3
z2 = ⇒ |z|2 – |z| + 1 < 3|z| + 6
a+c
⇒a(z1 – z2) + c(z3 – z2) = 0 ⇒ |z2| – 4|z| – 5 < 0
−c
⇒(z1 – z2) = (z3 – z2) ⇒ (|z| + 1) (|z| – 5) < 0
a
M a them a ti cs | 6.63

⇒ |z| + 1 ≥ 0 z1

So (|z| – 5) < 0 z3

Interior of circle x2 + y2 = 25

z2
Sol 34: |z| ω – | ω 2|z = z – ω
2
zˆ 3 = zˆ 2e ; zˆ 1 = zˆ 3eiθ

⇒ zzω − ωωz = z − ω
24i
zˆ 32 = zˆ 1 zˆ 2 =
⇒ z ω (z − ω) = z – ω 24

⇒ z(zω − 1) = ω(z ω − 1)  ….. (i) 1+i


ẑ3 = i=
2
⇒ z(zω − 1) = ω(z ω − 1) 1+i
Unit vector along bisector is , complex number
⇒ Either z ω = zω = 1 or z = ω 2
lying along this vector is 5(1+i)
x ω
= ⇒ zz = ωω ⇒ |z| = | ω |
ω z Sol 4: (C) z1, z2, z3, z4 vertices of square
|z| ( ω – z) = (z – ω )
2
z1 – z2 = i2Im(z1)
( ω – z) (|z| + 1) = 0 ⇒ ω = z
2
z3 – z2 = –2Re(z1)

Exercise 2
z1(x+iy)
z4(-x+iy)

Single Correct Choice Type


z3(-x-iy) z2(x-iy)
Sol 1: (C) z + 4 ≤ 3 Least and greatest value of |z + 1|
z1 − z 2
is imaginary
z3 − z 2
z2 – z4 = [–x + iy – (x – iy)]
-4 = 2Re(z) – i2lm(z)
z1 – z3 = 2x + 2iy = 2Re(z) + i2Im(z)
i.e. distance of z from (–1, 0) z1 − z3 2x + 2iy x + iy x2 − y 2 + 2ixy
= = =
Least is 0; maximum is 2r = 6 z 2 − z 4 2x − 2iy x − iy x2 + y 2

(x = y as it is aqueous)
Sol 2: (D) |z + 3| ≤ 3 Least & greatest value of |z + 1|
ie its minimum and maximum distance from (–1,0) is 1 2ixy
= (Purely Imaginary)
and 5. x + y2
2

az1 + bz 2 + cz3
Sol 5: (D) = z0
(-3,0)(-1,0) a+b+c
A(z1)

Sol 3: (B) z1 = 3 + i 3
c b
z2 = 2 3 + 6i

B(z2) a C(z3)
z0 is incentre.
6 . 6 4 | Complex Number

Sol 6: (A) δ – γ = α – b[parallel vector] |z3| = 1 ⇒ | z | =


| z1 z 2 |
δ=α+γ–β ⇒ z = z1z2
D() A()

A B
Sol 12: (A) + =1
B A
A
C() B() Let y =
B
1
Sol 7: (B) z ≥ 3 y+ 1
=
y
1 1 1 8
z+ ≥ z − =3– = ⇒ y2 − y + 1 =0
z z 3 3
1+ i 3
⇒y=
2
Sol 8: (C)
iπ iπ A 1+ i 3
⇒ =

± z3

± z2
z 2 = z1e 4
z3 = z 2e 4 B 2
/4 z1
(1 ± i) A
z3 – z2 = (z2 – z1) ⇒ 1
= A
2 B

(1 ± i) From Rotation Theorem
z3 = z2 + (z2 – z1)
2 A A iθ B
= e 
B B O
Sol 9: (D) x3 = (4)3 (–1)1/3
1+ i 3
x = –4, –4w, –4w2 ⇒ eiθ =
2
q1 q2 q3 −4 −4ω −4ω2 ⇒ θ =60o
2
q2 q3 q1 = −4ω −4ω −4
and A = B ⇒ ∠OAB = ∠OBA = 60o
q3 q1 q2 −4ω2 −4 −4ω
⇒ AOB is equilateral triangle
2
1 ω ω 0 0 0
2 2
= –64 ω ω 1 = –64 ω ω 1 =0 Sol 13: (D) |z|2 – (z + z ) + i(z – z ) + 2 = 0
2 2
ω 1 ω ω 1 ω
z z – (z + z ) + i(z – z ) + 2 = 0
Let z = a + ib
Sol 10: (D) z = (3 + 7i) (p + iq) is purely imaginary.
a2 + b2 – 2a + i(i2b) + 2 = 0
⇒ 3p – 7q = 0
a2 – 2a + 1 + b2 – 2b + 1 = 0
7q
⇒p= (a – 1)2 + (b – 1)2 = 0
3
2
|z|2 = |3 + 7i|2 |p + iq|2 = 58  p2 + q2  ⇒a=b=1
 
for minimum |z|, q = 3, p = 7 ⇒z=1+i
|z|2 = 58(49+9) = 3364
Sol 14: (D) z1 = –3 + 5i

Sol 11: (A) R(z) Q(z2)


P(z1) z2 = –5 – 3i

A(z3) Eqn of line y = 4x + 17


z1 z z1 z 2
= ⇒z=  4x + 17   17 
z3 arg z = tan–1 
z3 z 2  = tan 4 +
–1

 x   x 
M a them a ti cs | 6.65

 −5 −3  = |z|[(cosθ – sinθ) + i(sinθ + cosθ)]


x ∈ [–3, –5] ⇒ arg z ∈ tan−1 , tan−1 
 3 5  |z|i
= |z|[(cosθ+isinθ)+ (–sinθ + icosθ)]
 5π  i
Only option left is   |z|
 6  = |z| [cosθ + isinθ] + [–cosθ – isinq]
i
Sol 15: (B) | z − 3=| 2 : ( x − 3) + y=
22
4 = P + iP
2 2
| z=| 2 : x + y = 4  Q −P  π
⇒ arg  =
 P  2
Points of intersection lie on the radical axis S1 − S2 =
0
This is right angle triangle with |P| = |Q–P| and right
angle at P.

3 2
Sol 19: (A) |z1 – 1| = |z2 – 1| = |z3 – 1|
2
z 1 + z 2 + z3 = 3
Centroid is at z = 1
3
Radical axis is x =
2 
z1
z2  
x +y =4
2 2 

9 9
⇒ + y2 =
4 ⇒ y2 = 4 – 
4 4
7 ± 7
⇒ y2 = ⇒y=
4 2 Distance of vertical z1 , z 2 and z3 from centroid is same,
3 i 7 which mean centroid coincides with circumcenter.
Complex no. is ±
2 2 Therefore, ∆ is equilateral.

Sol 16: (C) |z – 3i| = 3 ⇒ (x)2 + (y – 3)2 = 9 Sol 20: (C) p = a + bw + cw2
y q = b + cw + aw2
⇒ tanθ =
x
6 x 6(x − iy) r = c + aw + bw2
x 6(x − iy)
cotθ = – = −
z y (x + iy)(x − iy) y x2 + y 2 p + q + r = a(1 + w + w2) + b(1 + w + w2)
x2 + y2 = 6y + c(1 + w + w2)
x 6(x − iy) = (a + b + c) (1 + w + w2) = 0
⇒ − =i
y 6y
p2+q2+r2=(p + q + r)2 – 2pq – 2qr – 2rp

Sol 17: (D) |z – 1| + |z + 1| = 2 = –2(pq + qr + rp)

pq + qr + rp = ab + acw + a2w2 + b2w2 + b2w + bcw2 +


abw3 + cbw2 + c2w3 + caw + …….
(-1,0) (1,0)
= ab+2acw+ab+c2+(a2+2bc)w2+b2w

The portion of real axis between (–1,0) & (1,0) as the = 2ab + c2 + 2acw + (b2 + 2bc)w2 + a2w + 2bc + a2 +
distance between both the point is 2 2baw + (b2 + 2ac)w2 + c2w + 2ac + b2 + 2bcw + (c2 +
a2 + 2ab)w2
2 π 
Sol 18: (C) Q = 2 z cis  + θ  = (2ab + 2bc + 2ac) (1 + w + w2)
4 
+ (c2 + b2 + a2) (1 + w2 + w)
 π  π 
= |z| 2 cos  + θ  + isin  + θ  
 4  4  p2 + q2 + r2 = 0 = 2(pq + qr + rp)
6 . 6 6 | Complex Number

Previous Years’ Questions ⇒


z1 + z3
=
z2 + z 4
⇒ z1 + z3 = z2 + z4
2 2
n
1 + i
Sol 1: (D) Since,   =1 Sol 5: (C) Given, |z1 + z2| = |z1| + |z2|
1 −i
On squaring both sides, we get
n
1 + i 1 + i
⇒  × 1
 = |z1|2 + |z2|2 + 2|z1| |z2|cos (arg z1 – z2)
1 −i 1 +i
n = |z1|2 + |z2|2 + 2|z1| |z2|
 2i 
⇒   = 1 ⇒ in=1
2 ⇒ 2|z1| |z2| cos (arg z1 – arg z2) = 2 |z1| z2|
The smallest positive integer n for which in = 1 is 4
⇒ cos (arg z1 – arg z2) = 1
∴n=4
⇒ arg (z1) – arg (z2) = 0
z − 5i
Sol 2:(A) Given, =1
z + 5i Sol 6: (D) Since, sin x + i cos 2x
⇒ |z – 5i| = |z + 5i| = cos x – i sin 2x
(if |z – z1| = |z – z2|,
⇒ sin x –i cos 2x = cos x – i sin 2x
Then it is a perpendicular bisector of z1
⇒ sin x = cos x and cos 2 x = sin 2 x < x
y
⇒ tan x = 1 and tan 2x = 1
π π
(0,5) ⇒x= and x = which is not possible at same time.
x’ 4 8
x
Hence, no such value exists.
(0,-5)

Sol 7: (B) (1 + ω)7 = (1 + ω) (1 + ω)6


y’
= (1 + ω) (–ω 2)6
∴ Perpendicular bisector of (0, 5) and (0, – 5) is x-axis.
=1+ω
⇒ A + Bω = 1 + 0 ⇒ A = 1, B = 1
Sol 3: (B) Since, |w| = 1
1 − iz
⇒ = 1 ⇒ |z – i| = |1 – iz| Sol 8: (D) Since, |z| =|w| and arg (z) =π – arg(w)
z −i
⇒ |z – i| = |z + i| Let w = reiθ, then ω = re–iθ

∴ z re
= =( ) rei π .e−i θ
π−θ
(∵ |1 – iz| = |– i| |z + i| = |z + i|)
∴ It is a perpendicular bisector of (0, 1) and (0, –1)     = – re–iθ = – ω
i.e., x-axis ∴ z lies on the perpendicular bisector of the line joining
Thus, z lies on real axis. –iw and – iω is the mirror image of – iw in the x-axis,
the locus of z is the x-axis
Sol 4: (B) Since, z1, z2, z3, z4 are the vertices of parallelogram.
Letz = x + iy and y = 0
D(z4) C(z3)
Now,|z| ≤ 1 ⇒ x2 + 02 ≤ 1
⇒–1≤x≤1 ….. (i)
∴ z may take values given in (i).

A(z1) B(z2)
Sol 9: (D)
∴ Mid-point of AC = mid-point of BD (1 + ω − ω2 )7 = ( −ω2 − ω2 )7 ( 1 + ω + ω2 = 0)
=( −2ω2 )7 =( −2)7 ω14 =−128ω2
M a them a ti cs | 6.67

Sol 10: (B) Applying R 2 → R1 : R3 → R3 − R1


13 13 13
1 1 1
∑ (in + in+1 ) =∑ in (1 + i) =(1 + i) ∑ in
2 2
=n 1=n 1 =n 1 = 0 −2 − ω ω −1
2 3 13  i(1 − i)  2
= (1 + i)(i + i + i + ..... + i ) = (1 + i)   0 ω −1 ω−1
 1−i 
=(1 + i)i =−1 + i = ( −2 − ω )(ω − 1) − (ω2 − 1)2
2

=−2ω + 2 − ω3 + ω2 − (ω4 − 2ω2 + 1)


Alternate solution: = 3ω2 − 3ω = 3ω(ω − 1) ( ω4 = ω)
Since, sum of any four consecutive powers of iota is
zero.
Sol 14: (B)
13
∴ ∑ (i + i n n+1 2 13 2 3
) =(i + i + ..... + i ) + (i + i + ..... + i ) 14
Given, (1 + ω2 )n= (1 + ω4 )n
n=1
⇒ ( −ω)n = ( −ω2 )n (ω3 = 1 and 1 + ω + ω2 = 0)
=i + i2 =i − 1
⇒ ωn =1
⇒ 3 is the least positive value of n.
n=
Sol 11: (D)
6i −3i 1 Sol 15: (D) Let OA=3, so that the complex number
Given, 4 3i −1 = x + iy associated with A is 3eiπ / 4 . If z is the complex number
20 3 i associated with P, then

6i 1 1 z − 3eiπ / 4 4 −iπ /2 4i
= e = −
⇒ − 3i 4 −1 −1 = x + iy 0 − 3e iπ / 4 3 3
20 i i ⇒ 3z − 9eiπ / 4 =
12ieiπ / 4
⇒ x + iy =
0 ( C2 and C3 are identical) ⇒ z =(3 + 4i)eiπ / 4
⇒ x=
0, y =
0 y
p

Sol 12: (A) 4


i/4
Given, | z=
1 | | z=
2 | | z=
3 | 1
3e
A
Now, | z1 |= 1
3
⇒ | z1 |2 =
1
/4
⇒ z1 z 2 =
1, z3 z3 =
1 x’
x
O
1 1 1
Again now, + + 1
= y’
z1 z 2 z3
⇒ | z1 + z 2 + z3 |=
1 z2
Sol 16: (A) Given: is real
⇒ | z1 + z 2 + z3 | =
1 z −1
⇒ | z1 + z 2 + z3 |=
1 Let z = a + ib, then

( a + ib ) a2 − b2 + 2aib × ( a − 1) − ib


2
z2
Sol 13: (B) =
=
( a − 1 ) + ib  ( a − 1 ) − ib 
z − 1 a + ib − 1    
1 1 1
Let =
∆ 1 −1 − ω 2
ω2 ⇒
( )
−b a2 − b2 + 2ab ( a − 1 )
0
=
( a − 1 ) + b2
2
1 ω2 ω

⇒ − a2b + b3 + 2a2b − 2ab =0


⇒ a2b + b3 − 2ab =
0
(
⇒ b a2 + b2 − 2a =
0 )
6 . 6 8 | Complex Number

b 0 or a2 + b2 −=
⇒= 2a 0 2 + 4i sin θ + 3 i sin θ − 6 sin2 θ
=
1 + 4 sin2 θ
⇒ Either real axis or circle passing through origin.
2 − 6 sin2 θ
Given =0
1+z  1 + 4 sin2 θ
Sol 17: (B) Let θ =arg  
1 + z  1
  ⇒ sin2 θ =
1+z  3
⇒ θ = arg   {| z |= 1 ⇒ z z = 1} 1
1 + 1  ⇒ sin θ = ±
  3
 z
 1  −1  1 
⇒ θ =arg ( z ) =⇒ θ sin−1   , − sin  
 3  3
1
Sol 18: (B) Given: The expression z + and z ≥ 2
2
Using triangle in equality JEE Advanced/Boards
z1 + z 2 ≥ | z1 | − | z 2 |
Exercise 1
1 1
⇒ z+ ≥ | z |− Sol 1: z2 + (p + ip’)z + (q + iq’) = 0
2 2
One real root
1 1
⇒ z+ ≥ z−
2 2 (z2 + pz + q) + i(p’z + q’) = 0
If z is real
1 3
⇒ z+ ≥ lies in (1, 2) −q'
2 2 P’z = –q’ ⇒ z =
p'
z2 + pz + q = 0
Sol 19: (B) z1 − 2z 2 = 1
2 − z1 z2 q'2 pq'
− +q=0
⇒ z1 − 2z 2 =2 − z1 z2 p' 2 p'
q'2 − pp'q'+ qp'2 =
0
⇒ ( z1 − 2z 2 )( z1 − 2 z2 ) =( 2 − z1 z2 )( 2 − z1 z 2 )
If eqn. has 2 equal roots
2 2
⇒ z1 − 2 z1 z2 − 2 z1 z 2 + 4 z 2
(p + ip’)2 = 4(q + iq’)
2 2
4 − 2 z1 z 2 − 2z1 z2 + z1
= z2 2
p= − p'2 4q and
= p'2 4q2
2 2 2 2
⇒ z1 + 4 z 2 4 z2
=+ z1 −(p + ip')
The roots are i.e. roots (equal) are imaginary.
2
⇒ z1 +  1 − z 2  − 4  1 − z 2  =
2 2 2
0
   
Sol 2: z = 18 + 26i
⇒  1 − z 2   z1 − 4  =
2 2
0 z0 = x0 + iy0 is cube root of z
  
z0 = (r{cosθ + isinq})1/3 = r1/3 eiθ/3
⇒ z1 = 4
2
{ z2 ≠1 }
 θ θ
⇒ z1 =
2 = r1/3  cos + isin 
 3 3 
2 + 3i sin θ r = (1000)1/2
Sol 20: (C)
1 − 2i sin θ r1/3 = (103/2)1/3 = 101/2

=
( 2 + 3i sin θ )(1 + 2i sin θ ) cosθ =
18
sinθ =
26

1 + 4 sin2 θ 1000 1000


M a them a ti cs | 6.69

cosθ = 0.57, sinθ = 0.82 2 π7(0)


i
(A7) =
7 e 7 + ei2π(1) + .......ei2π(6)
θ θ
cosθ = 3cos – 4cos3  
3 3 =cos0+isin0 + cos2π + isin2π…cos6π + isin6p
0.57 = 3t – 4t3
= [1+1 + …….. 7 times + i (0 + 0……….)] = 7
θ
cos   = 0.2, 0.74, 0.95 i

(K −1)
π  (2n+1)
i  2n

−2n 
3 det A2n = ∏ e 2n = e n 2 

θ
cos = 0.95
3 πi  4n2 −2n 
π
n 2  i(2n2 −n)
θ = e   = en = eiπ(2n – 1)
sin = 0.3162
3 det A2n+1 = ei2pn
z= 10 (0.95 + i 0.3162) = 3 + i det A2n + det A2n+1 = eiπ(2n – 1) + ei2pn
x0 = 3, y0 = 1 =cos(2n–1)π + isin(2n–1)π + cos2nπ + isin2np
x0y0(x0 + y0) = 3.4 = 12 M=0⇒L+M=7

Sol 3: z3 + iz = 1 Sol 5: z1(z12 – 3z22) = 10


z(z2 + i) = 1 z2(3z12 –z22) = 30
z can never be Purely real as (z2 + i) will be imaginary z1 = a + ib,Z2 = c + id
which multiplied with z will not be a real number,
0 as z12 + z22 is real
ab + cd =
hence its locus will never cut x-coo axis Z can never be
imaginary (pure) as (z2 + i) will be of form (a + ib) which ⇒ z13 – 3z1z22 + 3z12z2 – z23 = 40
multiplied by z, cannot form 1 (real) therefore its locus
⇒ z12(z1 + 3z2) – z22(z2 + 3z1) = 40
can never cut y axis.
⇒ (z1 + iz2)3 = z13 + iz23 – 3z12iz2 – 3z1z22
z(z2 + i) = 1
= 10 – 30i
(x + iy) [x2 – y2 + i(2xy + 1)] = 1
⇒ (z1 + iz2)3 = 10 + 30i
x3–xy2+i(2x2y+x+x2y–y3) – 2xy2 – y = 1
⇒ ((z1 + iz2)2)3 = 1000 ⇒ z12 + z22 = 10
x3–xy2–2xy2–y+i(3x2y+x–y3) = 1
x3 – 3xy2 – y = 1 & 3x2y + x – y3 = 0 Sol 6: (z + 1)7 + z7 = 0 has roots z1 …….. z7
x(x – 3y ) = 1 + y & y(y – 3x ) = x
2 2 2 2
Re(z1) + Re(z2) ……… Re(z7) = ?
1 1 + y x  1  y + x2 + y 2 
|z|2= −  +  ⇒ |z|2 = −   (z + 1)7 = (–z)7
2 x y 2  xy 
( ( a + 1) + ib ) = ( −a − ib )7
7

1   2xy + 1  − y
|z|2 = − y + | z |2  ⇒ |z|2  =
2xy    2xy  2xy One of the solution is
−1
−Im(z) ⇒ a + 1 = –a ⇒ a = and b = 0
⇒ |z| = 2
2Re(z)Im(z) + 1 −1
⇒z=
2
−7
Sol 4: An = dia(d1, d2, ……dn) z1 + z2 + z3 ……. z7 = + i(0)
2
di = ai-1; α = ei2π/n −7
Re(z1 +……….+ z7) =
2

⇒ dn = i (n−1) Im (z1 + z2 …+ z7) = 0
e n
L = Tr(A7)7
Sol 7: z = (1)1/7
M = det A(2n+1) + det(A2n)
α + β = (z + z2 + z3 + z4 + z5 + z6) = (-1) = -1
6 . 7 0 | Complex Number

αβ = (z + z4 + z2) (z3 + z5 + z6) a = 2c; b = 3c; b + c = 3


3 3 9
= z4 + z6 + z7 + + z7 + z9 + z10 + z5 + z7 +z8 4c = 3 ⇒ c = ,a = ,b =
4 2 4
= z 4 + z6 + 3 + z5 + z + z 3 + z2
0 0 1
= 3 + z + z 4 + z6 + z5 + z3 + z2 = 3 – 1 = 2
1 9 3
∆(0, z3, z4) = 1
Eqn → x2 + x + 2 = 0 2 4 4
3 3
1
Sol 8: z5 – 32 = z5 – 25 2 4
1 3 3 9 m
= (z – 2) (z2 – pz + 4) (z2 – qz + 4) = × × = =
2 4 4 32 n
= z5 + z4(–q – p – 2) + z3(+pq + 8 + 2p + 2q) + z2(–4p ⇒ m + n = 32 + 9 = 41
– 4q + (pq + 8) –2)

+ z(16 + 8p + 8q) – 32 = 0 Sol 11: (1 + x)n


an = C0 + C3 + C6 + …….C9
⇒p+q+2=0
bn = C1 + C4 + ………
2p + 2q + 8 + pq = 0
cn = C2 + C5 + C8 + ………..
⇒ pq + 4 = 0 ⇒ pq = –4
a3 + b3 + c3 – 3abc
4
p + q = –2 ⇒ p – = –2 (1+x)n = C0 + C1x+ C2x2+ C3x3……..Cnxn
p
⇒ p2 + 2p – 4 = 0 2n = C0 + C1 + C2 + ……..Cn

⇒ p2 + 2p = 4 (1+w)n = C0+w(C1 + C4 + C7 + ……..)


+ w2(C2 + C5 + C8 + ……..) + C3 + C6 ……

Sol 9: z + z ≥ 1 z + z z1 z2 (1 + w2)n = C0 + w2(C1 + C4 + C7 ……)


1 2
2 1 2 ( ) z1
+
z2
RHS + w(C2 + C5 + C8 + …..) + C3 + C6 …….

1 z1 z2 2n+ (1 + w)n + (1 + w2)n = 3(C0 + C3+C6 …)


(
z + z2
2 1
) z1
+
z2 2n+ (–w2)n + (–w)n = 3(C0 + C3+C6 …)

z1 z2 z1 z2 2nw2 + w(–w2)n + (–w)n = 3w2(C1 + C2+C7 …)


+ ≤ 2
=
z1 z2 z1 z2
2n + ( −ω2 )n + ( −ω)n
an =
1 1 z1 z2
2
(
× 2 z1 + z 2 ≥ )
z + z2
2 1
( ) z1
+
z2
3
2n ω2 + ω( −ω2 )n + ( −ω)n
bn =
1 z1 z2 3ω2
z1 + z 2 ≥
2 1
(
z + z2 ) z1
+
z2
ω2 2n + ( −ω2 )n + ω( −ω)n
cn =
Sol 10: 3ω2

z1 − z 2 iπ /2
Sol 12: = e= i
z4
(2,1)
z3 z3 − z 2

z1 − iz3 z1 (1 + i) + z3 (1 − i)
(0,0) z1 z2 (3,0) = z 2 ⇒ Z2 =
1−i 2
z1 − z 4
z1 = (a, 0), z2 = (b, 0), z3 = (b, c), z4 = (a, c) e−iπ /2 = −i ⇒ z1 – z4 = –i(z3 – z4)
z3 − z 4
b – a = c(square)
M a them a ti cs | 6.71

z1 (1 − i) + z3 (1 + i) a (1 + i ) + b (1 + i ) + c (1 + i ) + d =
3 2
⇒ z4 = 5
2
( ) ( )
⇒ a 1 + i3 + 3i2 + 3i + b 1 + i2 + 2i + c (1 + i ) + d =
5
Sol 13: f(z) = (a + ib)(z) = c + id
⇒ a (1 − i − 3 + 3i) + b (1 − 1 + 2i) + c (1 + i) + d =5
z = (x + iy)
⇒ a ( −2 + 2i) + b ( 2i) + c (1 + i) + d =5
Image = c – id
(c – x)2 + (y + d)2 = c2 + d2 ⇒ − 2a + c + d = 5 and 2a + 2b + c =0

x2 + y 2 − 2cx + 2dy =
0 From (i), we have

a2 + b2 = 64 d – a = 5 and 3a + 2d = 0

c = ax – by ⇒ a =c =−2 and b= d= 3
( −2) + 32 + ( −2) + 32
2 2
d = bx – ay ∴ a2 + b2 + c2 + d2 =

⇒ x2 + y2 – 2x(ax – by) + 2y(bx – ay) = 0 =4+9+4+9


= 26
⇒ x2(1 – 2a) + y2(1 – 2a) –2bxy + 2bxy = 0
⇒ (x2 + y2) (1 – 2a) = 0 n

1 1 255 u
Sol 16: ∑ ( zk − wk ) =
0
k =1
⇒a= ⇒ b2 = 64 – = =
2 4 4 v ⇒ z1 + z2 + z3 + z4 + z5
⇒ u + v = 259 ⇒ Z1 + Z 2 + Z3 + Z 4 + Z5
= 32 + 170i − 7 + 64i − 9 + 200i + 1 + 27i − 14 + 43i
Sol 14: = 3 + 504i

cosx + nC1cos2x + nC2cos3x + .......... + nCncos(n + 1)x If y-intercept is 3, then eq. of line

= cosx + nC1cos2x + nC2cos33x + .......... + nCncosn + 1x y = mx + 3

1 ++nnCCcisx n n 2 2 n n yi = mxi + 3 for i=1,2…….5


==cos
Cisx [1 11cosx++C2Cciscos + ..........++n CnC
x +x.............. cos
nCis x x]
 2 n 
Now, z1 + ……………..z5 = 3 + 504i
= cosx[1 + cosx]n
(x1 + x2 + ……… + x5) + i(y1 + y2 + ……..y5) = 3 + 504i +
n
 x x i{m(x1 + x2 ………… + x5) + 15} = 3 + 504i
=
= cosx
Cisx 2cos cosx
Cisx 
 2 2 x1 + x2 + ………….. + x5 = 3
xx nn
==22nnCos
Cosnn cos
Cis cos
Cis Cisx
Cisx m(x1 + x2 + ……….. + x5) + 15 = 504
22 22
⇒ 3m + 15 = 504
xx nn++22 ⇒m= 163
cosnn cos
==22nncos cis
cis xx
22  22 
R
x (n+ 2) x n + 2   Sol 17: P(3,9)
n n
2 cos cos + isin   x Q 37
2 2  2   53
(3,4)
By comparing, we get the desired results. Hence, proved
53

Sol 15: f ( x ) = ax3 + bx2 + cx + d f ( i ) = 0


⇒ − ai − b + ci + d = 0 + io ⇒ − a + c = 0 and d – b = 0
3
⇒a =c and b = d …(i) z − 3 − 4i =5 tan–1 (37°) in clockwise direction
4
And f (1 + i) = 5 It reaches Q (3, 9) above centre
Then it moves 2 unit downwards i.e. R (3, 7)
6 . 7 2 | Complex Number

p
32  10  2qπ 2qπ   2k + 2α1β1 + 2α2β2 − α12 − α22 − β12 − β22
Sol 18: ∑ ( 3p + 2 )  ∑  sin − icos  Radius =
= p 1=  q 1 11 11  4

p
  10 2qπi  Sol 21: f(z) = |z3 – z + 2 |z| = 1
= ∑ ( 3p + 2 )  −i  ∑ e 11 − 1 
  q=0 
 (f(z))2 = |z3 – z + 2|2=(z3 – z + 2)( z 3 – z + 2)

= 1–z2+2z3– z 2 +z z –2z+2 z 3 –2 z +4
32 32  32 p 
= ∑ (3p + 2 ) ip = ∑ (3p ) ip 2 ∑ i = 0  = 6 – (z2 – z 2 ) –2(z + z ) + 2(Z3 + z 3 )
p =1 p =1  1 
= 6–2(a2–b2)–4(a)+2(z+ z )(z2+ z 2 – z z )
= 3i(1 − 3 +  − 31) − 3(2 − 4 +  − 32) =48(1 − i)
= 6–2(a2–b2)–4(a)+4a(2a2–2b2–1)

a b c = 6–8a–2(2a2–1) + 8a(2a2–1)
Sol 19: = = = k
1−b 1−c 1−a f(z) = 16a3 – 4a2 – 16a + 8
a = k – kb f’(z) = 48a2 – 8a – 16 = 8(6a2 – a – 2)
b = k – kc = 8(6a2–4a + 3a – 2)
c = k – ka = 8(2a(3a – 2) + 1(3a – 1))
a = k – k2 + k2(k – ka) = 8(2a + 1) (3a – 2)
a = k – k + k –k a)
2 3 3
−1 2
a= a=
k −k +k 2 3 2 3
a= = b= c
1 + k3 2
For a =
3
but a ≠ b ≠ c 8 4 2
f(z) = 16 × –4× – 16 × +8
i.e. k = –1 ⇒ k = –w, –w
3 2 27 9 3
128 – 48 − 288 + 202
= <0
Sol 20: |z – a|2 + |z – b|2 = k 27
Locus of z is a circle −1
For a = ; f (z) = –2 – 1 + 8 + 8 = 13
(x – a1)2+(x – b1)2+(y – a2)2+(y – b2)2 = k 2
Maximum value of f(z) = 13
⇒ 2x2 + 2y2 – 2x(a1 + b1) – 2y2(a2 + b2)

+ a12 + a22 + b12 + b22 – k = 0 Sol 22: |a + bw + cw2| + |a + bw2 + cw| ≥


 α + β1   α2 + β2  |a + bw + cw2 + a + bw2 + cw|
⇒ x2 + y2 – 2x  1  − 2y  
 2   2  = |2a – b – c| = |a – b + a – c|
 α2 + α22 −k 
+ β12 + β22 Minimum value will occur when
+  1 =0
 2 
  a – b = k, a – c = –k
r > 0 i.e. g + f – c > 0
2 2
i.e. k = 1
2 2
 α1 + β1   α2 + β2   α2 + α2 + β2 + β2 − k 
+ − 2  1 2 1 2 >0 a – b = 1 and a – c = –1
 2   2   4 
      with a & b being least possible value integer values
⇒ – α12 + α22 − β12 − β22 + 2α1β1 + 2α2β2 + k > 0 ⇒ a = 2, b = 1, c = 3
⇒ k > α12 + α22 + β12 + β22 − 2α1β1 – 2α2β2 |a + bw + cw2| + |a + bw2 + cw|

⇒ k > (α1 − β1 )2 + (α2 − β2 )2 ⇒ k > |α – b|2 = 2|2 + w + 3w2| = 2|1 + 2w2|

 α + β1 α2 + β2  = 2|w2 – w| = 2|(– 3 )| = 2 3
Centre =  1 , 
 2 2  = 12 = n1/4 ⇒ n = 144
M a them a ti cs | 6.73

Sol 23: x7 + x6 + ……….. 1 = 0 ⇒ b = 26 + 25 = 51

 x8 − 1 
 =0 Sol 26: z5 + 1
 x −1  2
  3 1 = (z3 + 1) z2 + (1 – z2)

4 = (z + 1) (z2 – z + 1) z2 + (1 – z2)
= (1 + z)[(z2 – z + 1) z2 + 1 – z]
5 7 = (1 + z)[z4 – z3 + z2 – z + 1]
6
= (1 + z)[z2 + az + 1] [z2 + bz + 1]
Total area = unshaded area + shaded area
⇒ b + a = –1
ba + 2 = 1
0 0 1
Unshaded area = 6 × 1 0 1 1 ab = –1
1 1 1
1 ⇒a– = – 1 ⇒ a2 + a – 1 = 0
2 2 a
6 1 6 −1 ± 5 −1 + 5
= × = ⇒a= ⇒a= = –2cos108°
2 2 2 2 2 2
−1 − 5
1 1 1 1 And b = = –2cos36°
Shaded area = 2 × × × = 2
2 2 2 2
∴ Factors are (1 + z) (z2 – 2zcos36° + 1) (z2 – 2cos108° + 1)
6 1 6+ 2 6 2 +2 Since ab = –1
Total area = + = =
2 2 2 2 2 4 π π
⇒ 4cos36°cos108° = -1 ⇒ 4 cos cos = 1
5 10
3 2 +1 a b +c
= =
2 d Sol 27: x = 1 + i 3
a+b+c+d=3+2+1+2=8 y=1–i 3
z=2
Sol 24: N = (a + ib)3 – 107i x = –2wy = –2w2z = 2
= a3 – ib3 + 3a2ib – 3ab2 – 107i xp + ypp > 3 prime (P is odd)
N = (–b + 3a b – 107)i + a – 3ab
3 2 3 2
= –2pwp – 2p w2p = –2p(wp + w2p) = 2p = zp
⇒ 3a b – b = 107 ⇒ b(3a – b ) = 107
2 3 2 2

⇒ b =1 and 3a2 – b2 = 107 ⇒ a2 = 36 Sol 28: f(z) = a(z – i) + i ⇒ f(i) = i  … (i)

⇒ N = a3 – 3ab2 = 216 – 18 = 198 f(z) = b(z + i) +(1+ i) ⇒ f(–i) = 1+i  ... (ii)
f(z) = c(z2 + 1) + k1z + k2 … (iii)
Sol 25: x + ax + bx + cx + d has 4 non-real roots.
4 3 2
Substituting values from (i) and (ii) in (iii)
α + β = 3 + 4i ⇒ i = k1i + k2 and i + 1 = –k1i + k2
gδ = 13 + i(γ = α , δ = β ) 1 i
⇒ k2 = i + and k1 =
= 13 + i aβ = 13 – i
⇒ αβ 2 2
,
α + β = 3 + 4i iz 1
∴ Remainder = + i +
2 2
⇒ aβ + bγ + gδ + dα + aγ + bδ = b
⇒ aβ + β α + αβ + α α + β β + α β = b
⇒ 13 – i + 13 + i +( α + β )(α + β) = b
= 26+ (3 + 4i) (3 – 4i) = b
6 . 7 4 | Complex Number

Sol 29: (a) (c) α = 6 + 4iβ = 2 + 4i


z−α π
amp  =
 z −β  6
 (x − 6) + (y − 4)i (x − 2) − i(y − 4)  π
tan–1  (x − 2) + i(y − 4) (x − 2) − i(y − 4)  = 6
 
Re(z) = (x – 6) (x – 2) + (y – 4)2
Im(z) = (x – 2) (y – 4) + (x – 6) (y – 4)
A = {z| |z| ≤ 2}
(x − 2)(y − 4) + (x − 6)(4 − y) 1
B = {z| (1 – i)z + (1 + i) z ≥ 4} ⇒ =
2
(x − 6)(x − 2) + (y − 4) 3
Let z = a + ib
−4x − 2y + 8 + 4x + 6y − 24 1
⇒ (1 – i)(a + ib)+(1 + i)(a – ib) ≥ 4 ⇒
2 2
=
x + y − 8x − 8y + 12 + 16 3
⇒a+b+a+b ≥ 4⇒a+b ≥ 2
πr 2 1 2  π 1  2 ⇒ x2 + y2 – 8x – 8y + 28 = 4 3 y – 16 3
area = − r = −  r = (π – 2)
4 2  4 2 ⇒ x2+y2–8x+y(–8–4 3 )+28+16 3 =0
1 1  x +i
(b) f(x) = =  
x −i x −i x +i ⇒r= 16 + (4 + 2 3)2 − 28 − 16 3
x  1 
= + i 
2
x + 1  x2 + 1  = 16 + 16 + 12 − 28 + 16 3 − 16 3
k
x – coordinate = = 16 = 4(p)
2
k +1
1
y – coordinate =
k2 + 1
∴ Locus of the function will be
Exercise 2
x2 + y2 – y = 0 Single Correct Choice Type
This is circle with diameter 1
1 Sol 1: (C) z2 + z + 1 is real and +ve
Hence the area of the square =
2 x2 – y2 + x + 1 + i(2xy + y)
Sol 30: (a) (1 + w + ……..wn)m, m, n ∈ N x2 – y2 + x + 1 > 0 and 2xy + y = 0
−1
n ∈ 3k⇒ 1 ⇒ y(2x + 1) = 0 ⇒ y = 0 or x =
2
n ∈ 3k + 1⇒ (–w2)m If y = 0 x2 + x + 1 > 0 ⇒ z represents real axis
n ∈ 3k + 2⇒ 0 −1 3
If x = − y 2 > 0 , Line segment joining
m = 1 ⇒ (–w2), m = 2 ⇒ (w), m = 3 ⇒ (–1) 2 4
 1 − 3  1 3
m = 4 ⇒ (w2), m = 5 ⇒ (–w), m = 6 ⇒ (1) − ,  to  − , 
 2 2   2 2 
No. of distinct elements are 7(S)
(b) Real coefficient,
Sol 2: (B) zz 3 + z3 z =
350
Root → 2w, 2+3w, 2+3w , 2–w–w2 2

|z|2 (z2 + z 2 ) = 350


Roots are 2w, 2 + 3w, 2 + 3w2, 3

Other root will be 2ω -z- z


lzl
Total no. of roots are 5 (q)

-z -
z
M a them a ti cs | 6.75

Length of diagonal 2|z| ⇒ x ∈  −2,2 & y ∈  −2 3,2 3 


 
|z|2 (x2 - y2 + 2ixy + x2 – y2 – 2ixy)=350
Set of points lie on radius of circle
350
(x + y ) (x – y ) =
2 2 2
= 175 2
2
Sol 5: (C) z2
As x and y are integers z1+z2
x = ±4; y = ±3
z1-z2
⇒ |z| = 5
z1
Length of diagonal is 10

Sol 3: (A) z12 − 2z1z 2 + 2z 22 =


0
z1 + z 2 π
arg =
( z1 − z2 )
2
+ z 22 =
0 z1 − z 2 2

⇒ ( z1 − z 2 ) = z 2 and ( z1 − z 2 ) =
2
−z 22 =z1 − z 2 = iz 2  z + z2  π
tan–1  1  =
and  z1 − z 2  2

2 |z1 + z2| ≠ |z1 – z2|


 z1  z1
  − 2 + 2 =0 Diagonals are perpendicular but not equal ie figure
 z2  z2 z1
represented is rhombus but not a square.
z
⇒ 1 =+ 1 i z1
z2 z1- z2
Sol 6: (A) Condition for equilateral
z1
⇒ =2 z2 ∆ : z12 + z 22 + z32= z1 z 2 + z 2 z3 + z3 z1
z2
O z2 z3 – 3az2 + 3bz + x = 0
Now,
z1, z2, z3
z1 − 0 z1 i∠z1oz2
= e ⇒ z1 + z2 + z3 = 3a
z2 − 0 z2
i∠z1oz2 1+ i ⇒ z1z2 + z2z3 + z3z1 = 3b
e = = eiπ / 4
2
⇒ z1, z2, z3 = –x
⇒ ∠z1oz 2 = 45o = ∠oz1z 2 2 2 2
(z1 + z2 + z3)2 = z1 + z 2 + z3 + 2(3β)
Also, (3α)2 = 9b
z 2 − z1 z 2 − z1 i∠0z2 z1
= e a2 = b
z2 z2

⇒e
i∠oz2 z1
= i ⇒ ∠oz 2 z1 = 90o Sol 7: (D) z1

Sol 4: (B) 2 1
23 O G C
-2 z2 z3
4

23 z1 + z 2 + z3
= Centroid,
= zc 0
3
π
|Z| ≤ 4 and Arg ( z ) =
3 z1 + z 2 + z3 2z C + z O
= ⇒ zO = z1 + z2 + z3
3 3
x2 + y2 ≤ 16 and y = 3 x
6 . 7 6 | Complex Number

Sol 8: (C) Sol 11: (D) |z-4-3i|=2


z lies on a circle shown in the figure |z|=|z-0| is nothing
but distance from origin
-1 1 y
3
Q
C
(z + 1)4 = 16z4
(4i3)
⇒ z + 1 = 2z, 2iz, –2z, –2iz P
1 1 1 1 −1 − 2i −1 + 2i
⇒ z = 1, , − , = − , 1, ,
2i − 1 3 −1 − 2i 3 5 5 O x
z1 + z 2
Point equidistant is z =
2 Minimum = OP = OC-CP
 −1  =42 + 32 − 2 =5-2=3
Where z1 = (1, 0), z2 =  ,0 
 3 
Maximum = OC + CQ =5+9=7
1
1−
z= 3 =1
2 3 Sol 12: (A) z = 1 – sinα + icosa

|z| = 1 + sin2 α − 2sin α + cos2 α


Sol 9: (B) |z – 2| = 3
= 2 − 2sin α = 2(1 − sin α )

cos α  1 
arg z = tan–1 = tan−1  
1 − sin α  sec α − tan α 
2
 α α
 sin + cos 
1 + sin α  2 2
= tan–1 = tan–1
cos α 
2
2 α 2 α
 cos − sin 
 2 2
|z – 2 – 3i| = 4  α α  α
 sin + cos   1 + tan 
⇒S1 = (x – 2) + y = 9
2 2
= tan−1  2 2  = tan–1  2
⇒S2 = (x – 2)2 + (y – 3)2 = 16  cos α − sin α   1 − tan α 
   
 2 2  2
Both circles are intersecting. So, radical axis will be
  π α
S1 – S2 = 0 ⇒ 9 – 6y = 7 ⇒ 3y – 1 = 0 = tan–1 tan  π + α   = +
  4 2  4 2

Sol 10: (A) z3 + iz − 1 =0


z1 + z 2 z1 + z 2
Let z=k (real) Sol 13: (D) + z1 z 2 + − z1 z 2
2 2
⇒ k + ik ‒ 1 = 0
3

k3 + ik ‒ 1 = 0 and k = 0 z1 + z 2 + 2 z1 z 2 + z1 + z 2 − 2 z1 z 2
=
If k = 0, then 0 ‒ 1 = 0 2

( ) +( )
2 2
Not possible z1 + z 2 z1 − z 2
=
Therefore z ≠ k (real) 2
Hence, no real solution
2 ( z1 ) + 2 ( z 2 )
2 2 2 2
= = z1 + z2
2
M a them a ti cs | 6.77

Sol 14: (A) u2 – 2u + 2 = 0 a3 a1a4 b2 a4 b 4 a2 + a4 a2 b2a22


+ = a + = = = 1
⇒(u – 1)2 = –1 a1a2 a2a3 2 a2b2 a22b2 b2a22
⇒u – 1 = ±i
Sol 17: (D) (1+z)6+z6=0
⇒u = 1 + i, 1 – i = (α, β)
⇒ (1 + z ) =
6
cotθ = x + 1 −z6
Now,
(x + α)
n
( cot θ + i) − ( cot θ − i)
n n
− (x + β)n
= 6
α −β 2i  1+ z 
⇒ −1
 =
 z 
( cos θ + isin θ ) − ( cos θ − isin θ )
n n
(4, 3)
= 1
⇒ 1 + = ( −1)
n 1/6
2isin θ
z
eiθn − e−iθn sinnθ
= = 1 2Kπ + π
n
2isin θ sinn θ =
⇒ cos −1
z 6
Sol 15: (B) A1…….An vertices of regular polygon in a
circle of radius R 1  2Kπ + π  2Kπ + π π  
⇒ =− 2sin cos  − 
z  12  12 2  
A2
A1
A3 1   π 2Kπ + π  
⇒ z =− cos  − 
R 2Kπ + π  2 12  
2sin
12
A4
1 2Kπ + π 
z=
− 1 + icot 
(A1A2)2 + (A1A3)2 …….(A1An)2 2 12 
π 1
A1A2 = 2Rsin ⇒ z lies on line x =

n 2
2π ⇒ All roots are collinear.
A1A3 = 2Rsin
n

A1A2 = 2R Sol 18: (A)
n
 2π 2 2π 2 3π 2 (n − 1)π 
4R2 sin n + sin n + sin n ...sin n 


 2π 4π π
=−2R 2 1 − n + cos + cos + ...cos(2n − 1) 
 n n n
 
  z10 – z5 = 992
sin πk 2nπ 
=–2R2 1 − n + − cos = 2nR2
 π n  z5(z5 – 1) = 32(31)
 sinn   
 n  z5 = t
⇒ t2 – t = 992 ⇒ t2 – 32t + t – 992 = 0
Sol 16: (B) z4 + a1z3 + a2z2 + a3z + a4 = 0
⇒ t = 32, –31 ⇒ z5 = 32, –31
z = ib
⇒ z5 = 32 has 2 roots with –ve real Part
b4 – ib3a1 – b2a2 + iba3 + a4 = 0
and z5 = –31 has 3 roots with –ve real part
b – b a2 + a4 = 0 ⇒ b a2 + a4a2 = b a2
4 2 4 2 2

⇒ ba3 – b3a1 = 0 ⇒ a3 = b2a1


6 . 7 8 | Complex Number

Sol 19: (A) a|z1| = b|z2| Sol 24: (A, B, C, D) (a) a, b, x y is ∈ R

a z2 az bz az1 a + ib
= a − ib
= T = 1 + 2 let =z x + iy
b z1 bz 2 az1 bz 2
a2 − b2 + 2abi
⇒ x + iy =
1z z a2 + b2
T=z+ =z+
zz | z |2 2
a −b 2
2ab
⇒x= ,y=
|z| = 1 ⇒ T = z + z = 2Re(z) 2
a +b 2
a + b2
2

Re(z) ∈ (–1, 1) ⇒ x2 + y2 = 1 A is correct

⇒ T ∈ [–2, 2] 1 − ix
(b) = a − ib
1 + ix
Sol 20: (B) (p+q)3 + (pw+qw2)3 + (pw2+qw)3 1 − x2 − 2ix
⇒ = a − ib
= p3+q2+3p2q+3pq2+p3+q3+3pq2w5 1 + x2
1 − x2 2x
⇒a= ,b=
+3p2qw4+p3+q3+3p2qw5+3pq2w4 1+x 2
1 + x2
= 3(p3+q3) ⇒a2 + b2 = 1 B is correct
x
1 + i 3 
( )
x a + ib
Sol 21: (A) 1 + i 3 x
2 ⇒ 
=  =1 (c) = x − iy

 2  a − ib
⇒ (–w)x = 1 (a + ib)2
⇒ = x − iy
⇒ x = 6, 12, 18, ……... a2 + b2
a2 − b2 + 2iab
It forms an AP ⇒ x + iy =
a2 + b2
a2 − b2 − 2iab
⇒ x + iy = ⇒ |x + iy| = 1
Multiple Correct Choice Type a2 + b2
y − ix
Sol 22: (B, C) x2+(p+iq)x + 3i = 0 (d) = y + ix
a + ib
a2 + b2 = 8 y 2 − x2 − 2ixy
⇒ = a + ib
(α + β) = –(p + iq) y 2 + x2
aβ = 3i y 2 − x2 + 2ixy
⇒a – ib = ⇒ |a – ib| =1
x2 + y 2
⇒ (α + β)2 = 8 + 6i = p2 – q2 + 2ipq
p2 – q2 = 8 Sol 25: (A, D) z = x + iy = r(cosθ + isinθ)
2pq = 6
z
= x + iy
= r(cos θ + isin θ)
⇒p = 3, q = +1 or p = –3, q = –1
 θ θ
r eiθ /2
== r cos + isin  for y > 0
 2 2
Sol 23: (A, D) |z1| = |z2| ⇒ a2 + b2 = c2 + d2
z1 = a + ib, a > 0  1 + cos θ 1 − cos θ 
= r +i 
z2 = c + id, d < 0  2 2 
z1 + z 2 (a + c) + i(b + d) r  r +x +i r −x r +x +i r −x
= =   = [A]
z1 − z 2 (a − c) + i(b − d)
2  r  2
(a + c) & (b + d) can be zero, so value can be zero Similarly
(a – c) & (b – d) can be simultaneously zero ⇒ purely 1 
imaginary. = r + x − i r − x  for y < 0
2 
M a them a ti cs | 6.79

Sol 26: (A, D) α = eiθ = β [C and D are correct]


( az1 + bz1 )( cz2 + dz2 ) = ( cz1 + dz1 )( az2 + bz2 )
Sol 29: (A, B, D) α = 3z – 2
adz1 z2 + bc z1 z2 = bcz1 z2 + ad z1 z2
β = –3z – 2
⇒ ad = bc
|z| = 1
a c
= (A is correct)
b d |α + 2| = 3 → A is correct
b d
⇒ arg z1 = tan–1 a , arg z2 = tan–1 c |β + 2| = 3 → B is correct

α – β = 6Z D is correct
⇒ arg z1 = arg z2
z1 z2 ⇒ α – β and z, both move on same circle
or z1 z2 = z1 z 2 ⇒ =
z1 z2
a + ib c + id Sol 30: (B, D) (in + i–n) is
⇒ = ⇒ –ad + bc = 0
a − ib c − id
22n + 22n 2.2n 2n+1
(A) = = (1 + i)2n =
2n (1 − i)2n (1 − i)2n
2 2n
Sol 27: (A, B, C)
(2i)n
= 2in= in + in
n−1
2

(1 + i)2n + (1 − i)2n (2i)n + ( −2i)n


(B) = =in + (–i)n
2n 2n
= in + i–n
−(2i)n 2n
(C) + = –in + in = 0
|z1| = 1 and |z2| = 2 2n ( −2i)n
2n 2n
Max |2z1 + z2| (D) + i−n + in
=
n n
(2i) ( −2i)
|2z1 + z2| ≤ 2|z1| + |z2| = 4 (A is correct)

Min |z1 – z2| ≥ |z1| – |z2| Sol 31: (A, B, D)


 
|z1 – z2| ≥ 1(B is correct)  
( 2
log14 13 + z − 4i + log196  ) 1
=0
( )
2
1 1  13 + z 2 + 4i 
z2 + ≤ z2 + ≤ 2 (C is correct)  
z1 z1  
13 + z 2 − 4i
log14 =0
13 + z 2 + 4i
α −β
Sol 28: (A, B, C, D) =1 ⇒ 13 + |z2 – 4i| = 13 + |z2 + 4i|
1 − αβ
2 2 ⇒ |z2 – 4i| = |z2 + 4i|
α − β = 1 − αβ

⇒ (α – β) ( α – β ) = (1 – α β) (1 − αβ ) ⇒ |x2 – y2 + 2ixy – 4i| = |x2 – y2 + 2ixy + 4i|

|a|2 + |b|2 – α β – β α =1 – α β – β α + |a|2 |b|2 ⇒ (x2 – y2)2 + (2xy – 4)2 = (x2 – y2)2 + (2xy +4)2

|a|2 (α – |b|2) = (1 – |b|2)⇒(|a|2 – 1) (1 – |b|2) =0 ⇒ –16xy = 16xy ⇒ either x = 0 or y = 0

⇒|a| = 1 A is correct If y = 0 can be purely real

⇒|b| = 1 B is correct x = 0 can be purely imaginary


Must be real or purely imaginary.
6 . 8 0 | Complex Number

 x + 1 + 2i − 2  z = z implies z is purely real (B is correct)


Sol 32: (A, B) 1 − log2  ≥0
 2 − 1  z = −z implies z is purely imaginary (C is correct)

x + 1 + 2i − 2
⇒ log2 ≤1 Sol 37: (A, C)
2 −1 z2
⇒ |x + 1 + 2i| – 2 ≤ 2( 2 − 1 )

⇒ |x + 1 + 2i| ≤ 2 2 ⇒ (x + 1)2 + 4 ≤ 8

⇒ (x + 1)2 ≤ 4 ⇒ –2 ≤ x + 1 ≤ 2 z1 z3

⇒ –3 ≤ x ≤ 1 ⇒ x ∈ [–3, 1] z3 = (1 – z0)z1 + z0z2

z'3 = (1 – z0)z1' + z0z2'


Sol 33: (D) x = cos a
y = cos b
z1 z1 1
z = cos g z3 − z1 z'3 − z1'
z0 = = ⇒ z2 z2 1 =
0
z 2 − z1 z' − z'
Sx = cosα + cosβ + cosg 2 1
z3 z3 1
= cosα+cosβ+cosγ+i(sinα+sinβ+sinγ) angle is same
Px=cosa cosb cosγ = eiα eiβ eiγ = ei(α + β + γ) The two triangles are similar

π Sol 38: (A, B) Z multiplicative inverse is same as


Sol 34: (A, D) xr = cos  
 2r  additive inverse
 π π 
n i + ......  π π 
i + +  1
 r r +1  =−a − ib
Lim ∏xr = e2 2  =e2 4 
a + ib
n→∞
r =1
a − ib
π =−a − ib
i
 1 a2 + b2
2 1 − 
= e  2 = eiπ ⇒ a = –a3 – ab2 ⇒ a(1 + a2 + b2) = 0
= cosπ + isinπ = –1 + 0i ⇒ a = 0 and
Real part is –1 b = a2b + b3 ⇒ b(1 – a2 – b2) = 0
Imaginary part is 0 b ≠ 0 so a2 + b2 = 1 ⇒ b = ±1

n−1 z=0±i
Sol 35: (A, B, D) ∏ ( ω − zr )
r =1
Sol 39: (A, B, C, D) Z = a + bi =
(1 − ix ) (1 − ix )
ωn − 1 (1 + ix ) (1 − ix )
(w – z1) (w – z2) (w – z3)….. (w – zn-1) =
ω−1 1 − x2 − 2ix
a + bi =
If n is a multiple of 3, value can be zero as z1 can be w 1 + x2
or w2 ⇒ |z| = 1
If n = 3k + 1 value is 1  2x 
Arg z = tan–1  2 
If n = 3k + 2 value is 1 + w  x −1
Arg (z) ≡ ( −π, π 
Sol 36: (A, B, C) z = −4z
x – iy= –4x – 4iy
5x + i(3y) = 0
x = y = 0 (A is true)
M a them a ti cs | 6.81

Previous Years’ Questions ⇒ x + y =2  ….(ii)

Sol 1: Given, |z – 3 – 2i| ≤ 2  …. (i) And


To find minimum of |2z – 6 + 5i| Im ( z ) ≥ 1
5 ⇒ Im ( x + iy ) ≥ 1
Or 2 z − 3 + i ,
2
⇒ y ≥ 1  ….(iii)
By using triangle inequality

i.e., | z1 | − | z 2 | ≤ z1 + z2

5
∴ z −3+ i
2
5 9
= z − 3 − 2i + 2i + i = (z − 3 − 2i) + i
2 2

9
≥ | z − 3 − 2i | − From (i), (ii) and (iii) A ∩ B ∩ C has only one point P
2
shown in the figure.
9 5
≥ 2− ≥
2 2 Sol 4: (C) z + 1 − i + z − 5 − i
2 2

5 5
⇒ z − 3 + i ≥ or |2z – 6 + 5i| ≥ 5 2 2
2 2 = z − ( −1 + i ) + z − (5 + i )

The point A (-1, 1) and dB (5, 1) are end points of one


i
2π of the diameter
Sol 2: ω =e 3
In right angle ∆ APB
| x |2 + | y |2 + | z |2
Then, =3
| a |2 + | b2 | + | c |2
B(5,1)

Note: Here, ω = ei2π /3 , then only integer solution exists.

Sol 3: (B)
=A {z : Im ( z ) ≥ 1} A (-1,1)

( AB )
2
AP2 + BP2 =
B
= {z : z − 2 −=i 3}
{z : Re (1 − i) z + 2 }
2 2
=C ⇒ z +1 −i + z −5 −i

(=
6)
2
Taking z − 2 − i =3 = 36

Let z= z + iy
x + iy − 2 − i =3 Sol 5: (D) w − 2 − i < 3

2 From triangle in equality


⇒ ( x − 2) + i ( y − 1) =
(3)
2

z−w > |z|−|w|


⇒ ( x − 2) + ( y − 1) =
2 2
9  …(i)
⇒ − z − w < | z | − | w | < | z− w |  …(i)
And
Given w − 2 − i < 3 , which means that w lies inside the
Re (1 − i) z  =2 given circle. Since z is presented by point P and W has
Re (1 − i)( x + iy )  =2 to be the other end of diameter

| z − w | = length of diameter
6 . 8 2 | Complex Number

From (i), we get a u 1


−6 < | z | − | w | < 6 Consider b v 1
c w 1
⇒ − 6 + 3 < | z | − | w | + 3 < 6 +3
Applying R3 → R3 – {(1 – r) R1 + rR2}
⇒ −3 < | z | − | w | + 3 < 9
a u 1
5 5 = b v 1
 3 i  3 i
Sol 6: (B) Given, z =  +  + −  c − (1 − r)a − rb w − (1 − r)u − rv 1 − (1 − r) − r
 2 2  2 2
   
a u 1
 −1 + i 3 −1 − i 3 
=ω = and ω2  = b=v 1 0 (from eq. (i))
 2 2 
  0 0 0
3 +i  −1 + i 3 
Now, =–i   = −iω Hence, two triangles are similar.
2  2 
 

3 −i  −1 − i 3  6
 2kπ 2kπ 
And = i  = iω2 Sol 9: (D) ∑  sin − icos 
2  2  k =1  7 7 
 
 6 i2kπ 
∴ z = (– iω)5 + (iw2)5 = – iw2 + iω 6
 2kπ 2kπ  = −i  e 7 
∑ i  cos 7 + isin 7  ∑ 
( ) k =1   k =1 
= i (ω – w2) = i i 3 = – 3

⇒ Re (z) < 0 and Im (z) = 0  6 i2k π   6 i2k π 


∑ e = 0 = −i  ∑ e − 1 = −i 0 − 1  = i
k =0 7  k =0 7 
Alternate solution:
We known z + z =
2Re(z)
Sol 10: (C) Given, | z + iw |=| z − iw | 2 =2
5 5
 3 i  3 i ⇒ | z − ( −iw) |= | z − (iw) |= 2
If z =  +  + −  , then z is purely real,
 2 2  2 2 ⇒ | z − ( −iw) |= | z − ( −iw) | .
   
i.e. Im (z) = 0
iw iw
w
Sol 7: (D) Given, |z – 4| < |z – 2|
Since, |z – z1| > |z – z2| represents the region on right -1 1
side of perpendicular bisector of z1 and z2
w
∴ |z – 2| > |z – 4| -i w
⇒ Re(z) > 3 and Im (z) ∈ R
∴ z may take values given in (c).
y
Alternate solution
x’ y
x
O (2,0) (3,0) (4,0)

(-z)
y’ r -

x
O -

Sol 8: (B) Since a, b, c and u, v, w are the vertices of two r

triangles. (z)

Also, c = (1 – r) a + rb and w = (1 – r) u + rv … (i)


M a them a ti cs | 6.83

| z + iw |≤| z | + | iw | = | z | + | w |≤ 1 + 1= 2 Sol 12: (C) If in a complex number a + ib, the ratio a: b


∴ | z + iw |≤ 2 is 1 : 3 , then always convert the complex number in
⇒ | z + iw |=
2 holds when the form of ω .
Arg z – arg iw = 0 1 3
Since, ω= − + i
2 2
Similarly, when | z − iw |=2 334 365
 1 i 3  1 i 3
z ∴ 4 + 5 − +  + 3 − + 
Then is purely imaginary  2 2   2 2 
w  
Now, given relation
= 4 + 5ω334 + 3ω365
| z + iw |=| z − iw |=2 = 4 + 5 ⋅ (ω3 )111 ⋅ ω + 3 ⋅ (ω3 )121 ⋅ ω2
Put w = i, we get = 4 + 5ω + 3ω2 ( ω3 = 1)

| z + i2 |=z + i2 |=2 = 1 + 3 + 2ω + 3ω + 3ω2


⇒ | z − 1 |= 2 = 1 + 2ω + 3(1 + ω + ω2 ) = 1 + 2ω + 3 × 0
⇒ z= −1 (| z |≤ 1) = 1 + ( −1 + 3i) = 3i.
Put w = - i, we get
| z − i2 |=| z − i2 |= 2 Sol 13: (A) Since, arg (z) < 0
⇒ | z + 1 |= 2
⇒ arg(z) = −θ
⇒ =z 1 (| z |≤ 1) ⇒=z r cos( −θ) + isin(=
−θ) r(cos θ − isin θ)
and − z = −r[cos θ − isin θ]
∴ z = 1 or – 1 is the one correct option given.
= r[cos( π − θ) + isin( π − θ)]
∴ arg ( −z) = π − θ
Sol 11: (D)
Thus, arg (-z) –arg (z)
n n n2 n2
(1 + i) 1 + (1 − i) 1 + (1 + i) + (1 − i)
n n1 n n = π − θ − ( −θ) = π
= [ 1 C0 + C1i + 1 C2i2 + 1 C3i3 + ......]
n n n n Alternate solution:
+ [ 1 C0 − 1 C1i + i C2i2 − 1 C3i3 + ......]
 −z 
n n n n
+ [ 2 C0 + 2 C1i + 2 C2i2 + 2 C3i + .....] Reason: arg (-z) – arg z =arg   = arg( −1) =π
 z 
n n n n
+ [ 2 C0 − 2 C1i + 2 C2i2 − 2 C3i3 + ...]  z 
And also arg z – arg (-z) =arg   = arg( −1) =π
n
= 2[ 1 C0 +
n1
C2i2
n1
+ C 4 i4 + ....]  −z 
n n n
+2[ 2 C0 + 2 C2i2 + 2 C 4 i4 + ....]
= 2[
n1
C0 −
n1 n
C2 + 1 C 4= ....] Sol 14: (A, B,C) Since, z1 =
a + ib and z 2 =
c + id
n n n ⇒ | z1 |2 = a2 + b2 = 1 and | z 2 |2 = c2 + d2 = 1 ...(i)
+2[ 2 C0 − 2 C2 + 2 C 4 − ....]
(| z=
1 | | z=
2 | 1)
This is a real number irrespective of the values of n1 and n2 Also, Re (z1 z 2 ) =0 ⇒ ac + bd =0
Alternate solution a d
⇒ =− =λ (say)...(ii)
n n n n b c
{(1 + i) 1 + (1 − i) 1 } + {(1 + i) 2 + (1 − i) 2 }
From Eqs.(i) and (ii), b2 λ2 + b2= c2 + λ2c2
⇒ a real number for all n1 and n2 ∈ R.
2
n n ⇒ b= c2 and a=
2
d2
[ z=+ z 2Re(z) ⇒ (1 + i) 1 + (1 − i) 1
is real number for all n ∈ R] Also, given w1 =
a + ic and w2 =
b + id
Now, | w1 |= a2 + c2 = a2 + b2 = 1
| w2 |= b2 + d2 = a2 + b2 = 1
6 . 8 4 | Complex Number

and Re(w1 w2 )= ab + cd= (bλ )b + c( −λc) aq = 4 and 2a = 10


4
λ (b2 − c2 ) =
= 0 ⇒e=
5
B→p
(1 − t)z1 + tz 2
Sol 15: (A, C, D) Given, z = 1
(1 − t) + t (C) z= w −
w
A P B 1
⇒ z = w− ×w
z1 z z2 ww
t : (1 - t) w
= w−
| w |2
Clearly, z divides z1 and z 2 in the ratio of t:
w
= w−
(1-t), 0 < t < 1 4
⇒ AP + BP = AB Let w = p + I q, then
ie, | z − z1 | + | z − z 2 |= | z1 − z 2 | 3P 5q
z
= +i
⇒ Option (a) is true. 4 4
Now, let z = x + iy
And arg (z − =
z1 ) arg (z 2 −=
z) arg(z 2 − z1 )
3P 4x
⇒ x= ⇒ P=
⇒ (b) is false and (d) is true. 4 3
5q 4y
Also, arg (z − z1=
) arg(z 2 − z1 ) ⇒ y= ⇒ q=
4 5
 z − z1 
⇒ arg  0
 = | w |2 = P2 + q2 = 4
 z 2 − z1 
z − z1
∴ is purely real.
z 2 − z1

z − z1 z − z1 z − z1 z − z1
=
⇒ or= 0
z 2 − z1 z 2 − z1 z 2 − z1 z 2 − z1

Sol 16: (A) | z − i | z ||=


| z + i | z || 16 x2 16 y 2
⇒ + 4
=
9 25
⇒ | z − i | z ||2 =| z + i | z ||2
x2 y2
⇒ ( z − i | z |) ( z + i| z |) = ( z + i | z |) ( z − i | z |) ⇒ + 1 , its an ellipse
=
9 / 4 25 / 4
⇒ z z + i z| z | − i | z | z + | z |2 b2
e2= 1 −
a2
=z z − i z | z | + i| z | z + | z |2
9 16
⇒ 2i z | z |=
2i| z | z =1− =
25 25

⇒ 2i | z | ( z − z ) =
0 4
⇒e=
5
| z | 0 or z=
⇒= −z 0
From figure, we can conclude that
⇒ Im ( z ) =
0
Re ( z ) ≤ 2
Also Im ( z ) ≤ 1

A → q, r And z ≤ 3

(B) |z + 4| + |z - 4| = 0
C → p, s, t
Its an equation of ellipse having
M a them a ti cs | 6.85

1 w w z − z1 z 2 − z1
(D ) z= w + w
=+ w
=+ w w
=+ =
w ww | w |2 z − z1 z2 − z1
Let w = a + i b z − z1 z − z1
⇒ 0
=
z = a + i b + a – i b = 2a z 2 − z1 z2 − z1

Now, |z| = 2 |a| and Im (z) = 0, Im ( z ) ≤ 1 Sol 19: 2z − 6 − 5i


⇒ Re ( z ) ≤ 2 and | z | ≤ 3 5
= 2 z −3+ i
2
D → q, r , s and t
 5 
Here z −  3 − i  is nothing but the distance between
Sol 17: (B)  2 
 −5 
z +1 ω ω2 any point on the circle z − 3 − 2i ≤ 2 and point  3, 
 2 
ω z + ω2 1 =0
2
ω 1 z+ω
C1 → C1 + C2 + C3 (3,2)

z ω ω2
z z + ω2 1 =0
z 1 z+ω

 2π 2π )3, -5 )
=ω cos + i sin 2

 3 3 
1 ω ω2 5
∴ Minimum value =
2
1 z + ω2 1 =0
1 1 z+ω 5
2z − 6 + 5i = 2× =5
min 2
Expanding the determinant, we get
Sol 20: Given: a + b + c = x
 (
z  z + ω2 ) ( z + ω) − 1 − ω ( z + ω − 1) + ω (1 − z − ω ) =0
2 2
a + b ω + c ω2 = y
⇒ z  z 2 + z ω + zω2 + ω2 − 1 − zω − ω2 + ω + ω2 − ω2 z − ω4  =0
  a + b ω2 + c ω = z
⇒ z  z2  =
0 | x |2 + | y |2 + | z |2 xx + yy+zz
  =
2 2 2
|a| + |b| + |c| | a |2 + | b |2 + | c |2
⇒z=0

Sol 18: (A, C, D) Given z =(1 − t ) z1 + t z2


( ) (
= ( a + b + c ) a + b + c + a + b ω + c ω2 + a + b ω+ c ω2 ) ( )
This equation represents line segment between z1 and ( 2
+ a + bω + cω a + bω + c ω)( 2
)
z2 | a | + | b | + | c |2
2 2

From fig.  i π /3
 ω =e
| z − z1 | + | z − z 2 |= | z1 − z 2 |  −i π /3
 ω =e
 π
ω2 = e2i 3 = − ω

( )
= ( a + b + c ) a + b + c + ( a + b ω− ω c ) a + b ω + c ω2 ( )
Arg ( z − z=
1) Arg ( z 2 − z1 ) (
+ ( a − b ω + c ω) a + b ω + c ω 2
)
| a | + | b | + | c |2
2 2
6 . 8 6 | Complex Number

(
3 | a |2 + | b |2 + | c |2
= 3
) 1 1 
4r 2
⇒  − z 0   − z0  =
 α  α 
| a |2 + | b |2 + | c |2
 1 z z 
⇒  − 0 − 0 + | z 0 |2  =
4r 2
 α α α α 
1 a b
   | α |2 
Sol 21: (A) Given  ω 1 c 2
α α = | α | ⇒ α = 
ω2  α 
 ω 1 
1 z0 α z0 α
1 a b ⇒ − − + | z 0 |2 =
4r 2
2 2 2
  |α| |α| |α|
Determinant (D )=  ω 1 c
ω2 ω 1 
 ⇒ 1 − z 0 α − z0 α+ | α |2 | z 0 |2 = 4r 2 | z |2  …(ii)
|= (
D | 1 (1 − c ω ) − a ω − c ω 2
) + b (ω2
−ω 2
) Subtracting (ii) from (i), we get
⇒ | D | = 1 − ( a + c ) ω + acω 2
1− | α |2 + | α |2 | z 0 |2 − | z 0=
|2 r 2 4 | α |2 −1 ( )
| D | ≠ 0 , only when a = ω and c = ω
r2 + 2
(
∴ ( a,b,c ) ≡ ( ω, ω, ω) or ω, ω2 , ω ) 1− | α |2 +
2
(
| α |2 −= )
1 r 2 4 | α |2 −1 ( )
∴ Two non-singular matrices are possible.  r2 + 2  2
(
⇒ | α |2 −1 
 2
)
−1
=

2
 r 4 | α | −1 ( )
Sol 22: (D) a = z 2 + z + 1 = 0  r2  2
⇒ z2 − z + 1 − a =0 (
⇒ | α |2 −1  =
2
 
) 2
 r 4 | α | −1 ( )
Discriminant < 0 {Since imaginary part of z us not zero}
⇒ | α |2 −1 = 8 | α |2 −2
1 – 4 (1 - a) < 0
⇒ 7| α |2 =1
⇒ 1 − 4 + 4a < 0
1
⇒ 4a < 3 ⇒ | α |=
7
3
⇒a <
4 Sol 24: (C,=
D) P {=
w :n n
1,2,3,..... }
3 1
Sol 23: (C) ( x − x0 ) + ( y − y 0 ) =
2 2
r w= + ⇒ | w |= 1
2 2
⇒ | z − z 0 |=
r ⇒ All the complex number belong to set P lie on circle
of unit radius, centre at origin.
α lies on it, then

| α − z 0 |=r

⇒ ( α − z 0 ) ( α − z0 ) =r 2

| |2 − z0 α − z 0 α + | z 0 |2 =
=α r2  ….(i)

Similarly,

( x − x0 ) + ( y − y 0 )
2 2
2r
=

⇒ | z − z 0 |=
2r
1
lies on it, then
α
1
− z0 =
2r
α
M a them a ti cs | 6.87

All the complex number belong to PnH1 lie right of the  


 z − 1 + 3i 
1 S2 =
line x = on circle. Possible positions are shown in the z ∈ C : Im   > 0
2   1 − 3 i  
figure as A1 , A2 , A3 .

Similarly, all the complex number belong to PnH2 lie left (


 z −1 + 3i 1 + 3i 
Im   >0 )( )
−1 4 
of the line x = on circle. Possible positions are shown
2  
in the figure as B1 ,B2 ,B2 < z1 0 z 2 = angle between A2 Let z = x + i y
2π 5π
and B3 =
3
or angle between A1 and B3 =
6 ( ) (
 ( x + iy ) 1 + 3 i − 1 − 3 i 1 + 3 i 
⇒ Im   >0 )( )
4 
 

( ) (
 x + 3 y + i 3x + y − 4 
⇒ Im   >0 )
4 
 

3x + y
⇒ > 0 ⇒ 3x+ y > 0
4
{
S3 z ∈ C : Re ( z ) > 0 }
⇒ z lies in either first or fourth quadrant.
Now, points of intersection of circle x2 + y 2 =
16 and

Sol 25: (B, C, D) P = pij  pij = ωi+ j 3x + y =0 is x2 + 3x2 =


16
n×n
⇒ 4x2 =
16 ⇒ x =± 2
 ω2 ω3 ω4  ω 
n+1
  ⇒ y =± 2 3
ω3 ω4 ω5  ωn+ 2 
P=  1 2 5π
      Area =
2
( 4) × 6
ω2n+1 ωn+ 2 ωn+3  ωn+n 
  1 5 π 20 π
= × 16 × = .sq. unit2
2 6 3
ω4 + ω6 + ω8 + .... ω5 + ω7 + ω9  
 
    Sol 27: (C) | 1 − 3i− z |
⇒ P2 =
    
  = z − 1 + 3i = z − (1 − 3i )
n 3 n 5
 ω + ω + .....
+ +  

Lets take element P11 =ω4 + ω6 + ω8 + ....n terms ( )


Min z − (1 − 3i) = distance of point (1, -3) from line

=
(
ω4 ω2n − 1 ) 3 x+y =0

ω−1 3×1 − 3 3−3


= =
If n is multiple of 3, then this element will vanish. Which 3+1 2
is the case for every element.
3− 3
=
∴ n can not be multiple of 3, for P2 ≠ 0 2
∴ Possible values of n are 55, 58, 56
2πx 2k π
Sol 28: (C) zk =
cos + i sin ,k =
1,2,....9
10 10
{z C :| z | < 4}
Sol 26: (B) S1 =∈
i
2 πk
(p) zk = e 10
z lies inside of a circle given by |z| = 4
6 . 8 8 | Complex Number

2π π π π
i (k + j) 12 ik i 12 i
⇒ zk .z=
j e 10 = 1 ∑ e 7 e 7 −1 ∑ e 7 −1
k =1 k =1
2π 2π = =
⇒ cos
10
( k + j) + i sin
10
(k + j ) =
1 3 i
( 4k −2) i
π 3 i
π

∑e 7 e 7 −1 ∑e 7 −1
If k + j = 10 m (multiple of 10), then above equation is k =1 k =1

True.

(q) z1 .z = zk 12
π π
2 πk
∑ 2 sin 14 12 × 2 sin
i (k −1) = k =1
= 4 =4
zk e 10 i 2π 3 π
⇒ z= = = e 10 π
z1 i
2π ∑ 2 sin 14 3 × 2 sin
4
e 10 k =1

Clearly, this equation has many solutions


 −z r z 2s 
⇒ Q is False Sol 30: P = 
( )
 z 2s zr 
(r) Consider, z10 = 1 

z10 − 1 = ( z − 1) ( z − z1 )( z − z2 )( z − z3 ) .... ( z − z9 )
10
 −z 2r + z 4s
P = 
2
( ) ( ) ( −z ) z 2s + zr z 2s 
r

Where zk represents the roots of equation z = 10 
( −z ) z + z z
r 2s r 2s 2r 4s
z +z 
=1 + z + z 2 + .... + z 9
 
z 2s ( −z ) + zr  
2r 4s r


Z10 − 1
( z − z1 ) ( z − z2 ) ...... ( z − z9 )
=  z +z  
z −1 = 
 z 2s  ( −z )r + zr  z 2r + z 4s 
= 1 + z + z 2 + z3 + .... + z 9    

z10 − 1
( z − z1 ) ( z − z2 ) .... ( z − z9 )
= 1 0  2r 4s
z −1 Given that P2 − I =  ⇒ z +z = −1
 0 1 
= 1 + 1 + 1 …… 10
= 10 2s  r
And z ( −z ) + z  =
r
0
10  
(s) We know that sum of roots of z − 1 =0 is zero
−1 + i 3
2k π9 9
2k π Now, we have z = =ω
⇒ 1 + ∑ cos 0 ⇒ ∑ cos
= −1
= 2
k =1 10 k =1 10
⇒ ω2r + ω4s = −1 and ω2s ( −ω) + ωr  = 0
r
9
2k π  
⇒ 1 − ∑ cos =2
k =1 10 Only (r, s ) ≡ (1, 1 ) satisfies both the equation.
kπ Only one pair exists.
kπ kπ i
Sol 29: Given
= αk cos + i sin =e 7
7 7
 1 
Sol 31: (A, C, D) S =z ∈ C : z = , t ∈R, t ≠ 0 
12 i
(k +1) π i
kπ  a + ib t 
12
∑ | αk +1 − αk | ∑e 7 −e 7
1 a−i b t
k =1
k =1 =z ×
= a + ib t a − ib t
3
3 i ( 4k −1 )
π ( 4k −2)
∑ | α 4 k −1 − α 4k −2 | ∑e 7
i
−e 7
a−i b t
k =1 ⇒z= = x + i y (Let)
k =1
a2 + b2 t2
a bt
=⇒x = and y
22 2
a +b t a + b2 t 2
2
M a them a ti cs | 6.89

x a ay
⇒ = ⇒t=
y bt bx
a
Substituting ‘t’ in x =
a + b2 t 2
2
a
x=
a2 y 2
a2 + b2
b2 x 2

ax2
⇒ x= ⇒ a2 x2 + a2 y 2 − ax =
0
a2 x2 + a2 y 2
x
⇒ x2 + y 2 − 0
=
a
2 2
 1  2 1 
x −  + y = 
 2a   2a 

 1 
Centre ≡  , 0 
 2a 
1
Radius = , when a > 0 , b ≠ 0
2a

If=
b 0, a ≠ 0
y = 0 ⇒ x − axis
If=a 0, b ≠ 0
x = 0 ⇒ y − axis

You might also like